Manual of PrescriptionWriting
Manual of PrescriptionWriting
Manual of PrescriptionWriting
Prescription
Writing
with Case Scenarios
As per CBME Guidelines | Competency Based
Undergraduate Curriculum for the Indian Medical Graduate
t.me/Dr_Mouayyad_AlbtousH
t.me/Dr_Mouayyad_AlbtousH
Manual of
Prescription
Writing
with Case Scenarios
As per CBME Guidelines | Competency Based
Undergraduate Curriculum for the Indian Medical Graduate
Saroj Kothari
MBBS, MD (Pharmacology)
Professor and Head
Department of Pharmacology
Gajra Raja Medical College, Lashkar, Gwalior, MP
e-mail: saroj.kothari@rediffmail.com
t.me/Dr_Mouayyad_AlbtousH
Disclaimer
Science and technology are constantly changing fields. New research and experience broaden the scope of
information and knowledge. The authors have tried their best in giving information available to them while
preparing the material for this book. Although, all efforts have been made to ensure optimum accuracy of the
material, yet it is quite possible some errors might have been left uncorrected. The publisher, the printer and
the authors will not be held responsible for any inadvertent errors, omissions or inaccuracies.
eISBN: 978-93-546-6230-0
Copyright © Authors and Publisher
All rights reserved. No part of this eBook may be reproduced or transmitted in any form or by any means,
electronic or mechanical, including photocopying, recording, or any information storage and retrieval system
without permission, in writing, from the authors and the publisher.
Head Office: CBS PLAZA, 4819/XI Prahlad Street, 24 Ansari Road, Daryaganj, New Delhi-110002, India.
Ph: +91-11-23289259, 23266861, 23266867; Fax: 011-23243014; Website: www.cbspd.com;
E-mail: publishing@cbspd.com; eduportglobal@gmail.com.
Branches
Bengaluru: Seema House 2975, 17th Cross, K.R. Road, Banasankari 2nd Stage, Bengaluru - 560070,
Kamataka Ph: +91-80-26771678/79; Fax: +91-80-26771680; E-mail: bangalore@cbspd.com
Chennai: No.7, Subbaraya Street Shenoy Nagar Chennai - 600030, Tamil Nadu
Ph: +91-44-26680620, 26681266; E-mail: chennai@cbspd.com
Kochi: 36/14 Kalluvilakam, Lissie Hospital Road, Kochi - 682018, Kerala
Ph: +91-484-4059061-65; Fax: +91-484-4059065; E-mail: kochi@cbspd.com
Mumbai: 83-C, 1st floor, Dr. E. Moses Road, Worli, Mumbai - 400018, Maharashtra
Ph: +91-22-24902340 - 41; Fax: +91-22-24902342; E-mail: mumbai@cbspd.com
Kolkata: No. 6/B, Ground Floor, Rameswar Shaw Road, Kolkata - 700014
Ph: +91-33-22891126 - 28; E-mail: kolkata@cbspd.com
Representatives
Hyderabad
Pune
Nagpur
Manipal
Vijayawada
Patna
t.me/Dr_Mouayyad_AlbtousH
Preface
A t the end of the second phase, it is found that most medical students are not
confident of how to prescribe a drug to the patient or what information they
need to provide. This is because earlier pharmacology training has focused more on
theory than on practice. This method of teaching although helped in acquiring
knowledge of pharmacology but failed to impart practical prescribing skills. Now
National Medical Commission has introduced Competency Based Undergraduate
Curriculum in 2019 therefore a need is felt to write this manual to impart competency
of prescription writing in medical undergraduate second phase students. The
knowledge students gain from textbook is difficult to retain and even more difficult is
to recall this information when the clinical setting is encountered. Revised
undergraduate medical education regulations (GMR, 2017) have focused on problem-
based learning approach hence carefully constructed clinical case scenarios are given
to teach prescription writing in this manual to fill the gap between basic science and
clinical exposure. With the availability of so many drugs, at times many drug options
are found suitable for a particular disease and the physicians choose the drug according
to their experience. The general perception in students is that they will learn prescribing
skills after completing the fourth phase of MBBS but research shows that there is a
gain in experience, but prescribing skills do not improve much after graduation. It is
well recognized that drug errors are a major cause of patient’s morbidity and mortality,
so good training is needed before poor habits get a chance to develop. To impart
prescribing skill in students this manual is written with the following learning
objectives:
1. Students should be able to understand the general principles of prescribing and
able to write a complete and legible prescription for a given condition.
2. Students should be aware of some common errors of prescribing and able to correct
them appropriately.
3. Students should be able to communicate with patients on prescriptions
4. Students should be able to perform a critical appraisal of a given prescription/
should know about prescription audit.
5. Students should be able to understand the legal and ethical aspects of prescribing.
This book covers the latest drugs to be prescribed for various diseases. It reflects
credible information also on other suitable alternative options.
Medicine is an ever-changing science. With the day-to-day advancement in research
and clinical experience, we need to update options of drug therapy. The authors and
the publisher of this work have put in their best efforts to provide information that is
t.me/Dr_Mouayyad_AlbtousH
vi Manual of Prescription Writing
complete and updated in accord with the standards accepted at the time of publication.
Despite all efforts, there is the possibility of certain human errors or changes in medical
sciences may have been overlooked. Therefore, neither the authors nor the publisher
warrants that the information contained herein is in every respect accurate or complete,
and they disclaim all responsibility for any errors or omissions or for the results obtained
from the use of the information contained in this work. We wish the readers to confirm
the information with other sources and refer to the manufacturer’s recommendations
for all dosages, especially for the new drugs. This book will serve as a resource for
medical students/professionals, dental students, nurses, pharmacists and
physiotherapists.
We acknowledge the feedback provided by our undergraduate, postgraduate
students, pharmacology and medicine faculty members for the presentation of this
manual. We are pretty sure that students will find this manual a tool to achieve the
competency of safe prescribing. The authors would be grateful for any comments or
constructive criticism to ensure that this manual continues to evolve and remains
adapted to the reality of the field.
Saroj Kothari
Ashok Kumar Jain
t.me/Dr_Mouayyad_AlbtousH
General Considerations in Prescription Writing vii
Acknowledgements
W ith deep gratitude and respect we remember almighty God for giving us the
ability to understand, remember and write a book to impart prescribing skills in
medical students.
We are thankful to our parents for their constant encouragement, love and care.
We are grateful to our teachers.
We are indebted to Professor (Dr) SP Dhaneria Sir, Academic Dean, Professor and
Head, Department of Pharmacology, All India Institute of Medical Sciences, Raipur
(CG), who consented to review the book and thankful to him for his valuable
suggestions, support and encouragement.
We wish to express our thanks to our families for their support and encouragement.
We sincerely thank our departmental colleagues and postgraduate students for
assistance in preparing different chapters.
We wish to express our gratitude to Mr YN Arjuna (Senior Vice President—
Publishing, Editorial and Publicity) of CBS Publishers & Distributors Pvt. Ltd. for
accepting our proposal, continuous support, innovative ideas, extraordinary
cooperation for publishing this book.
Saroj Kothari
Ashok Kumar Jain
t.me/Dr_Mouayyad_AlbtousH
t.me/Dr_Mouayyad_AlbtousH
Contents
Preface v
Abbreviations xi
Background 1
Elements of Prescription 1
Prescription Format 3
Model Prescription 4
Important Points for Correct and Safe Prescribing 4
Introduction 8
Prescriptions for Case Scenarios of Common Diseases 9
t.me/Dr_Mouayyad_AlbtousH
x Manual of Prescription Writing
Background 168
Pros and Cons of Off Label Prescribing 168
Proposed Points of Consideration for Appropriateness of
Off Label Prescribing 169
When Off Label use is Illegal or Problematic? 170
Bibliography 171
Index 173
t.me/Dr_Mouayyad_AlbtousH
General Considerations in Prescription Writing xi
Abbreviations
t.me/Dr_Mouayyad_AlbtousH
t.me/Dr_Mouayyad_AlbtousH
1
General Considerations in
Prescription Writing
Background
Once a patient is diagnosed with a disease and the practitioner opts for drug therapy,
he has to select drugs from the available therapeutic options. This requires the writing
of a prescription. A written prescription is an instruction or prescriber’s order to a
pharmacist to prepare and dispense a specific medicine for a particular patient.
Prescriptions in the olden days were compounded and pharmacist used to prepare
dosage forms. In modern times prescription drugs are precompounded and
pharmaceutical companies supply different dosage forms of the drugs as per need.
Presently prescription is an instruction from a prescriber to the patient and the
dispenser.
The art of prescription writing is ancient. Originally its process was a little complex
and written in the Latin language, subsequently replaced by English. The contemporary
practice of prescribing is more systematic and simple. Every country has its own
regulations of prescription; in fact there are no universal standards for this. Prescription
is a legal document and is prima facie evidence in the court of law for which the
prescriber is responsible. More often it is found that incomplete and incorrect
prescriptions are handed over to the patients and also overlooked by pharmacists.
This trend is putting patient safety at risk.
Prescription is an important intervention by the physician and it is the ethical and
legal duty of the practitioner to write clearly and legibly. It should indicate precisely
the drugs, dosage, route, timings and total quantity to be supplied.
Elements of Prescription
A prescription should be written in a systematic order. The correct prescription should
contain the following elements:
1. Doctor’s particulars: Name, professional degrees, address, telephone number and
E-mail address of prescriber should be written or printed on top of the prescription
paper. This will help the patient or pharmacist to contact the physician for any
query or clarification. In government setup name of the hospital is printed on top of
the prescription and the prescriber does his signature at the bottom. The size of the
prescription paper should not be very small. The ideal minimum size is 14 × 21 cm
(A5 size).
t.me/Dr_Mouayyad_AlbtousH
2 Manual of Prescription Writing
2. Date: Prescriptions should be dated at the time they are written. The date is important
in creating the medication record of the patient and to check his compliance
behaviour. The date is also important to pharmacist for dispensing of drugs
prescribed under the Narcotic Drugs and Psychotropic Substances (NDPS) Act. To
mention time along with date may be especially important while writing
prescriptions for certain ailments, e.g. in case of poisoning or serious accidents/
illnesses.
3. Patient’s particulars: Name, age, sex, weight and address of the patient should be
written after prescriber’s particulars. Patient’s age, especially in paediatric and
geriatric populations is required for proper drug and dose assurance. This step also
helps to avoid age-inappropriate prescriptions. Patient's particulars are required so
that the correct medication goes to the proper patient and also for identification
and record-keeping purposes. Address of the patient is required so that he/she can
be contacted if the need arises.
4. Superscription: It is a prescription symbol “RX” written on the top left side of the
body of prescription. This has been accepted all over the world as an invocational
sign to the Jupiter who is considered as God of healing. The sign of Jupiter is
employed as a request for healing. It represents the specific Latin verb “recipe”
which means “take thou” or for you in the name of God.
5. Inscription: This is the most important part of the prescription. In this part, prescriber
writes names of selected drugs in an order. The drug name should be written in
capital letters. It is preferable to write generic names of drugs unless there are
bioavailability issues. It is also important to mention medication strength, quantity
to be taken and the duration. Computer typed prescriptions avoid errors due to
bad handwriting.
6. Subscription: In this part of the prescription prescriber used to provide direction to
the pharmacist regarding dispensing of preparation. Examples of prescription
directions to the pharmacist are “make a mixture,” “mix and place into 30 capsules".
This part is no more applicable these days. Today, the majority of the prescriptions
are written for medications already prepared as ready dosage forms by drug
manufacturing industries.
7. Transcription: In this part, instructions to the patient are given like, to swallow or
chew, shake well before use, to be taken with meals or on empty stomach, etc. This
part comes just below the inscription. Renewal or refill instructions are written on
the left side at the bottom. It is particularly important in the treatment of chronic
diseases when the treatment is to be continued for a longer period. If refills are not
allowed it should be clearly documented in prescription especially when drugs
causing dependence are prescribed. In modern practice inscription, subscription
and transcription are combined together.
8. Signature: The prescription ends with the signature of the doctor, date with his
registration number and name of State Council or National Medical Commission
under which he or she is registered. This will validate and authenticate the
prescription. No blank space should be left between the body of the prescription
and signature.
t.me/Dr_Mouayyad_AlbtousH
General Considerations in Prescription Writing 3
PRESCRIPTION FORMAT
t.me/Dr_Mouayyad_AlbtousH
4 Manual of Prescription Writing
MODEL PRESCRIPTION
A 30-year-old patient Mr. X weighing 45 kg was diagnosed as a case of generalized
tonic clonic seizures. Write a prescription for this patient.
Dr XY Jain
MBBS, MD (General Medicine)
Registration no. MPMC 223344
Family Wellness Clinic, Fort View, Gwalior, M.P.
Tel. no. 111222 Email: ---------@-------.com
Date: 10.01.2021
Patient’s name: Mr X
Age & Sex: 30 years, Male
Weight: 45 kg
Address: 24, Modern Colony, Gwalior
RX
Tab PHENYTOIN SODIUM 100 mg orally 12 hrly for 7 days
Review after 7 days
Avoid driving vehicles and operating machinery
Refills: None
XY Jain
Stamp
t.me/Dr_Mouayyad_AlbtousH
General Considerations in Prescription Writing 5
the prescriber and also the professional who administered the drug (pharmacist/
nurse). Side effects, adverse reactions, toxicity, drug interactions, and
contraindications should be kept in mind while writing a prescription. It is, therefore,
the doctor must take utmost precaution to make all prescriptions very clear and
informative, as any mistake could be directly attributed to the prescriber however
good the intention may be.
2. Name of drug: The international non-proprietary name or generic name of the drug
should always be used. The trade name can be added if there is a specific reason to
prescribe a special brand, e.g. drug with a specialized release system or a drug
with a narrow therapeutic index. If it is handwritten prescription the name of the
drug should be in capital letters so that the chances of errors due to bad handwriting
are avoided however typed or computer generated prescriptions are better. Always
write the correct spelling of the drug’s name. Do not use abbreviations for drug
names as these can be misinterpreted. For example, write Hyoscine methyl bromide,
not HMN, Isosorbide dinitrate not IDN.
3. Approach medication names with caution: Names of drugs that look alike when
spelt out and sound alike when spoken may contribute to dispensing errors, e.g.
a. Normace: Norpace (Normace is Enalapril which is an antihypertensive drug while
Norpace is Disopyramide which is an antiarrhythmic drug)
b. Neocare: Neocalm (Neocare is Levodopa + Carbidopa used in Parkinsonism while
Neocalm is Trifluoperazine used in Schizophrenia).
c. Isodril: Isoprine (Isodril is Isosorbide dinitrate used in angina pectoris while
Isoprine is Isoprenaline used in bradycardia).
4. Pharmaceutical dosage form of the drug: The pharmaceutical dosage form should
be clearly stated, e.g. ‘tablet’, ‘oral solution’, ‘eye ointment’, ‘vaginal pessary’, etc.
Each type of dosage form has unique pharmaceutical characteristics and designed
for optimal drug action. When oral and injectable drugs are equally effective
parenteral administration is only justified in case of emergency or when the patient
is unable to take orally.
5. Dosage unit: The strength of the drug should be stated in standard units using
abbreviations that are consistent with the System International (SI). Avoid common
household measurements such as “teaspoons” or “tablespoons” as these are variable
and cannot be reliable. ‘Microgram’ should not be abbreviated to μg, e.g. Clonidine
100 microgram or mcg but not μg. Also, ‘units’ should not be abbreviated, e.g.
Injection Penicillin 4 Million Units, not MU to avoid confusion because U may be
interpreted as 0 or 4.
6. Use of decimal points: Many errors in drug strength have occurred with unnecessary
decimal points, e.g. write 100 mg, not 0.1 g, use leading zero before decimal point
like write 0.4 mg and not .4 mg and avoid terminal zero after decimal point, e.g.
write 8 mg, not 8.0 mg because it can resemble 80. When the strength of the drug is
more than 1 g write the unit gram and for strength of the drug less than 1 g write
the unit milligrams.
7. Dose frequency: The use of abbreviations in medication's frequency of administration
is also a common source for medical errors. OD, BD, TDS and SOS though are
t.me/Dr_Mouayyad_AlbtousH
6 Manual of Prescription Writing
acceptable Latin abbreviations may work if all your patients are Latin scholars.
Usually, these abbreviations confuse most of the patients. For example, it is better
to write 12 hourly for BD. The drug with less frequent dosing should be preferred
for better patient compliance. Dose frequency should be clear and explicit. The
minimum dose interval should be stated together with the maximum daily dose
for preparations given on ‘as required’ basis, where relevant. It is good practice to
qualify such prescriptions with the purpose of the medication (for example ‘6 hourly
as required for pain’, ‘at night as required for sleep’). Always specify times (6 a.m.,
2 p.m., 10 p.m.) rather than simply frequency (three times a day) and especially
relationship to meals for orally consumed medication.
8. Duration of therapy: Specify the therapeutic duration correctly. The number of
prescribed pills should be matched to the expected duration of treatment and
necessary reassessment visits. Write review dates for all antibiotic prescriptions.
9. Directions for patients: Write directions in easy English or in a language that the
patient understands better. Avoid writing misleading or inappropriate directions
such as ‘take as directed’ or ‘take as before’. Provide clear and specific directions to
reinforce treatment plans. The directions should be both drug-specific and patient-
specific. Add additional instructions which guide patients to avoid medication
adverse effects, e.g. take it with meals (Indomethacin), take the first dose before
going to bed (Prazosin) and take while going to bed, e.g. Lorazepam.
10. Indication: It is mostly encouraged to write the indication of prescription. It helps
the pharmacist to dispense appropriate medications and the patient to understand
the drug’s purpose.
11. Avoid polypharmacy: Limit each prescription to a minimum number of medications,
since multiple drugs and instructions may overlap and confuse the pharmacist
and the patient. It is likely that patients with multiple illnesses require multiple
drugs even then attention should be paid to restrict polypharmacy.
12. Communication with patients: For proper use of prescribed medicines and better
compliance communicate effectively with patients.
13. Refills and repeat prescription: It is necessary to ensure regular review of the patient
for any adverse effects/ side-effects, ongoing need for this medication and if there
is a need to change doses over time. The minimum duration between refills and
number of refills should be clearly mentioned. Mention in prescription “do not
refill” in case of drugs causing dependence.
14. Age of the patient: Do not forget to write the patient’s age in order to check the
correct dose. With the change in age there is a change in weight and body surface
area, further in old age there is decline of renal and hepatic functions; all these age-
related factors need to be considered for dose adjustment. Selection of drug is also
age related, e.g. tetracycline is avoided in a child less than 8 years of age and lithium
is avoided in the elderly. The patient’s age orients pharmacists/chemists during
their checks of appropriate drug and dose.
15. Record keeping: Always write prescriptions with ink which is indelible or cannot
be cleaned or washed out. Any alterations should only be made in exceptional
circumstances by rewriting and must be countersigned by the concerned doctor.
t.me/Dr_Mouayyad_AlbtousH
General Considerations in Prescription Writing 7
t.me/Dr_Mouayyad_AlbtousH
8 Manual of Prescription Writing
INTRODUCTION
The medical world is expanding at a very fast pace. New drugs are continuously being
developed and introduced in clinical practice to replace obsolete ones. A busy
practitioner or a new degree holder in the medical profession should keep him abreast
of these developments and reorient to the changing circumstances. Drug resistance is
usually being encountered and is the main explanation for treatment failure.
In this small work, an attempt has been made to prescribe the most effective drugs
for a specific disease. In the case of intolerance or allergic reaction or drug resistance,
alternative regimens are also mentioned. However, the choice of drugs for a particular
patient in a particular situation depends upon the treating physician.
Case scenarios of common ailments are given in alphabetical order for early clinical
exposure of students and to initiate the thought process to make correct diagnosis and
then to write drug/drugs of choice for that particular disease. The medications and
the direction to the patients are written in a prescription format for a particular disease
depicted in the case scenario. The doctor’s particulars, patient's particulars, date, signature
and registration number of prescriber that are other essential elements of prescription are also
mentioned for better understanding. Along with main drug therapy, supplementary drugs
are given for symptoms like fever, cough or vomiting, on as required basis in a particular
case. Students should write complete prescription incorporating all the elements of
prescription.
Drugs are prescribed preferably using generic names. Injectables are prescribed only
when required and should be replaced by suitable oral drugs as soon as the patient is
stable. As a general principle injectables are advised for a maximum period of 4 to
6 weeks and then therapy should be switched over to oral drugs if treatment is to be
continued for a longer duration.
Reviews are also mentioned in those prescriptions where dose adjustment/
escalation/omission or addition of drugs are required during follow up care.
Refills are mentioned where the same treatment is to be continued after evaluation
of the patient.
t.me/Dr_Mouayyad_AlbtousH
Case Scenarios and Prescription Writing 9
CASE SCENARIO 1
A 10-year-old boy named X presented to a neurologist after an episode of loss of
consciousness that lasted for 10–20 seconds, with a blank stare, giddiness, eyelid
fluttering and lip-smacking and dimness of vision. The patient reported that he regained
from his loss of consciousness after a few seconds, however, the giddiness persisted.
He experienced similar episodes quite sometimes in the past six months. His
electroencephalogram (EEG) tracing showed a characteristic 3-Hz spike and dome
pattern. Write a prescription for this patient incorporating all the elements.
Dr XY Jain
MBBS, MD (General Medicine)
Registration no. MPMC 223344
25, Park Street, Gwalior MP
Tel. no. 111222 Email: -------@-----.com
Date: 10.01.2021
Patient’s name: Mr X
Age & Sex: 10 year, Male
Weight: 25 kg
Address: 24, Modern Colony, Gwalior
RX
Tab Sodium Valproate 125 mg orally 8 hrly for 7 days
Review after 7 days
Do not stop the drug without consulting doctor
Treatment is to be continued for 2–3 year after the last episode of seizure
XY Jain
Stamp
Alternative Agents
Tab Ethosuximide 250 mg orally 12 hrly for 7 days OR
Tab Divalproex Sodium 125 mg orally 8 hrly for 7 days
t.me/Dr_Mouayyad_AlbtousH
10 Manual of Prescription Writing
CASE SCENARIO 2
A 22-year-old female patient consulted a dermatologist with the complaints of multiple
lesions over cheeks, chin, and forehead. She has had acne since she was a teenager and
was treated with topical antibiotics and sunscreens without significant effect. On
systemic examination, all the vital functions were normal. On local examination,
multiple papulo-pustular lesions were present over cheeks, chin, and forehead. Under
magnifying glass there were multiple blackhead and whitehead comedons. Write a
prescription for this patient incorporating all the elements.
Dr XY Jain
MBBS, MD (General Medicine)
Registration no. MPMC 223344
25, Park Street, Gwalior MP
Tel. no. 111222 Email: -------@-----.com
Date: 00.00.0000
Patient’s name: Ms X
Age & Sex: 22 year, Female
Address: .................................
RX
Cap Doxycycline 100 mg orally 12 hrly for 2 weeks
Benzoyl Peroxide 2.5% Cream apply locally 12 hrly for 2 weeks
Take Doxycycline on empty stomach. Avoid dairy products, antacids and iron
for two hr before and after Doxycycline
Avoid exposure to sunlight Review after 2 weeks
Refills: 2
XY Jain
Stamp
Alternative Agents
Cap Tetracycline 500 mg orally 12 hrly for 6 weeks OR
Tab Erythromycin 250 mg orally 6 hrly for 6 weeks OR
Tab Minocycline 50 mg orally 12 hrly for 6 weeks OR
Tab Trimethoprim/Sulfamethoxazole 160/800 mg orally 12 hrly for 6 weeks
t.me/Dr_Mouayyad_AlbtousH
Case Scenarios and Prescription Writing 11
CASE SCENARIO 3
A 58-year-old male was referred to a tertiary hospital with complaints of fever, chest
pain, anorexia, weight loss, night sweats and productive cough with purulent sputum
for 1 month. On examination, his temperature was 101.7°F and blood pressure was
150/80 mm Hg. Chest X-ray and CT scan showed necrosis and consolidation of the
left lower lobe with pleural involvement. Bronchoalveolar lavage showed non-acid
fast, gram-positive, branching, filamentous bacteria, identified as actinomycetes in
culture on blood agar. Patient was admitted in medicine ward. Write a prescription
for this patient incorporating all the elements.
Dr XY Jain
MBBS, MD (General Medicine)
Registration no. MPMC 223344
Family Wellness Clinic, Fort View, Gwalior, MP
Tel. no. 111222 Email: ---------@-------.com
Date: 00.00.0000
Patient’s name: Mr X
Age & Sex: 58 year, Male
Address: .....................
Diagnosis: Actinomycosis
RX
Inj Penicillin G 3 Million Units IV 4 hrly for 6 weeks
Followed by
Tab Penicillin V 500 mg orally 6 hrly for 6 weeks to be taken 1 hr before or 2 hr
after meals
Tab Paracetamol 500 mg orally 6–8 hrly as required for fever
Give first injection after sensitivity test (AST)
Review after 6 weeks
Refills: 3 for Tab Penicillin V
XY Jain
Stamp
t.me/Dr_Mouayyad_AlbtousH
12 Manual of Prescription Writing
CASE SCENARIO 4
A 55-year-old man presents to a clinic with complaints of bloody, mucoid diarrhea for
4 days. On examination, his pulse is 90/minute and BP is 140/62 mm Hg. He has
generalized abdominal tenderness. Entamoeba histolytica trophozoites were seen in stool
samples. Write a prescription for this patient incorporating all the elements.
Dr XY Jain
MBBS, MD (General Medicine)
Registration no. MPMC 223344
Family Wellness Clinic, Fort View, Gwalior, MP
Tel. no. 111222 Email: ---------@-------.com
Date: 00.00.0000
Patient’s name: Mr X
Age & Sex: 55 year, Male
Address: ..................
RX
Tab Metronidazole 400 mg Orally 8 hrly for 10 days
Followed by
Tab Diloxanide Furoate 500 mg orally 8 hrly for 10 days
XY Jain
Stamp
t.me/Dr_Mouayyad_AlbtousH
Case Scenarios and Prescription Writing 13
CASE SCENARIO 5
A 28-year-old female presented to an ophthalmologist with complaints of redness,
purulent discharge, irritation, burning, sticking of the eyelids upon waking and light
sensitivity in both the eyes. The symptoms started 1 week ago; the right eye was affected
first followed by left eye, 2 days later. On examination, bulbar conjunctiva was
congested and mucopurulent discharge was present. Write a prescription for this
patient incorporating all the elements.
Dr XY Jain
MBBS, MD (General Medicine)
Registration no. MPMC 223344
Family Wellness Clinic, Fort View, Gwalior, MP
Tel. no. 111222 Email: ---------@-------.com
Date: 00.00.0000
Patient’s name: Ms X
Age & Sex: 28 year, Female
Address: .....................
RX
Moxifloxacin eye drops 0.5% 2 drops apply locally 6 hrly for 5 days
XY Jain
Stamp
Alternative Agents
Ofloxacin eye drops 0.3% 2 drops apply locally 4 hrly for 5 days OR
Ciprofloxacin eye drops 0.3% 2 drops apply locally 4 hrly for 5 days OR
Chloramphenicol eye drops 0.5% 2 drops apply locally 4 hrly for 5 days OR
Gentamicin eye drops 0.3% 2 drops apply locally 4 hrly for 5 days
t.me/Dr_Mouayyad_AlbtousH
14 Manual of Prescription Writing
CASE SCENARIO 6
A 26-year-old male presented with difficulty in breathing, wheezing, and cough. On
examination, his pulse was 100/min, BP was 120/70 mm Hg and respiratory rate was
20/min. Chest auscultation revealed reduced breath sounds with prolonged expiration.
On spirometry, FEV1 was 70% of predicted. Repeat spirometry after giving Salbutamol
recorded FEV1 as 90% of predicted. Write a prescription for this patient incorporating
all the elements.
Dr XY Jain
MBBS, MD (General Medicine)
Registration no. MPMC 223344
Family Wellness Clinic, Fort View, Gwalior, MP
Tel. no. 111222 Email: ---------@-------.com
Date: 00.00.0000
Patient’s name: Mr X
Age & Sex: 26 year, Male
Address: .....................
RX
Salbutamol 100 mcg/puff by Metered Dose Inhaler. Two puffs to be inhaled stat
and then 1 puff 4 hrly, if required
Tab Bromhexine 8 mg orally 8 hrly
Review after 24 hr
XY Jain
Stamp
Alternative Agents
Terbutaline 250 mcg/puff by metered dose inhaler. Two puffs to be inhaled stat and
then 1 puff 4 hrly if required.
t.me/Dr_Mouayyad_AlbtousH
Case Scenarios and Prescription Writing 15
CASE SCENARIO 7
A 34-year-old woman presents to her primary care physician with nasal congestion,
sore throat, productive cough and fever for the last 4 days. Initially she was not short
of breath, but now she becomes short of breath with exertion. She denies any
hemoptysis. Auscultation of chest revealed diminished breath sounds. On physical
examination, her temperature is 101.2°F with normal vital signs. On spirometry, FEV1
was 80% of predicted. Repeat spirometry after giving salbutamol recorded FEV1 as
85% of predicted. X-ray chest is normal. Write a prescription for this patient
incorporating all the elements.
Dr XY Jain
MBBS, MD (General Medicine)
Registration no. MPMC 223344
Family Wellness Clinic, Fort View, Gwalior, MP
Tel. no. 111222 Email: ---------@-------.com
Date: 00.00.0000
Patient’s name: Ms X
Age & Sex: 34 year, Female
Address: .....................
XY Jain
Stamp
Alternative Agents
Tab Azithromycin 500 mg orally once daily for 3 days
t.me/Dr_Mouayyad_AlbtousH
16 Manual of Prescription Writing
CASE SCENARIO 8
A 46-year-old male presented with rapid onset of redness, pain and swelling in his
right knee. Physical examination showed an obese man in acute distress with the
following vital signs: Blood pressure 130/78 mm Hg, pulse 106 bpm, respirations
22/min, temperature 97.8°F. The patient's right knee was exquisitely tender and
erythematous, warm to touch, and swollen with overlying skin tense, warm, and dusky
red. The rest of the physical examination was normal. Blood test revealed raised uric
acid. X-ray of the right knee showed soft tissue swelling around the joint but no erosion.
Synovial fluid analysis showed needle and rod shaped strongly negative birefringent
urate crystals both free in the fluid and engulfed by white blood cells. Write a
prescription for this patient incorporating all the elements.
Dr XY Jain
MBBS, MD (General Medicine)
Registration no. MPMC 223344
Family Wellness Clinic, Fort View, Gwalior, MP
Tel. no. 111222 Email: ---------@-------.com
Date: 00.00.0000
Patient’s name: Mr X
Age & Sex: 46 year, Male Address: .....................
Diagnosis: Acute Gout
RX
Tab Ibuprofen 800 mg orally 8 hrly for 3 days then 400 mg 8 hrly for 4 days or until
the pain is resolved
Review after 7 days
XY Jain
Stamp
t.me/Dr_Mouayyad_AlbtousH
Case Scenarios and Prescription Writing 17
CASE SCENARIO 9
A 60-year-old male was brought to emergency department with nausea, sweating and
severe chest pain for 2 hr. The chest pain radiated to his left arm and jaw. His vital
signs were blood pressure 124/80 mm Hg, pulse 110 bpm and regular, respirations
28/minute. ECG showed ST elevation in anterior leads. Patient was admitted in ICU.
Write a prescription for this patient incorporating all the elements.
Dr XY Jain
MBBS, MD (General Medicine)
Registration no. MPMC 223344
25, Park Street, Gwalior MP
Tel. no. 111222 Email: -------@-----.com
Date: 00.00.0000
Patient’s name: Mr X
Age & Sex: 60 year, Male
Address: .................................
RX
Complete bed rest for two days humidified O2 inhalation
Tab Aspirin 150 mg orally once
Tab Clopidogrel 75 mg orally once
Tab Nitroglycerine 0.5 mg SL can repeat every 5 min for 3 doses
Tab Isosorbide Dinitrate 5 mg orally 4 hrly
Tab Metoprolol 50 mg orally 12 hrly
Inj Tenecteplase 30 mg IV bolus over 5 sec max dose 50 mg single dose
Inj Diazepam 10 mg IV once slowly
Inj Enoxaparin 40 mg SC once
Diet: Liquid diet on 1st day, salt restriction, semi-solid diet from 2nd day onwards.
Inj Pethidine 50 mg IV if pain not relieved by Nitroglycerine
To be reviewed after 24 hr
XY Jain
Stamp
t.me/Dr_Mouayyad_AlbtousH
18 Manual of Prescription Writing
CASE SCENARIO 10
A 45-year-old immunocompromised male injures his right thigh after falling to the
ground. He sustained a piercing wound in thigh. He was given dressing for wound.
Patient took analgesics for intense pain which subsides after taking drug but the pain
returns off and on. The patient develops fever with chills after 5 days. He consults a
physician next day. Physician notices that the lower right thigh is hot, swollen and
tender. The right knee joint appears normal and has a full range of motion. The patient
has a temperature 101.5°F. X-rays of the right femur indicates no obvious abnormalities
of the bone. In blood test ESR and C-reactive protein are elevated. Blood cultures are
positive for Staphylococcus aureus. Patient was admitted in orthopedic ward. Write a
prescription for this patient incorporating all the elements.
Dr XY Jain
MBBS, MD (General Medicine)
Registration no. MPMC 223344
25, Park Street, Gwalior MP
Tel. no. 111222 Email: -------@-----.com
Date: 00.00.0000
Patient’s name: Mr X
Age & Sex: 45 year, Male
Address: .................................
RX
Inj Ceftriaxone 2 g IV once daily for 7 days
Followed by
Tab Cloxacillin 500 mg orally 8 hrly for 4 weeks
XY Jain
Stamp
t.me/Dr_Mouayyad_AlbtousH
Case Scenarios and Prescription Writing 19
CASE SCENARIO 11
A 22-year-old female presented to her family physician with a 3-day history of
rhinorrhea and a 2-day history of pain, fever and decreased hearing in right ear
associated with fever. Her vital signs were normal except for an oral temperature of
101.5°F. On otoscopic examination of right ear the tympanic membrane was
erythematous and bulging with decreased mobility. Write a prescription for this patient
incorporating all the elements.
Dr XY Jain
MBBS, MD (General Medicine)
Registration no. MPMC 223344
25, Park Street, Gwalior MP
Tel. no. 111222 Email: -------@-----.com
Date: 00.00.0000
Patient’s name: Ms X
Age & Sex: 22 year, Female
Address: .................................
RX
Cap Amoxicillin 500 mg orally 8 hrly for 7 days
Tab Paracetamol 500 mg orally 6–8 hrly as required for fever
XY Jain
Stamp
Alternative Agents
Tab Azithromycin 500 mg orally once on day 1, followed by 250 mg once daily for
4 days OR
Tab Cephalexin 250 mg orally 8 hrly for 7 days OR
Tab Cefuroxime 250 mg orally 12 hrly for 7 days OR
Tab Trimethoprim/Sulfamethoxazole 160/800 mg orally 12 hrly for 5–7 days
t.me/Dr_Mouayyad_AlbtousH
20 Manual of Prescription Writing
CASE SCENARIO 12
A 42-year-old female presented with high fever, rigors, nausea, vomiting, dysuria,
urinary frequency, urinary urgency and flank pain for 5 days. Physical examination
revealed a temperature of 102° F, respiratory rate of 18 breaths per minute, heart rate
of 108 beats per minute, blood pressure of 110/64 mm Hg. Her heart, lungs, and
abdomen were normal, but there was mild costovertebral angle tenderness over the
area of the left kidney. Urinalysis revealed large occult blood, proteinuria and 10 WBC/
HPF (normal 0–3). A non-contrast computed tomography (CT) scan of the abdomen
revealed severe left hydroureter and hydronephrosis without evidence of stones. Urine
was sent for culture and sensitivity. Patient was admitted in medicine ward. Write a
prescription for this patient incorporating all the elements.
Dr XY Jain
MBBS, MD (General Medicine)
Registration no. MPMC 223344
25, Park Street, Gwalior MP
Tel. no. 111222 Email: -------@-----.com
Date: 00.00.0000
Patient’s name: Ms X
Age & Sex: 42 year, Female
Address: .................................
RX
Inj Ceftriaxone 1 g IV once daily for 3 days
Inj Gentamicin 240 mg IV once daily for 3 days
Followed by
Tab Ciprofloxacin 500 mg orally 12 hrly for 7 days
Tab Paracetemol 500 mg orally 6–8 hrly as required for fever
XY Jain
Stamp
Alternative Agents
Inj Piperacillin/Tazobactam 4.5 g IV 6 hrly for 10 days OR
Inj Cefoperazone/Sulbactam 3 g IV 12 hrly for 10 days OR
Inj Ertapenem 1 g IV once a day for 10 days
t.me/Dr_Mouayyad_AlbtousH
Case Scenarios and Prescription Writing 21
CASE SCENARIO 13
A 30-year-old male presented with a history of alcohol intake over the last two year,
with no periods of abstinence. The patient stated that he started drinking alcohol when
he was 22-year-old along with his friends at college. Initially, he was taking alcohol
two pegs, occasionally. Two year back his alcohol intake was increased due to his job
pressure. He started having decreased sleep and appetite, irritability, tremors and
poor concentration. He wanted to quit alcohol and was admitted in medicine ward.
Write a prescription for this patient incorporating all the elements.
Dr XY Jain
MBBS, MD (General Medicine)
Registration no. MPMC 223344
25, Park Street, Gwalior MP
Tel. no. 111222 Email: -------@-----.com
Date: 00.00.0000
Patient’s name: Mr X
Age & Sex: 30 year, Male
Address: .................................
RX
Tab Disulfiram 500 mg orally once daily for 7 days followed by 250 mg orally
once daily for 7 days
Tab Ondansetron 8 mg orally as required for vomiting
Review after 14 days
XY Jain
Stamp
Alternative Agents
Tab Naltrexone 50 mg orally once daily for 14 days
Tab Acamprosate 333 mg two Tabs 8 hrly for 14 days
t.me/Dr_Mouayyad_AlbtousH
22 Manual of Prescription Writing
CASE SCENARIO 14
A 23-year-old female presented with sneezing, rhinorrhea, obstruction of the nasal
passages, lacrimation and conjunctival, nasal and pharyngeal itching. Her symptoms
started three days previously. On questioning, she told that her symptoms were worst
in the late summer and in the early spring season. Examination revealed transverse
nasal creases. The mucosa of the inferior turbinates was pale and oedematous with
clear nasal secretion. Her eyes were both pink with conjunctival congestion. Write a
prescription for this patient incorporating all the elements.
Dr XY Jain
MBBS, MD (General Medicine)
Registration no. MPMC 223344
25, Park Street, Gwalior MP
Tel. no. 111222 Email: -------@-----.com
Date: 00.00.0000
Patient’s name: Ms X
Age & Sex: 23 year, Female
Address: .................................
RX
Beclomethasone nasal spray 42 mcg apply locally per nostril 12 hrly for 5 days
Tab Loratadine 10 mg orally once daily for 5 days
XY Jain
Stamp
t.me/Dr_Mouayyad_AlbtousH
Case Scenarios and Prescription Writing 23
CASE SCENARIO 15
A 72-year-old male is referred to a neurologist by his family physician as he has memory
impairment for one year. He tells that his memory has worsened gradually over the
last one year. The patient reports that he sometimes has difficulty finding the words
he needs to express himself. He also has difficulty recalling the names of people, places
or objects. Brain imaging with MRI is suggestive of neurodegenerative disease. Write
a prescription for this patient incorporating all the elements.
Dr XY Jain
MBBS, MD (General Medicine)
Registration no. MPMC 223344
25, Park Street, Gwalior MP
Tel. no. 111222 Email: -------@-----.com
Date: 00.00.0000
Patient’s name: Mr X
Age & Sex: 72 year, Male
Address: .................................
RX
Tab Donepezil 5 mg orally once daily for 4 weeks, then 10 mg daily for next
4 weeks
Review after 8 weeks
XY Jain
Stamp
Alternative Agents
Tab Galantamine 4 mg orally 12 hrly for 4 weeks, then 8 mg daily for next 4 weeks OR
Tab Memantine 5 mg orally once daily for 4 weeks, then 10 mg daily for next 4 weeks
t.me/Dr_Mouayyad_AlbtousH
24 Manual of Prescription Writing
CASE SCENARIO 16
A 15-year-old boy presents to his family physician with a sore throat and fever. He is
diagnosed as a case of pharyngitis caused by group A -hemolytic Streptococcus. He
is given an IM injection of penicillin. Approximately 5 minutes later, he is found to be
in respiratory distress and has audible wheezing. His heart rate is 110 per minute and
his blood pressure is 70/30 mm Hg. Patient was admitted in medicine ward. Write a
prescription for this patient incorporating all the elements.
Dr XY Jain
MBBS, MD (General Medicine)
Registration no. MPMC 223344
25, Park Street, Gwalior MP
Tel. no. 111222 Email: -------@-----.com
Date: 00.00.0000
Patient’s name: Mr X
Age & Sex: 15 year, Male
Address: .................................
RX
Inj Adrenaline 1:1000 (1 mg/mL) 0.5 mL IM with repeated doses at 5–20 min
intervals as needed for a severe reaction.
Inj Chlorpheniramine 10 mg IV stat
Inj Hydrocortisone 200 mg IV stat
XY Jain
Stamp
t.me/Dr_Mouayyad_AlbtousH
Case Scenarios and Prescription Writing 25
CASE SCENARIO 17
A 60-year-old male came to medical OPD with complaints of heaviness in chest that
he experienced earlier in the day while climbing upstairs which was relieved after
taking rest. He stated that such episodes occurred previously also on exertion
or sometimes after taking large meals which were relieved after rest. His BP was
150/84 mm of Hg and pulse rate was 88/minute. ECG revealed mild ST depression.
Patient was admitted in medicine ward. Write a prescription for this patient
incorporating all the elements.
Dr XY Jain
MBBS, MD (General Medicine)
Registration no. MPMC 223344
25, Park Street, Gwalior MP
Tel. no. 111222 Email: -------@-----.com
Date: 00.00.0000
Patient’s name: Mr X
Age & Sex: 60 year, Male
Address: .................................
RX
Tab Nitroglycerine 0.5 mg SL stat, if no relief then repeat same dose after
10 min till pain is relieved or for 3 doses
Tab Isosorbide Dinitrate 10 mg orally 8 hrly
Tab Aspirin 150 mg orally once daily
Tab Metoprolol 50 mg orally 12 hrly
Review after 24 hr
XY Jain
Stamp
t.me/Dr_Mouayyad_AlbtousH
26 Manual of Prescription Writing
CASE SCENARIO 18
A 60-year-old man, living in rural area, presented with fever, malaise, headache,
dyspnoea, cough, loss of appetite and myalgia for the last 15 days. The patient stated
that he was treated with outpatient antibiotics a week ago but there was no
improvement despite treatment. On physical examination, his temperature was 102.5°F,
blood pressure was 150/80 mm Hg, pulse was 110/minute, and the respiratory rate
18/minute. Radiographic images showed mediastinal widening, bilateral pulmonary
infiltrates and pleural effusion. Blood and sputum culture showed growth of Bacillus
anthracis and the results were confirmed by polymerase chain reaction assay (PCR).
Patient was admitted in medicine ward. Write a prescription for this patient
incorporating all the elements.
Dr XY Jain
MBBS, MD (General Medicine)
Registration no. MPMC 223344
25, Park Street, Gwalior MP
Tel. no. 111222 Email: -------@-----.com
Date: 00.00.0000
Patient’s name: Mr X
Age & Sex: 60 year, Male
Address: .................................
Diagnosis: Anthrax
RX
Inj Ciprofloxacin 400 mg IV 12 hrly till required and then switch to Tab Ciprofloxacin
500 mg orally 12 hrly until 60 days
Tab Paracetamol 500 mg orally 6–8 hrly as required for fever
XY Jain
Stamp
Alternative Agents
Inj Doxycycline 100 mg IV 12 hrly for 60 days (IV and oral combined) OR
Inj Penicillin G 12–20 Million Units AST IM once a day till required then switch to oral
therapy when clinically stable with
Tab Ciprofloxacin 500 mg orally 12 hrly until 60 days
t.me/Dr_Mouayyad_AlbtousH
Case Scenarios and Prescription Writing 27
CASE SCENARIO 19
A 10-year-old boy weighing 30 kg presented with colicky abdominal pain for 5 days.
Abdominal USG showed multiple, long, linear, echogenic tubular structures in the
ileal segment that were consistent with A. lumbricoides infestation. Stool testing showed
a large number of A. lumbricoides eggs. Write a prescription for this patient incorporating
all the elements.
Dr XY Jain
MBBS, MD (General Medicine)
Registration no. MPMC 223344
25, Park Street, Gwalior MP
Tel. no. 111222 Email: -------@-----.com
Date: 00.00.0000
Patient’s name: Mr X
Age & Sex: 10 year, Male
Weight: 30 kg
Address: .................................
Diagnosis: Ascariasis
RX
Tab Albendazole 400 mg orally single dose
XY Jain
Stamp
Alternative Agents
Tab Mebendazole 100 mg orally 12 hrly for 3 days OR
Tab Pyrantel Pamoate 250 mg orally single dose OR
Tab Ivermectin 6 mg orally single dose
t.me/Dr_Mouayyad_AlbtousH
28 Manual of Prescription Writing
CASE SCENARIO 20
A 46-year-old male presented with sudden onset of breathlessness after a 5-day history
of productive cough, chest discomfort and fever. On examination, he was febrile and
tachypnoeic, with oxygen saturation of 84% on high-flow oxygen. Respiratory system
examination showed bronchial breath sounds in the right supraclavicular and
interscapular areas. All other systems were within normal limits. X-ray chest showed
a round infiltration in the upper part of the right lung. Direct microscopic examination
of the sputum showed septate hyphae compatible with a filamentous fungus. A high-
resolution CT scan of the chest showed a localized ground-glass appearance around a
consolidation (halo sign) suggestive of invasive aspergillosis. Fine needle lung biopsy
was performed and culture grew Aspergillus species. Write a prescription for this patient
incorporating all the elements.
Dr XY Jain
MBBS, MD (General Medicine)
Registration no. MPMC 223344
25, Park Street, Gwalior MP
Tel. no. 111222 Email: -------@-----.com
Date: 00.00.0000
Patient’s name: Mr X
Age & Sex: 46 year, Male
Address: .................................
Diagnosis: Aspergillosis
RX
Tab Voriconazole 200 mg orally 12 hrly for 6 weeks
XY Jain
Stamp
Alternative Agents
Tab Itraconazole 200 mg orally 12 hrly for 6 weeks OR
Tab Isavuconazole 200 mg orally once daily for 6 weeks OR
Tab Posaconazole 300 mg orally 12 hrly on day 1 and then 300 mg once daily for
6 weeks
t.me/Dr_Mouayyad_AlbtousH
Case Scenarios and Prescription Writing 29
CASE SCENARIO 21
A 35-year-old male presents with fever, headache, malaise, sore throat and cough. He
was well until 3 days earlier, when he suffered from mild sore throat and productive
cough with small amount of clear sputum. On examination, his temperature is 101°F,
pulse 102 beats/min, BP 110/70 mm Hg, respiratory rate 18 beats/min. On auscultation,
inspiratory crackles and bronchial breath sounds are present. The chest radiograph
shows pneumonic consolidation on the right upper lung field. Anti-Mycoplasma IgM
antibodies are present at a titer of 1:160 in serologic test and Mycoplasma pneumoniae is
detected in respiratory tract secretions in polymerase chain reaction (PCR) test. Write
a prescription for this patient incorporating all the elements.
Dr XY Jain
MBBS, MD (General Medicine)
Registration no. MPMC 223344
25, Park Street, Gwalior MP
Tel. no. 111222 Email: -------@-----.com
Date: 00.00.0000
Patient’s name: Mr X
Age & Sex: 35 year, Male
Address: .................................
RX
Tab Azithromycin 500 mg orally once on first day followed by 250 mg once daily
until 14 days
Tab Bromhexine 8 mg orally 8 hrly as required for cough or for 7 days
Tab Paracetamol 500 mg 6–8 hrly as required for fever
XY Jain
Stamp
Alternative Agents
Tab Clarithromycin 500 mg orally 12 hrly for 14 days OR
Cap Doxycycline 100 mg orally 12 hrly for 14 days OR
Tab Levofloxacin 750 mg orally once daily for 14 days OR
Tab Moxifloxacin 400 mg orally once daily for 14 days
t.me/Dr_Mouayyad_AlbtousH
30 Manual of Prescription Writing
CASE SCENARIO 22
An 80-year-old man presented with a 1-year history of increased frequency of
micturition, urgency, nocturia, poor stream, sensation of incomplete bladder emptying,
straining to urinate, and post-void dribbling. Digital rectal examination revealed an
enlarged, firm prostate without nodules or tenderness. The estimated total serum
prostate specific antigen (PSA) level was 3.1 ng/mL. The prostatic volume measured
by transrectal ultrasound was 55 mL. Write a prescription for this patient incorporating
all the elements.
Dr XY Jain
MBBS, MD (General Medicine)
Registration no. MPMC 223344
25, Park Street, Gwalior MP
Tel. no. 111222 Email: -------@-----.com
Date: 00.00.0000
Patient’s name: Mr X
Age & Sex: 80 year, Male
Address: .................................
RX
Tab Silodosin 4 mg orally once daily for 1 month
Tab Dutasteride 0.5 mg orally once daily for one month
Review after 1 month
XY Jain
Stamp
t.me/Dr_Mouayyad_AlbtousH
Case Scenarios and Prescription Writing 31
CASE SCENARIO 23
A 24-year-old male presented to a psychiatrist with a history of marked mood swings
between depression and mania. The cycle had begun two year previously following a
suicidal attempt. During his depressive periods, he was apathetic, retarded and
extremely drowsy. He felt that he was worthless, had no motivation, and lacked energy.
He felt more distressed in crowded places. When manic he had higher levels of self-
confidence, got more sociable with strangers, was more talkative, behaved aggressively,
showed a flight of ideas, elation, distractibility and constant restless movements. He
had no hallucinations or delusions, and his cognitive state was unimpaired. The swing
between the two moods was relatively fast and reliable, showing periodicity of about
four to five weeks. His blood tests, including a complete blood count and thyroid
function tests, were normal. Write a prescription for this patient incorporating all the
elements.
Dr XY Jain
MBBS, MD (General Medicine)
Registration no. MPMC 223344
25, Park Street, Gwalior MP
Tel. no. 111222 Email: -------@-----.com
Date: 00.00.0000
Patient’s name: Mr X
Age & Sex: 24 year, Male
Address: .................................
RX
Tab Lithium carbonate 200 mg orally 8 hrly for 15 days
Review after 15 days
XY Jain
Stamp
Alternative Agents
Tab Divalproex Sodium 500 mg orally 12 hrly for 30 days OR
Tab Carbamazepine 800 mg orally once daily for 30 days OR
Tab Lamotrigine 50 mg orally once daily for 30 days
t.me/Dr_Mouayyad_AlbtousH
32 Manual of Prescription Writing
CASE SCENARIO 24
A 23-year-old male presented with complaints of high-grade intermittent fever with
chills and rigors, malaise, fatigue, myalgias and loss of appetite of four weeks duration.
There was an evening rise of temperature. He was investigated many times and treated
with antimalarials and various antibiotics without any significant relief. On
examination, he had a temperature of 101°F and pulse rate of 108 beats/minute.
Haematology revealed haemoglobin of 11.4 gm%, total white blood cell count of
5200 cells/mm3 and ESR 28 mm at one hr. Ultrasound of the abdomen showed mild
splenomegaly. ELISA test for Brucella IgM antibodies was positive. Blood culture
yielded growth of Brucella melitensis and was confirmed by polymerase chain reaction
(PCR). Write a prescription for this patient incorporating all the elements.
Dr XY Jain
MBBS, MD (General Medicine)
Registration no. MPMC 223344
25, Park Street, Gwalior MP
Tel. no. 111222 Email: -------@-----.com
Date: 00.00.0000
Patient’s name: Mr X
Age & Sex: 23 year, Male
Address: .................................
Diagnosis: Brucellosis
RX
Inj Streptomycin 1 g IM once daily for 21 days
Cap Doxycycline 100 mg orally 12 hrly for 6 weeks
Tab Paracetamol 500 mg orally 6–8 hrly as required for fever
XY Jain
Stamp
t.me/Dr_Mouayyad_AlbtousH
Case Scenarios and Prescription Writing 33
CASE SCENARIO 25
A 35-year-old male presented with painful penile lesions that had progressed for
approximately two weeks prior to presentation. The patient reported unprotected sex
with multiple partners. The patient stated that the lesions began as small erythematous
macules that slowly progressed to form ulcers. The man’s vital signs were normal.
The physical examination revealed three painful, soft ulcers on penis, with a necrotic
base, surrounding erythema, and undermined edges associated with tender inguinal
lymphadenopathy. Culture of a swab of the lesion was positive for Haemophilus ducreyi.
Write a prescription for this patient incorporating all the elements.
Dr XY Jain
MBBS, MD (General Medicine)
Registration no. MPMC 223344
25, Park Street, Gwalior MP
Tel. no. 111222 Email: -------@-----.com
Date: 00.00.0000
Patient’s name: Mr X
Age & Sex: 35 year, Male
Address: .................................
Diagnosis: Chancroid
RX
Tab Azithromycin 1 g orally as single dose
XY Jain
Stamp
Alternative Agents
Inj Ceftriaxone 250 mg IM single dose OR
Tab Ciprofloxacin 500 mg orally 12 hrly for 3 days OR
Tab Erythromycin 500 mg orally 6 hrly for 7 days
t.me/Dr_Mouayyad_AlbtousH
34 Manual of Prescription Writing
CASE SCENARIO 26
A 32-year-old woman presents to a hospital with a history of sudden onset of severe
diarrhoea and vomiting for one day. She also complains of profound weakness. She
has just returned from a trip to a remote village where she worked for two days in a
camp. There she developed intestinal gurgling and vomiting followed by three loose
bowel movements. Soon she was having profuse watery diarrhoea occurring frequently.
The liquid stools are grey, turbid, and without faecal odour, blood, or pus. She looks
very weak but has no fever. Blood pressure is 100/60 mm Hg. V. cholerae are identified
in stool examination directly by darkfield microscopy on a wet mount of fresh stool.
Write a prescription for this patient incorporating all the elements.
Dr XY Jain
MBBS, MD (General Medicine)
Registration no. MPMC 223344
25, Park Street, Gwalior MP
Tel. no. 111222 Email: -------@-----.com
Date: 00.00.0000
Patient’s name: Ms X
Age & Sex: 32 year, Female
Address: .................................
Diagnosis: Cholera
RX
Cap Doxycycline 100 mg orally 12 hrly daily for 3 days
Tab Metoclopramide 10 mg orally 8 hrly as required
XY Jain
Stamp
Alternative Agents
Tab Azithromycin 1 g orally once daily for 3 days OR
Tab Ciprofloxacin 500 mg orally 12 hrly for 3 days
t.me/Dr_Mouayyad_AlbtousH
Case Scenarios and Prescription Writing 35
CASE SCENARIO 27
A 50-year-old man presented to the urology clinic complaining of urinary urgency,
dysuria, frequency and urethral discharge. He also complained of suprapubic pain
during voiding. He gave history of similar episodes frequently during the last one
year which were relieved by brief courses of antibiotics prescribed by his family
physician. On examination, his vital signs were normal. Per rectal examination showed
a painful, enlarged, symmetrical smooth prostate. Cystoscopic examination revealed
prostatic enlargement with congested prostatic mucosa suggestive of chronic prostatitis.
Culture of urine obtained after prostatic massage was positive for Proteus mirabilis.
Write a prescription for this patient incorporating all the elements.
Dr XY Jain
MBBS, MD (General Medicine)
Registration no. MPMC 223344
25, Park Street, Gwalior MP
Tel. no. 111222 Email: -------@-----.com
Date: 00.00.0000
Patient’s name: Mr X
Age & Sex: 50 year, Male
Address: .................................
RX
Tab Ciprofloxacin 500 mg orally 12 hrly for 28 Days
XY Jain
Stamp
Alternative Agents
Tab Levofloxacin 500 mg orally once daily for 28 days OR
Cap Doxycyclin 100 mg orally 12 hrly for 28 days OR
Tab Azithromycin 500 mg orally once daily for 28 days OR
Tab Trimethoprim/Sulfamethoxazole (160/800 mg) orally 12 hrly for 28 days
t.me/Dr_Mouayyad_AlbtousH
36 Manual of Prescription Writing
CASE SCENARIO 28
A 60-year-old male presents with progressive shortness of breath on exertion for the
past several days. His present problem began five days ago when he developed sore
throat, rhinorrhea and myalgia. Later he developed a productive cough with
mucopurulent sputum. He reports that he developed similar symptoms six months
ago and he was diagnosed with acute bronchitis by his family physician and was
treated. Patient had similar episodes frequently over the past six months. He had a
20 year history of smoking a pack of cigarettes a day however; he quit smoking shortly
prior to the onset of symptoms. On examination, his blood pressure is 134/82 mmHg,
heart rate is 96/minute, respiratory rate is 28/minute and temperature is 100.2°F.
Examination of the chest reveals bilaterally diminished breath sounds with coarse
crackles. On spirometry FEV1/FVC is less than normal. Write a prescription for this
patient incorporating all the elements.
Dr XY Jain
MBBS, MD (General Medicine)
Registration no. MPMC 223344
25, Park Street, Gwalior MP
Tel. no. 111222 Email: -------@-----.com
Date: 00.00.0000
Patient’s name: Mr X
Age & Sex: 60 year, Male
Address: .................................
RX
Tab Amoxicillin/Clavulanic acid 500/125 mg orally 8 hrly for 7 days
Tab Bromhexine 8 mg orally 8 hrly for 7 days
Tab Paracetamol 500 mg 6–8 hrly as required for fever
XY Jain
Stamp
Alternative Agents
Tab Cefuroxime 500 mg orally 12 hrly for 7 days
t.me/Dr_Mouayyad_AlbtousH
Case Scenarios and Prescription Writing 37
CASE SCENARIO 29
A 62-year-old male presented with gouty arthritis for 7 year. He was taking colchicine
and urate-lowering drug for the last 7 year but he was noncompliant with the therapy.
On physical examination, he had massive and deforming tophi and active synovitis of
bilateral small joints of the hands, wrists, elbows, knees and ankles. There were
ulcerated lesions over the tophi in both ankles. Laboratory data revealed uric acid
8.3 mg/dL and C-reactive protein 7.8 mg/dL. X-ray of hands showed erosion. Write a
prescription for this patient incorporating all the elements.
Dr XY Jain
MBBS, MD (General Medicine)
Registration no. MPMC 223344
25, Park Street, Gwalior MP
Tel. no. 111222 Email: -------@-----.com
Date: 00.00.0000
Patient’s name: Mr X
Age & Sex: 62 year, Male
Address: .................................
RX
Tab Allopurinol 100 mg orally once daily from 1st to 7th day then
200 mg orally once daily from 8th to 14th day
300 mg orally once daily for 15th to 21st day
Tab Paracetamol 500 mg orally 8 hrly as required for pain
Drink at least 10 glass of water daily.
XY Jain
Stamp
Alternative Agents
Tab Febuxostat 40 mg orally once daily for 21 days
t.me/Dr_Mouayyad_AlbtousH
38 Manual of Prescription Writing
CASE SCENARIO 30
A 25-year-old healthy female, married two months back, attends gynaecology OPD.
She does not want to become pregnant for 2 year. Her menstrual cycle is of 28 days
and menstruation lasts for 4–5 days. Her clinical examination including blood pressure
was normal. Write a prescription for this woman.
Dr XY Jain
MBBS, MD (General Medicine)
Registration no. MPMC 223344
25, Park Street, Gwalior MP
Tel. no. 111222 Email: -------@-----.com
Date: 00.00.0000
Patient’s name: Ms X
Age & Sex: 25 year, Female
Address: .................................
RX
Tab Norgestrel + Ethinyl Oestradiol 300 mcg/30 mcg once daily from 5th day of
menstrual period for 21 days.
Repeat same course after 7 days every month
Take one pill before going to bad daily uninterruptedly.
Report if more than two tablets are missed Take colored iron tablets for 7 days
after hormonal pills.
Refill every month for 23 months
XY Jain
Stamp
Alternative Agents
Tab Levonorgestrel 250 mcg + Tab Ethinyl Oestradiol 50 mcg orally once daily from
5th day of menstrual period for 21 days
t.me/Dr_Mouayyad_AlbtousH
Case Scenarios and Prescription Writing 39
CASE SCENARIO 31
A 57-year-old female patient, weighing 60 kg presented to the emergency department
of hospital, with a history of fever and headache for the last two weeks and neck
rigidity for the last 3 days. On examination, she had photophobia and a temperature
of 102.5°F. A brain computed tomography scan was carried out but did not reveal any
abnormal findings. Cerebrospinal fluid (CSF) examination under direct microscopy,
following India ink preparation, detected multiple encapsulated yeast cells, typical of
C. neoformans. CSF culture also showed growth of Cryptococcus neoformans. Write a
prescription for this patient incorporating all the elements.
Dr XY Jain
MBBS, MD (General Medicine)
Registration no. MPMC 223344
25, Park Street, Gwalior MP
Tel. no. 111222 Email: -------@-----.com
Date: 00.00.0000
Patient’s name: Ms X
Age & Sex: 57 year, Female
Weight: 60 kg
Address: .................................
RX
Inj Amphotericin B 50 mg in 500 mL of 5% dextrose, initially 10 mL IV slowly in
30 min to check sensitivity if tolerated then 3 mL/min once daily for 2 weeks
Cap Flucytocine 250 mg orally 6 hrly for 2 weeks
Tab Paracetamol 500 mg orally 6–8 hrly as required for fever.
Review after 2 weeks
XY Jain
Stamp
t.me/Dr_Mouayyad_AlbtousH
40 Manual of Prescription Writing
CASE SCENARIO 32
A 64-year-old male presents with complaints of left lower extremity pain that is worse
on walking. The patient has no chest pain or shortness of breath. He states that a left
total knee replacement was performed two weeks ago. Upon physical examination
the patient's left calf is erythematous and swollen with dusky appearance and pitting
oedema is present in the left lower extremity extending into the patient’s thigh.
Localized tenderness is also noted upon light palpation. The total knee replacement
incision is healing and well approximated without any drainage. A Doppler
ultrasonography of the left leg reveals left popliteal vein thrombosis extending into
the superficial femoral vein. Write a prescription for this patient incorporating all the
elements.
Dr XY Jain
MBBS, MD (General Medicine)
Registration no. MPMC 223344
25, Park Street, Gwalior MP
Tel. no. 111222 Email: -------@-----.com
Date: 00.00.0000
Patient’s name: Mr X
Age & Sex: 64 year, Male
Address: .................................
RX
Inj Enoxaparin 60 mg SC 12 hrly for 5 days
Followed by
Tab Dabigatran 150 mg orally 12 hrly for 4 weeks
Tab Paracetamol 500 mg orally 6–8 hrly as required for pain
Review after 5 days
XY Jain
Stamp
Alternative Agents
Inj Dalteparin 200 International Units/kg SC once daily for 5 days OR
Inj Tinzaparin 175 International Units/kg SC once daily for 5 days
Followed by
Tab Dabigatran 150 mg orally 12 hrly for 4 weeks OR
Tab Edoxaban 60 mg orally once daily for 4 weeks
t.me/Dr_Mouayyad_AlbtousH
Case Scenarios and Prescription Writing 41
CASE SCENARIO 33
A 35-year-old female presents to a physician with complaints of fatigue. She states
that she has daily crying spells, feels sad “all the time” and, not sleeping well and is
overeating. She reports that her sleeping pattern has also been altered and it frequently
takes her several hours to fall asleep. She reveals that she is constantly thinking and
worrying. She denies suicidal behavior and ideations. Her physical examination is
normal. Her blood tests, including a complete blood count and thyroid function tests,
are normal. Write a prescription for this patient incorporating all the elements.
Dr XY Jain
MBBS, MD (General Medicine)
Registration no. MPMC 223344
25, Park Street, Gwalior MP
Tel. no. 111222 Email: -------@-----.com
Date: 00.00.0000
Patient’s name: Ms X
Age & Sex: 35 year, Female
Address: .................................
Diagnosis: Depression
RX
Tab Fluoxetine 20 mg orally 12 hrly for 15 days
Review after 15 days
XY Jain
Stamp
Alternative Agents
Tab Venlafaxine 200 mg orally once daily for 15 days OR
Tab Escitalopram 10 mg orally once daily for 15 days OR
Tab Sertraline 25 mg orally once daily for 15 days
t.me/Dr_Mouayyad_AlbtousH
42 Manual of Prescription Writing
CASE SCENARIO 34
A 56-year-old male patient weighing 85 kg came to the OPD with complaints of
frequency of urination and easy fatigability. His fasting blood sugar was 124 mg/dL,
postprandial blood glucose 188 mg/dL and HbA1C 7.2%. His blood pressure was
130/80. His father was diabetic. His fasting blood glucose 6 months back was normal.
Write a prescription for this patient incorporating all the elements.
Dr XY Jain
MBBS, MD (General Medicine)
Registration no. MPMC 223344
25, Park Street, Gwalior MP
Tel. no. 111222 Email: -------@-----.com
Date: 00.00.0000
Patient’s name: Mr X
Age & Sex: 56 year, Male
Weight: 85 kg
Address: .................................
RX
Tab Metformin 500 mg orally 12 hrly with meals for 7 days
Reduce weight
Brisk walking for at least 30 min daily
Review after 7 days for dose escalation
XY Jain
Stamp
t.me/Dr_Mouayyad_AlbtousH
Case Scenarios and Prescription Writing 43
CASE SCENARIO 35
A 29-year-old male with type I diabetes since the age of 13 year was brought to
emergency department with a history of fever, cough, vomiting and drowsiness since
4 hr. On examination, his respiratory rate was 24/min and tongue was dry and coated.
His temperature was 101°F. His blood sugar was 500 mg/dL, arterial pH was 6.7.
Urine examination was positive for ketone bodies. Patient was admitted in medicine
ICU. Write a prescription for this patient incorporating all the elements.
Dr XY Jain
MBBS, MD (General Medicine)
Registration no. MPMC 223344
25, Park Street, Gwalior MP
Tel. no. 111222 Email: -------@-----.com
Date: 00.00.0000
Patient’s name: Mr X
Age & Sex: 29 year, Male
Address: .................................
RX
Inj Regular Insulin 5 Units IV stat Followed by 5 Units IV infusion/hr for 6 hr
Followed by 2 Units/hr till the patient becomes fully conscious then switch to SC
insulin
Normal saline 0.9% IV at the rate of 1 L/hr, followed by 5% Glucose in 0.45% saline
after the blood sugar has reached 300 mg/dL
Inj Pot Chloride 5 mL in 100 mL Normal Saline IV infusion slowly in 1 hr to start 4 hr
after initiation of Insulin infusion
Inj Sodium Bicarbonate (10%) 100 mL IV 2 hrly till pH rises above 7.2
Inj Sod/Pot Phosphate 5 mL in 25 mL of 0.9% saline in IV infusion in 15 min if
phosphate is low
Inj Ceftriaxone 1 g IV once
XY Jain
Stamp
t.me/Dr_Mouayyad_AlbtousH
44 Manual of Prescription Writing
CASE SCENARIO 36
A 24-year-old man presented to the emergency department with a five-day history of
sore throat, fever, malaise, difficulty in swallowing and hoarseness of voice. On physical
examination, temperature 100.4°F, heart rate 100 beats/minute, blood pressure 108/
64 mm Hg and respiratory rate 30/minute. Examination of throat showed a grey-
white membrane covering the tonsils, uvula, and soft palate. The patient was also
noted to have markedly enlarged cervical lymph nodes. Throat and nasopharyngeal
swabs culture were positive for Corynebacterium diphtheriae. Write a prescription for
this patient incorporating all the elements.
Dr XY Jain
MBBS, MD (General Medicine)
Registration no. MPMC 223344
25, Park Street, Gwalior MP
Tel. no. 111222 Email: -------@-----.com
Date: 00.00.0000
Patient’s name: Mr X
Age & Sex: 24 year, Male
Address: .................................
Diagnosis: Diphtheria
RX
Inj Procaine Penicillin G 1.2 Million Units IM once daily until the patient can swallow
comfortably then switch to
Tab Penicillin V 250 mg orally 6 hrly to complete a 14-day course
Tab Paracetamol 500 mg 6–8 hrly as required for fever
Give first injection after sensitivity test (AST)
XY Jain
Stamp
Alternative Agents
Inj Erythromycin 500 mg IV until the patient can swallow comfortably then,
Tab Erythromycin 500 mg orally 6 hrly to complete a 14-day course.
t.me/Dr_Mouayyad_AlbtousH
Case Scenarios and Prescription Writing 45
CASE SCENARIO 37
A 29-year-old married lady having one child of 2 year age presents in gynaecology
out patient clinic. She wants to avoid pregnancy as she has to appear in a competitive
exam after two months. She gave the history of unprotected intercourse two days
back. Her date of the last menstrual period was 20 days back. Her blood pressure and
other parameters are normal. Write a prescription for this woman.
Dr XY Jain
MBBS, MD (General Medicine)
Registration no. MPMC 223344
25, Park Street, Gwalior MP
Tel. no. 111222 Email: -------@-----.com
Date: 00.00.0000
Patient’s name: Ms X
Age & Sex: 29 year, Female
Address: .................................
RX
Tab Levonorgestrel 750 mcg orally stat followed by 750 mcg after 24 hr (total
two tablets)
XY Jain
Stamp
Alternative Agents
Tab Levonorgestrel 1.5 mg orally single dose OR
Tab Ethinylestradiol 100 mcg + Levonorgestrel 500 mcg orally stat followed by same
dose after 12 hr (total two tablets) OR
Tab Mifepristone 600 mg single dose followed two days later by
Tab Misoprostol 400 mcg orally as single dose
t.me/Dr_Mouayyad_AlbtousH
46 Manual of Prescription Writing
CASE SCENARIO 38
A 12-year-old boy presented of abdominal pain and perianal pruritus, particularly at
night for 2 weeks. His abdominal pain was localized to the left upper quadrant and
was sharp in nature. Bowel sounds were present on auscultation. An application of
clear cellulose acetate tape to the perianal region revealed adult worms and eggs which
were identified as Enterobius vermicularis or pinworm. Write a prescription for this
patient incorporating all the elements.
Dr XY Jain
MBBS, MD (General Medicine)
Registration no. MPMC 223344
25, Park Street, Gwalior MP
Tel. no. 111222 Email: -------@-----.com
Date: 00.00.0000
Patient’s name: Mr X
Age & Sex: 12 year, Male
Address: .................................
RX
Tab Mebendazole 100 mg orally once
Repeat one tab after 2 weeks
XY Jain
Stamp
t.me/Dr_Mouayyad_AlbtousH
Case Scenarios and Prescription Writing 47
CASE SCENARIO 39
A 70-year-old woman presented to the emergency department due to the rapid onset
of fever with chills, malaise and a suspected skin infection. On examination, she had
temperature of 101°F, respiratory rate of 15 breaths/min, heart rate of 86 beats/min,
and blood pressure of 124/72 mmHg. She had a sharply demarcated painful erythema
with well-defined indurated margins on her left upper arm. The lesion was warm to
the touch, tender, and appeared shiny and swollen. Blood culture was positive for
Streptococcus pyogenes. Write a prescription for this patient incorporating all the
elements.
Dr XY Jain
MBBS, MD (General Medicine)
Registration no. MPMC 223344
25, Park Street, Gwalior MP
Tel. no. 111222 Email: -------@-----.com
Date: 00.00.0000
Patient’s name: Ms X
Age & Sex: 70 year, Female
Address: .................................
Diagnosis: Erysipelas
RX
Tab Penicillin V 250 mg orally 6 hrly for 7 days
Tab Paracetamol 500 mg orally 6–8 hrly as required for fever
XY Jain
Stamp
Alternative Agents
Tab Cephlexin 250 mg orally 6 hrly for 7 days OR
Tab Clindamycin 250 mg 12 hrly for 7 days OR
Tab Erythromycin 250 mg 6 hrly for 7 days
t.me/Dr_Mouayyad_AlbtousH
48 Manual of Prescription Writing
CASE SCENARIO 40
A 62-year-old male patient presented with episodic fever, pain and swelling in the
scrotum and gross swelling of his right lower limb for 3 months. Physical examination
revealed massive oedema in the right lower limb with non-pitting oedema and skin
thickening. Hydrocele was also present. There were 2–3 palpable, non-tender lymph
nodes in the right inguinal region. The systemic examination was normal.
Haematological investigations were completely normal. A night blood smear
examination with Giemsa stain confirmed Bancroftian filariasis. Write a prescription
for this patient incorporating all the elements.
Dr XY Jain
MBBS, MD (General Medicine)
Registration no. MPMC 223344
25, Park Street, Gwalior MP
Tel. no. 111222 Email: -------@-----.com
Date: 00.00.0000
Patient’s name: Mr X
Age & Sex: 62 year, Male
Address: .................................
Diagnosis: Filariasis
RX
Tab Diethylcarbamazine 100mg 8 hrly for 21 days
XY Jain
Stamp
t.me/Dr_Mouayyad_AlbtousH
Case Scenarios and Prescription Writing 49
CASE SCENARIO 41
A 25-year-old male was brought to the emergency department by his brother. The
patient was playing a football game when suddenly the patient fell to the ground and
appeared to lose consciousness. His body stiffened with arms and legs extended. He
suddenly let out a shrill cry and appeared to stop breathing for about 10 seconds
followed by a series of violent, rhythmic, muscle contractions accompanied by
hyperventilation and rolling eyes. The jerking movements progressively became less
intense until they stopped altogether. The entire episode lasted approximately
3–4 minutes. The patient woke up confused with no recall of the attack but he
complained of a headache and extremely sore muscles. Write a prescription for this
patient incorporating all the elements.
Dr XY Jain
MBBS, MD (General Medicine)
Registration no. MPMC 223344
25, Park Street, Gwalior MP
Tel. no. 111222 Email: -------@-----.com
Date: 00.00.0000
Patient’s name: Mr X
Age & Sex: 25 year, Male
Address: .................................
RX
Tab Phenytoin 100 mg orally 12 hrly for 7 days
Tab Clobazam 10 mg orally at bed time for 7 days
Avoid driving vehicles and operating heavy machineries
Review after 7 days for dose escalation
XY Jain
Stamp
Alternative Agents
Tab SodiumValproate 300 mg orally 12 hrly for 7 days OR
Tab Carbamazepine 100 mg orally 12 hrly for 7 days
t.me/Dr_Mouayyad_AlbtousH
50 Manual of Prescription Writing
CASE SCENARIO 42
A 26-year-old male patient presented with severe dysuria and burning sensation during
urination with purulent discharge for 4 days. On physical examination, vital signs
showed: Blood pressure 110/80 mm Hg, pulse 78 bpm, and temperature 99.6°F. His
systemic examination was normal except for spontaneous discharge of pus from the
urethral meatus. Gram stain of the urethral discharge showed gram-negative diplococci.
Culture of urethral swab was positive for Neisseria gonorrhoeae. Write a prescription
for this patient incorporating all the elements.
Dr XY Jain
MBBS, MD (General Medicine)
Registration no. MPMC 223344
25, Park Street, Gwalior MP
Tel. no. 111222 Email: -------@-----.com
Date: 00.00.0000
Patient’s name: Mr X
Age & Sex: 26 year, Male
Address: .................................
Diagnosis: Gonorrhoea
RX
Inj Ceftriaxone 250 mg IM single dose
Tab Azithromycin 1g orally single dose
Tab Paracetamol 500 mg orally 6–8 hrly as required for fever
XY Jain
Stamp
Alternative Regimens
Tab Cefixime 400 mg orally single dose + Tab Azithromycin 1 g orally single dose OR
Inj Cefotaxime 500 mg IM single dose + Tab Azithromycin 1 g orally single dose OR
Cap Doxycycline 100 mg orally 12 hrly for 7 days
t.me/Dr_Mouayyad_AlbtousH
Case Scenarios and Prescription Writing 51
CASE SCENARIO 43
A 38-year-old man presented with a two-month history of a painless ulcerated lesion
on the penis. The patient reported painless nodule that slowly evolved to red ulcerated
lesion over 1 month. Physical examination revealed a 1 cm painless ulcer with rolled
edges. The base of the ulcer was irregular and beefy-red with friable base of granulation
tissue. Inguinal lymph nodes were not palpable. The rest of the physical examination
and routine blood and urine tests were normal. A serologic test for syphilis was
nonreactive. A biopsy was performed which showed many Donovan bodies. Write a
prescription for this patient incorporating all the elements.
Dr XY Jain
MBBS, MD (General Medicine)
Registration no. MPMC 223344
25, Park Street, Gwalior MP
Tel. no. 111222 Email: -------@-----.com
Date: 00.00.0000
Patient’s name: Mr X
Age & Sex: 38 year, Male
Address: .................................
RX
Tab Azithromycin 1 g orally on day 1 followed by 500 mg daily for 7 days
XY Jain
Stamp
Alternative Agents
Tab Trimethoprim/Sulfamethoxazole 160/800 mg orally 12 hrly for 14 days OR
Cap Doxycycline 100 mg orally 12 hrly for 14 days OR
Tab Erythromycin 500 mg orally 6 hrly for 14 days (in pregnant women) OR
Cap Tetracycline 500 mg orally 6 hrly for 14 days OR
Tab Ciprofloxacin 750 mg orally 12 hrly for 14 days
t.me/Dr_Mouayyad_AlbtousH
52 Manual of Prescription Writing
CASE SCENARIO 44
A 54-year-old woman presented to a district hospital with a 3-week history of
breathlessness, productive cough, weight loss and lethargy. There was no history of
chest pain, wheeze or hemoptysis. She had been recently treated with levofloxacin for
a lower respiratory tract infection. On examination, her temperature 100.6°F, with
heart rate 110/min and BP 108/64 mm Hg. Auscultation of chest revealed bilateral
basal crackles. X-ray chest revealed bilateral basal consolidation. Blood cultures showed
growth of gram-negative bacilli—Haemophilus influenzae. Write a prescription for this
patient incorporating all the elements.
Dr XY Jain
MBBS, MD (General Medicine)
Registration no. MPMC 223344
25, Park Street, Gwalior MP
Tel. no. 111222 Email: -------@-----.com
Date: 00.00.0000
Patient’s name: Ms X
Age & Sex: 54 year, Female
Address: .................................
RX
Cap Amoxicillin 750 mg 12 hrly for 10 days
Tab Paracetamol 500 mg orally 6–8 hrly as required for fever
XY Jain
Stamp
Alternatives Agents
Tab Azithromycin 500 mg orally once on day 1 followed by 250 mg daily for 4 days OR
Tab Cefuroxime Axetil 250 mg orally 12 hrly for 7 days OR
Tab Ciprofloxacin 500 mg orally 12 hrly for 7 days OR
Tab Levofloxacin 750 mg orally once daily for 7 days OR
Tab Moxifloxacin 400 mg orally once daily for 7 days OR
Tab Trimethoprim/Sulfamethoxazole (160/800 mg) orally 12 hrly for 7 days
t.me/Dr_Mouayyad_AlbtousH
Case Scenarios and Prescription Writing 53
CASE SCENARIO 45
A 27-year-old male reported to the department of dermatology with a complaint of
ulcers on upper lip associated with pain, itching and burning for 3 days. Medical history
revealed that he was suffering from fever and generalized weakness prior to the
ulcerations. On general physical examination, all the vital signs were within normal
limits. On extraoral examination, submandibular lymph nodes were palpable both on
the left and right sides which were mobile, tender and soft in consistency. Intraoral
examination revealed multiple vesicles and shallow ulcers measuring less than 0.5 cm
on lower labial mucosa surrounded by an erythematous area. Write a prescription for
this patient incorporating all the elements.
Dr XY Jain
MBBS, MD (General Medicine)
Registration no. MPMC 223344
25, Park Street, Gwalior MP
Tel. no. 111222 Email: -------@-----.com
Date: 00.00.0000
Patient’s name: Mr X
Age & Sex: 27 year, Male
Address: .................................
RX
Tab Acyclovir 200 mg orally 5 times daily for 5 days
Penciclovir cream 1% apply locally 2 hrly for 5 days
Tab Paracetamol 500 mg orally 6–8 hrly as required for pain
XY Jain
Stamp
Alternative Agents
Tab Famciclovir 250 mg orally 8 hrly for 7 days OR
Tab Valacyclovir 500 mg orally12 hrly for 7 days
t.me/Dr_Mouayyad_AlbtousH
54 Manual of Prescription Writing
CASE SCENARIO 46
A 45-year-old businessman, presents with loose, watery stools, abdominal cramping,
sweats, fevers and poor appetite for 2 months. He also lost 12 kg weight. He gave a
history of frequent business trips to West Africa during the last five year. A month
before consulting this clinic, he was treated in Africa for a soft chancre. Three weeks
later he developed flu-like symptoms associated with splenomegaly and
lymphadenopathy. Several antibiotic courses and malaria treatment had already been
prescribed in Western Africa. When seen in this clinic, he was in an unchanged clinical
condition and was severely ill. On examination, his temperature 102.4°F, pulse 94/min,
respiratory rate 18/min and blood pressure 122/80 mm Hg. Laboratory tests showed
anaemia and a CD4 count of 100 cells/mm3. Serology revealed a positive HIV-1 ELISA
and Western blot tests. Write a prescription for this patient incorporating all the
elements.
Dr XY Jain
MBBS, MD (General Medicine)
Registration no. MPMC 223344
25, Park Street, Gwalior MP
Tel. no. 111222 Email: -------@-----.com
Date: 00.00.0000
Patient’s name: Mr X
Age & Sex: 45 year, Male
Address: .................................
RX
Tab Tenofovir 300 mg orally once daily for 1 month
Tab Lamivudine 300 mg orally once daily for 1 month
Tab Dolutegravir 50 mg once daily for 1 month
Tab Paracetamol 500 mg orally 6–8 hrly as required for fever
Refill every month for 11 months
XY Jain
Stamp
Alternative Regimen
Tab Tenofovir 300 mg orally once daily for 1 month +
Tab Lamivudine 300 mg orally once daily for 1 month +
Tab Efavirenz 600 mg orally once daily for 1 month
t.me/Dr_Mouayyad_AlbtousH
Case Scenarios and Prescription Writing 55
CASE SCENARIO 47
A 58-year-old man with hypertension and type II diabetes is taking an angiotensin-
converting enzyme (ACE) inhibitor-thiazide diuretic combination for his hypertension
and metformin for his diabetes. His blood pressure and diabetes are under control
and renal function and liver function tests are normal. His lipid profile shows total
cholesterol 260 mg/dL, LDL 190 mg/dL, triglycerides 130 mg/dL and high density
lipoprotein 38 mg/dL. To reduce his risk of developing coronary artery disease, a
lipid-lowering drug is to be added to his present treatment regimen. Write a
prescription for this patient incorporating all the elements.
Dr XY Jain
MBBS, MD (General Medicine)
Registration no. MPMC 223344
25, Park Street, Gwalior MP
Tel. no. 111222 Email: -------@-----.com
Date: 00.00.0000
Patient’s name: Mr X
Age & Sex: 58 year, Male
Address: .................................
Diagnosis: Hypercholesterolemia
RX
Tab Atorvastatin 20 mg orally once daily for 1 month
Review after one month
XY Jain
Stamp
Alternative Agents
Tab Lovastatin 20–40 mg orally once daily for 1 month OR
Tab Pravastatin 40–80 mg orally once daily for 1 month OR
Tab Simvastatin 20–40 mg orally once daily for 1 month OR
Tab Fluvastatin 20–40 mg orally once daily for 1 month OR
Tab Rosuvastatin 5–20 mg orally once daily for 1 month OR
Tab Pitavastatin 1–2 mg orally once daily for 1 month
t.me/Dr_Mouayyad_AlbtousH
56 Manual of Prescription Writing
CASE SCENARIO 48
A 28-year-old female having 24 weeks of pregnancy attended antenatal clinic with
complaints of heaviness in head for 4 days. On examination, her BP was 150/90 mm
Hg with oedema over feet. Her Hb level was 12 g/dL. Per abdominal examination
was suggestive of 24 weeks of pregnancy. Write a prescription for this patient
incorporating all the elements.
Dr XY Jain
MBBS, MD (General Medicine)
Registration no. MPMC 223344
25, Park Street, Gwalior MP
Tel. no. 111222 Email: -------@-----.com
Date: 00.00.0000
Patient’s name: Ms X
Age & Sex: 28 year, Female
Address: .................................
RX
Tab Methyldopa 250 mg orally 8 hrly for 7 days
Review after 7 days
XY Jain
Stamp
Alternative Agents
Tab Labetalol 50 mg orally 12 hrly for 7 days OR
Tab Nifedipine 30 mg orally once daily for 7 days
t.me/Dr_Mouayyad_AlbtousH
Case Scenarios and Prescription Writing 57
CASE SCENARIO 49
A 32-year-old male patient of hypertension was taking tablet Metoprolol since
one year and was alright. He suffered from peptic ulcer and was treated for that
3 months back. He has not taken metoprolol for last three months. He was brought to
ICU with severe headache, swelling over limbs and reduced urine output. On
examination, his blood pressure 200/120 mm Hg, blood urea 44 mg/dL, serum
creatinine 1.9 mg/dL and random blood sugar 140 mg/dL. Patient was admitted in
medicine ICU. Write a prescription for this patient.
Dr XY Jain
MBBS, MD (General Medicine)
Registration no. MPMC 223344
25, Park Street, Gwalior MP
Tel. no. 111222 Email: -------@-----.com
Date: 00.00.0000
Patient’s name: Mr X
Age & Sex: 32 year, Male
Address: .................................
RX
Inj Labetalol 20 mg IV in 2 min then 40 mg every 10 min until BP is reduced by 25%
then switch over to oral drugs
(Max dose 300 mg in 24 hr)
XY Jain
Stamp
Alternative Agents
Inj Nicardipine 5 mg/hr, titrate by 2.5 mg/hr at 5–10 min interval (Max. 15 mg/hr) OR
Inj Nitroglycerine 5 mcg/min, titrate by 5 mcg/min at 5 min interval up to 20 mcg/
min OR
Inj Glyceryl trinitrate 10 mcg/minute IV OR
Inj Sodium Nitroprusside 0.25–10 mcg/kg/min
t.me/Dr_Mouayyad_AlbtousH
58 Manual of Prescription Writing
CASE SCENARIO 50
A 35-year-old male complained of nervousness, weakness, weight loss, heat intolerance
and occasional palpitations for the past 3 months. On examination, his pulse was
110/minute and BP was 136/70 mm of Hg. There were tremors of outstretched fingers
and lid lag was present. He had diffuse, nontender, smooth enlargement of the thyroid.
His thyroid-stimulating hormone level (TSH) was low and T4 level was elevated. Write
a prescription for this patient incorporating all the elements.
Dr XY Jain
MBBS, MD (General Medicine)
Registration no. MPMC 223344
25, Park Street, Gwalior MP
Tel. no. 111222 Email: -------@-----.com
Date: 00.00.0000
Patient’s name: Mr X
Age & Sex: 35 year, Male
Address: .................................
Diagnosis: Hyperthyroidism
RX
Tab Carbimazole 15 mg orally 8 hrly for 15 days
Refills: 5
Repeat TSH estimation after 3 months for dose adjustment
XY Jain
Stamp
Alternative Agents
If hyperthyroidism in pregnancy
Tab Propylthiouracil 100–150 mg orally 8 hrly till delivery
t.me/Dr_Mouayyad_AlbtousH
Case Scenarios and Prescription Writing 59
CASE SCENARIO 51
A 65-year-old, Type II diabetic patient was maintained on insulin for last ten year. His
blood sugar was well controlled and his treating physician was satisfied. One evening
this patient went to gym after taking insulin and escaped meals. He was brought to
the casualty department in semiconscious state. His blood sugar level was 30 mg/dL.
Patient was admitted in casualty ward. Write a prescription for this patient.
Dr XY Jain
MBBS, MD (General Medicine)
Registration no. MPMC 223344
25, Park Street, Gwalior MP
Tel. no. 111222 Email: -------@-----.com
Date: 00.00.0000
Patient’s name: Mr X
Age & Sex: 65 year, Male
Address: .................................
Diagnosis: Hypoglycemia
RX
Inj Dextrose (50%) 50 mL IV bolus in 3–5 min
XY Jain
Stamp
Alternative Agents
Inj Glucagon 1 mg SC or IM or IV stat
t.me/Dr_Mouayyad_AlbtousH
60 Manual of Prescription Writing
CASE SCENARIO 52
A 45-year-old female presents with complaints of progressive weight gain of
4 kilograms in 6 months, fatigue, slight memory loss, slow speech, deepening of her
voice, dry skin, constipation, and cold intolerance. On examination, her blood pressure
is 124/82 mm Hg and heart rate is 70/minute. Her thyroid gland feels diffusely
enlarged, nontender and firm in consistency. Her thyroid-stimulating hormone level
(TSH) is elevated, and T4 level is reduced. Patient was admitted in medicine ward.
Write a prescription for this patient incorporating all the elements.
Dr XY Jain
MBBS, MD (General Medicine)
Registration no. MPMC 223344
25, Park Street, Gwalior MP
Tel. no. 111222 Email: -------@-----.com
Date: 00.00.0000
Patient’s name: Ms X
Age & Sex: 45 year, Female
Address: .................................
Diagnosis: Hypothyroidism
RX
Tab Levothyroxine 50 mcg orally once daily 30 min before breakfast for 4 weeks.
Refils: 2
Repeat TSH after 12 weeks for dose adjustment
XY Jain
Stamp
t.me/Dr_Mouayyad_AlbtousH
Case Scenarios and Prescription Writing 61
CASE SCENARIO 53
A 62-year-old male with prior medical history of hypertension, presented to the
emergency department of a tertiary care hospital with symptoms of rigors, chills, fever,
weakness and lower extremity edema for two weeks. On examination, his temperature
was 100.4°F, pulse rate was 94/minute, respiratory rate was 24/minute, blood pressure
was 104/56 mm Hg and oxygen saturation on room air was 90%. Bilateral lower
extremity edema was present with erythema. ECG was normal. A transthoracic
echocardiogram was performed which revealed mitral valve vegetations, minimal
mitral valve regurgitation, and a normal ejection fraction of 55%. Patient was admitted
in medicine ward. Blood culture was positive for Enterococcus faecalis. Write a
prescription for this patient incorporating all the elements.
Dr XY Jain
MBBS, MD (General Medicine)
Registration no. MPMC 223344
25, Park Street, Gwalior MP
Tel. no. 111222 Email: -------@-----.com
Date: 00.00.0000
Patient’s name: Mr X
Age & Sex: 62 year, Male
Address: .................................
RX
Inj Penicillin G 4 Million Units IV 4 hrly for 2 weeks
Inj Gentamicin 60 mg IV 8 hrly for 2 weeks
Tab Paracetamol 500 mg orally 6–8 hrly as required for fever
Inj Penicillin to be started after sensitivity test
Review after 2 weeks
Refills: 2
XY Jain
Stamp
t.me/Dr_Mouayyad_AlbtousH
62 Manual of Prescription Writing
CASE SCENARIO 54
A 60-year-old man presents to his family physician with a complaint of persistent
insomnia for approximately 4 months. Initially, he only had difficulty staying asleep
but now he has frequent nocturnal awakenings. He wakes up two or more times per
night and on most nights has difficulty falling asleep. Over the past two months,
daytime sleepiness and fatigue have increased in severity associated with diminished
energy, poor concentration, memory impairment, irritability and depressed or anxious
mood. His physical examination is normal. Write a prescription for this patient
incorporating all the elements.
Dr XY Jain
MBBS, MD (General Medicine)
Registration no. MPMC 223344
25, Park Street, Gwalior MP
Tel. no. 111222 Email: -------@-----.com
Date: 00.00.0000
Patient’s name: Mr X
Age & Sex: 60 year, Male
Address: .................................
Diagnosis: Insomnia
RX
Tab Zopiclone 7.5 mg orally once at bed time for 7 days
Review after 7 days
XY Jain
Stamp
Alternative Agents
Tab Eszopiclone 3 mg orally once at bed time for 7 days OR
Tab Zolpidem 10 mg orally once at bed time for 7 days OR
Tab Nitrazepam 5 mg orally once at bed timefor 7 days
t.me/Dr_Mouayyad_AlbtousH
Case Scenarios and Prescription Writing 63
CASE SCENARIO 55
A 30-year-old woman presented to her family physician with the complaints of
generalized weakness, easy fatigability, palpitations, and dyspnoea on exertion. She
stated that these symptoms had started about 3 months ago and had become
progressively worse over the past 2 weeks. The patient has no history of menorrhagia
or overt evidence of bleeding from any site. Physical examination revealed a smooth
tongue, brittle nails, spooning of nails and pale conjunctiva. Her vitals were as follows:
Blood pressure 128/76 mm Hg, pulse 108 bpm, respirations 24/min, body temperature
97.7°F. Stools were negative for occult blood. Complete blood count showed Hg
7.1 gm/dL with a low ferritin level. Write a prescription for this patient incorporating
all the elements.
Dr XY Jain
MBBS, MD (General Medicine)
Registration no. MPMC 223344
25, Park Street, Gwalior MP
Tel. no. 111222 Email: -------@-----.com
Date: 00.00.0000
Patient’s name: Ms X
Age & Sex: 30 year, Female
Address: .................................
RX
Tab Ferrous Sulfate 200 mg orally 8 hrly after meals for 1 month
Oral iron makes the stool black. Review after one month
XY Jain
Stamp
Alternative Agents
Tab Ferrous Fumarate 200 mg orally 8 hrly for 1 month OR
Tab Ferrous Gluconate 300 mg orally 8 hrly for 1 month
t.me/Dr_Mouayyad_AlbtousH
64 Manual of Prescription Writing
CASE SCENARIO 56
A 63-year-old man weighing 70 kg was brought to ICU with breathlessness and swelling
over feet for 7 days. He gave history of stent placement in left coronary artery five
year back during angiography for unstable angina and was well controlled. On
examination, his BP was 160/100 mm Hg. On auscultation, basal crepitations were
present on both sides of lungs. Urine output was 800 mL in 24 hr. Blood urea was
30 mg/dL. His echocardiography revealed ejection fraction below normal. Patient was
admitted in medicine ICU. Write a prescription for this patient.
Dr XY Jain
MBBS, MD (General Medicine)
Registration no. MPMC 223344
25, Park Street, Gwalior MP
Tel. no. 111222 Email: -------@-----.com
Date: 00.00.0000
Patient’s name: Mr X
Age & Sex: 63 year, Male
Weight: 73 kg
Address: .................................
RX
Immediate hospitalization in ICU
Oxygen inhalation, if saturation is less than 90%
Inj Furosemide 40 mg IV stat
Inj Dopamine 50 mcg /min IV infusion
Tab Isosorbide Dinitrate 20 mg orally 12 hrly
Pot Chloride solution 5 mL orally with 100 mL of water 12 hrly.
Review after 24 hr
XY Jain
Stamp
t.me/Dr_Mouayyad_AlbtousH
Case Scenarios and Prescription Writing 65
CASE SCENARIO 57
A 45-year-old woman presented to an emergency department with a three-day history
of fever, chills, headache, nausea, vomiting, abdominal pain, conjunctival congestion,
arthralgia, myalgia, and loss of appetite. She gave history of a recent rat bite for which
she received tetanus toxoid and wound care at a nearby clinic. Her complete blood
count showed anemia, leukocytosis with neutrophilia and thrombocytopenia. Clinical
examination recorded a temperature of 104°F, blood pressure of 118/76 mm Hg, heart
rate of 104 beats/minute and respiratory rate of 16/minute. On abdominal palpation
splenomegaly was present. Leptospiral strip test for IgM antibodies was positive.
Patient was admitted in medicine ward. Write a prescription for this patient
incorporating all the elements.
Dr XY Jain
MBBS, MD (General Medicine)
Registration no. MPMC 223344
25, Park Street, Gwalior MP
Tel. no. 111222 Email: -------@-----.com
Date: 00.00.0000
Patient’s name: Ms X
Age & Sex: 45 year, Female
Address: .................................
Diagnosis: Leptospirosis
RX
Inj Penicillin G 1.2 Million Units IV 6 hrly till required then switch to Tab Penicillin V
500 mg four times daily until 10 days
Tab Paracetamol 500 mg orally 6–8 hrly as required for fever
Inj Penicillin to be started after sensitivity test
XY Jain
Stamp
t.me/Dr_Mouayyad_AlbtousH
66 Manual of Prescription Writing
CASE SCENARIO 58
A 48-year-old male presented with a 5-day history of fever, headache, nausea and
diarrhoea. He had up to 15 episodes of diarrhea in the last 24 hours. The patient revealed
that he frequently attended parties and enjoyed fruits, salads, and cheese. On physical
examination, his temperature 101.4°F, blood pressure 96/60 mm Hg, and pulse
120 per minute. CBC showed leukocytosis. CT of abdomen revealed enteritis and colitis
of the descending sigmoid colon. The patient’s blood culture was positive for Listeria
monocytogenes. Patient was admitted in medicine ward. Write a prescription for this
patient incorporating all the elements.
Dr XY Jain
MBBS, MD (General Medicine)
Registration no. MPMC 223344
25, Park Street, Gwalior MP
Tel. no. 111222 Email: -------@-----.com
Date: 00.00.0000
Patient’s name: Mr X
Age & Sex: 48 year, Male
Address: .................................
Diagnosis: Listeriosis
RX
Inj Ampicillin 2 g IV 6 hrly for 14 days
Inj Gentamicin 240 mg IV once daily for 14 days
Tab Paracetamol 500 mg orally 6–8 hrly as required for fever
XY Jain
Stamp
t.me/Dr_Mouayyad_AlbtousH
Case Scenarios and Prescription Writing 67
CASE SCENARIO 59
A 50-year-old male presented with a 5-day history of extreme headaches, fever with
chills, generalized joint pain and sudden appearance of a painful red papule on back
of his right thigh, which was increasing progressively. He gave a history of visit to the
nearby forest a few days before the appearance of the lesion. On examination, there
was an erythematous annular plaque on back of the right thigh approximately
5 centimeter in diameter with a central brownish fluid filled blister. Serology for Borrelia
burgdorferi antigen was done with enzyme-immunoassay technique. IgM was raised,
whereas IgG was found to be normal. Write a prescription for this patient incorporating
all the elements.
Dr XY Jain
MBBS, MD (General Medicine)
Registration no. MPMC 223344
25, Park Street, Gwalior MP
Tel. no. 111222 Email: -------@-----.com
Date: 00.00.0000
Patient’s name: Mr X
Age & Sex: 50 year, Male
Address: .................................
RX
Cap Doxycycline 100 mg orally 12 hrly on empty stomach for 14 days
Tab Paracetamol 500 mg orally 6–8 hrly as required for fever
Avoid iron, milk and dairy products with the drug
XY Jain
Stamp
Alternative Agents
Cap Amoxicillin 500 mg orally 8 hrly for 21 days OR
Tab Cefuroxime Axetil 500 mg orally 12 hrly for 21 days OR
Tab Erythromycin 250 mg orally 6 hrly for 21 days
t.me/Dr_Mouayyad_AlbtousH
68 Manual of Prescription Writing
CASE SCENARIO 60
A 40-year-old man presented at STD clinic with a painful penile ulcer, mild fever and
general malaise. On physical examination, the temperature was 100.4 °F with a marked
swelling in the right groin which was firm, fluctuant, and slightly tender. The penile
ulcer was 2.5 cm in diameter. The bottom of the ulcer was red and showed some
purulent exudates. Edges of the ulcer were elevated and round. PCR was done in the
biopsy of the penis lesion which was positive for Chlamydia trachomatis. Serology for
C. trachomatis showed relatively high levels of IgA and IgG. Write a prescription for
this patient incorporating all the elements.
Dr XY Jain
MBBS, MD (General Medicine)
Registration no. MPMC 223344
25, Park Street, Gwalior MP
Tel. no. 111222 Email: -------@-----.com
Date: 00.00.0000
Patient’s name: Mr X
Age & Sex: 40 year, Male
Address: .................................
RX
Cap Doxycycline 100 mg orally 12 hrly for 21 days
Tab Paracetamol 500 mg orally 6–8 hrly as required for fever
XY Jain
Stamp
Alternative Agents
Tab Erythromycin 500 mg orally 6 hrly for 21 days OR
Tab Azithromycin 1 g orally once weekly for 3 weeks
t.me/Dr_Mouayyad_AlbtousH
Case Scenarios and Prescription Writing 69
CASE SCENARIO 61
A 55-year-old male presents with fever with chills, rigors, vomiting and headache for
5 days. He has recently came from an area where chloroquine resistant falciparum
malaria is prevalent. Per abdominal examination reveals splenomegaly. Complete blood
count shows Hb 7.5 g/dL. Peripheral blood smear showed P. falciparum. Write a
prescription for this patient.
Dr XY Jain
MBBS, MD (General Medicine)
Registration no. MPMC 223344
25, Park Street, Gwalior MP
Tel. no. 111222 Email: -------@-----.com
Date: 00.00.0000
Patient’s name: Mr X
Age & Sex: 55 year, Male
Address: .................................
RX
Tab Artemether 80 mg + Lumefantrine 480 mg orally 12 hrly for 3 days
Tab Paracetamol 500 mg orally 6–8 hrly as required for fever
Tab Metoclopramide 10 mg orally 8 hrly before meals as required for vomiting
XY Jain
Stamp
Alternative Agents
Tab Artesunate 100 mg orally 12 hrly for 3 days +
Tab Mefloquine 750 mg orally stat followed by 500 mg after 12 hr OR
Tab Artesunate 100 mg orally 12 hrly for 3 days +
Tab Sulfadoxine + Pyrimethamine 1500 mg/75 mg orally single dose OR
Tab Quinine 600 mg orally 8 hrly for 7 days +
Cap Doxycycline 100 mg orally once for 7 days OR
Tab Clindamycin 600 mg orally 12 hrly for 7 days OR
Cap Tetracycline 250 mg orally 6 hrly for 7 days
t.me/Dr_Mouayyad_AlbtousH
70 Manual of Prescription Writing
CASE SCENARIO 62
A 35-year-old male presented to the hospital with a 5-day history of intermittent
fever with chills and rigors, headache, nausea and vomiting. On examination, his
temperature was 102°F and heart rate was 112/minute. Mild tenderness in the right
hypochondrium with hepatomegaly was also noted. Examination of peripheral blood
smears revealed rings and trophozoites typical of the P. vivax. The patient was treated
with oral chloroquine 10 mg/kg, 10 mg/kg and 5 mg/kg body wt on days 1, 2 and 3
respectively. But the patient remained febrile on completion of chloroquine therapy,
therefore peripheral blood smear examination was repeated which continued to show
P. vivax. Write a prescription for this patient incorporating all the elements.
Dr XY Jain
MBBS, MD (General Medicine)
Registration no. MPMC 223344
25, Park Street, Gwalior MP
Tel. no. 111222 Email: -------@-----.com
Date: 00.00.0000
Patient’s name: Mr X
Age & Sex: 35 year, Male
Address: .................................
RX
Tab Mefloquine 750 mg orally once and 500 mg after 12 hr
Tab Primaquine 15 mg orally once daily for 14 days.
Tab Paracetamol 500 mg orally 6–8 hrly as required for fever
Tab Metoclopramide 10 mg orally 8 hrly before meals as required for vomiting
XY Jain
Stamp
t.me/Dr_Mouayyad_AlbtousH
Case Scenarios and Prescription Writing 71
CASE SCENARIO 63
A 34-year-old female presented to the hospital with a 5-day history of intermittent
fever with chills, headache, nausea, vomiting, loss of appetite, generalized weakness
and body pains. On examination, her temperature was 102°F and heart rate was
112/minute. Mild tenderness in the right hypochondrium with hepatomegaly was
also noted. Examination of peripheral blood smears revealed rings and trophozoites
typical of the P. vivax. Write a prescription for this patient incorporating all the elements.
Dr XY Jain
MBBS, MD (General Medicine)
Registration no. MPMC 223344
25, Park Street, Gwalior MP
Tel. no. 111222 Email: -------@-----.com
Date: 00.00.0000
Patient’s name: Ms X
Age & Sex: 34 year, Female
Address: .................................
RX
Tab Chloroquine phosphate 600 mg base (1g salt) orally once
Followed by
Tab Chloroquine phosphate 300 mg base (500 mg salt) orally after meals at 6,
24 and 48 hours. Followed by:
Tab Primaquine 15 mg orally once daily for 14 days.
Tab Paracetamol 500 mg orally 6–8 hrly as required for fever
Tab Metoclopramide 10 mg orally 8 hrly before meals as required for vomiting
Get test done for G6PD status before starting Primaquine
XY Jain
Stamp
t.me/Dr_Mouayyad_AlbtousH
72 Manual of Prescription Writing
CASE SCENARIO 64
A 45-year-old male, weighing 60 kg, diagnosed with primary tuberculosis was on first
line anti-tubercular treatment. The patient did not show improvement in cough, fever
and breathlessness after one month of therapy. His sputum for AFB was positive. He
was advised CB NAAT test which showed rifampicin resistance. Write a prescription
for his patient incorporating all the elements.
Dr XY Jain
MBBS, MD (General Medicine)
Registration no. MPMC 223344
25, Park Street, Gwalior MP
Tel. no. 111222 Email: -------@-----.com
Date: 00.00.0000
Patient’s name: Mr X
Age & Sex: 45 year, Male
Weight: 60
Address: .................................
RX
Inj Kanamycin 1 g IM one daily for 30 days
Tab Moxifloxacin 800 mg orally once daily for 30 days
Tab Ethionamide 500 mg orally once daily for 30 days
Cap Clofazimine 100 mg orally once daily for 30 days
Tab Pyrazinamide 750 mg orally 12 hrly for 30 days
Tab Isoniazid 900 mg (high dose) orally once daily for 30 days
Tab Ethambutol 1200 mg orally once daily for 30 days
Tab Pyridoxine 50 mg orally once daily for 30 days
Review every month
Get eye examination done before starting Ethambutol
Refills: 3
XY Jain
Stamp
t.me/Dr_Mouayyad_AlbtousH
Case Scenarios and Prescription Writing 73
Note: For above shorter MDR TB regimen: Patient should come for follow up every
month. Sputum for AFB will be repeated after 4 months of initial phase treatment. If
sputum is positive extend the same treatment for 2 months with injectable on alternate
days then switch to continuation phase with following five drugs for 5 months. Total
duration of treatment is 9–11 months)
Tab Moxifloxacin 800 mg orally once daily
Cap Clofazimine100 mg orally once daily
Tab Ethambutol 1200 mg orally once daily
Tab Pyrazinamide 750 mg orally 12 hrly
Tab Pyridoxine 50 mg orally once daily
Alternative Regimen
All oral longer MDR TB regimen
Tab Bedaquiline 400 mg orally once daily for 2 weeks then 200 mg orally three times
per week for 3–24 weeks
Tab Linezolid 600 mg orally once daily for 18 months
Tab Clofazimine 100 mg orally once daily for 18 months
Tab Cycloserine 750 mg orally once daily for 18 months
t.me/Dr_Mouayyad_AlbtousH
74 Manual of Prescription Writing
CASE SCENARIO 65
A 55-year-old woman presented to her family physician with the complaints of
generalized weakness, easy fatigability, pain in the legs on walking and dyspnoea on
exertion. She stated that these symptoms had started about 6 months ago and had
become progressively worse over the past 2 months. She also noted recently increased
forgetfulness, some pain on her tongue, tingling and numbness of the fingers. The
patient has no history of menorrhagia or overt evidence of bleeding from any site. She
was a strict vegetarian. Her vitals were normal except for a heart rate of 120 beats/
minute. The patient’s conjunctiva, oral mucosa, and nail beds were pale, and her tongue
was red and painful. Neurological examination disclosed decreased cutaneous pain
and vibratory sensation in both the upper and lower extremities. Her hematological
investigations revealed haemoglobin 7.2 g/dL, mean corpuscular volume (MCV)
116 fL (normal 80–100), ferritin levels 300 ng/mL (normal 20–250). Write a prescription
for this patient incorporating all the elements.
Dr XY Jain
MBBS, MD (General Medicine)
Registration no. MPMC 223344
25, Park Street, Gwalior MP
Tel. no. 111222 Email: -------@-----.com
Date: 00.00.0000
Patient’s name: Ms X
Age & Sex: 55 year, Female
Address: .................................
RX
Inj Hydroxocobalamin 1 mg IM once weekly for 6 weeks
Once initial correction of the deficiency has occurred, then
Inj Hydroxocobalamin 1 mg IM once every 3 months
Tab Folic acid 10 mg orally once daily for 6 weeks
XY Jain
Stamp
t.me/Dr_Mouayyad_AlbtousH
Case Scenarios and Prescription Writing 75
CASE SCENARIO 66
A 51-year-old male presents with a history of severe episodic vertigo spells lasting
30 minutes to several hours accompanied by right sensorineural hearing loss and
tinnitus. He states that he has experienced vertigo spells with right-sided tinnitus a
few times per month for the last 6 months. Audiogram revealed a severe sensorineural
hearing loss at 35.0 dB, with a predominance of low-frequency impairment in the right
ear. Write a prescription for this patient incorporating all the elements.
Dr XY Jain
MBBS, MD (General Medicine)
Registration no. MPMC 223344
25, Park Street, Gwalior MP
Tel. no. 111222 Email: -------@-----.com
Date: 00.00.0000
Patient’s name: Mr X
Age & Sex: 51 year, Male
Address: .................................
RX
Tab Betahistine 16 mg orally 12 hrly for 7 days
XY Jain
Stamp
Alternative Agents
Tab Dimenhydrinate 50 mg orally 8 hrly for 7 days OR
Tab Meclizine 25 mg orally 12 hrly for 7 days OR
Tab Promethazine 25 mg orally 6 hrly for 7 days
t.me/Dr_Mouayyad_AlbtousH
76 Manual of Prescription Writing
CASE SCENARIO 67
A 25-year-old man presented with acute onset of intense headache, fever, nausea,
vomiting, photophobia and stiff neck. On admission, the patient’s BP was 108/50 mm Hg,
pulse rate was 114 bpm and oral temperature was 102.5°F. He was fully conscious and
clinically stable. On neurologic examination, nuchal rigidity was present and Kernig
and Brudzinski signs were positive. CSF was turbid with predominating polymorphs,
decreased glucose concentration and increased protein concentration. Culture of CSF
identified the aetiologic organism as Neisseria meningitidis. Patient was admitted in
medicine ward. Write a prescription for this patient incorporating all the elements.
Dr XY Jain
MBBS, MD (General Medicine)
Registration no. MPMC 223344
25, Park Street, Gwalior MP
Tel. no. 111222 Email: -------@-----.com
Date: 00.00.0000
Patient’s name: Mr X
Age & Sex: 25 year, Male
Address: .................................
RX
Inj Ceftriaxone 2 g IV 12 hrly 10 days
XY Jain
Stamp
Alternative Agents
Inj. Penicillin G 4 Million Units AST IV 4 hrly for 7 days OR
Inj Cefotaxime 2 g IV 6 hrly for 10 days OR
Inj Ampicillin 2 g IV 4 hrly for 7 days
Note: All close contacts should receive chemoprophylaxis with:
Tab Rifampin 600 mg orally 12 hrly for 2 days OR
Tab Azithromycin 500 mg orally single dose OR
Tab Ciprofloxacin 500 mg orally single dose OR
Inj Ceftriaxone 250 mg IM single dose
t.me/Dr_Mouayyad_AlbtousH
Case Scenarios and Prescription Writing 77
CASE SCENARIO 68
A 30-year-old male weighing 52 kg took about 300 mL of locally made alcohol. He
was brought to the emergency department 2 hours later. On examination, the patient
was drowsy. His vital signs were BP 128/82 mm Hg, pulse rate 70/min, temperature
98°F, respiratory rate 18/min. Oxygen saturation was 96% on room air. Respiratory
and cardiovascular examinations were normal. The neurological evaluation showed
evidence of bilateral nystagmus. Both pupils were dilated with sluggish light reflexes.
Arterial blood gas analysis revealed severe metabolic acidosis with pH 6.9 and
bicarbonate level being 5 mEq/L. Patient was admitted in medicine ward. Write a
prescription for this patient incorporating all the elements.
Dr XY Jain
MBBS, MD (General Medicine)
Registration no. MPMC 223344
25, Park Street, Gwalior MP
Tel. no. 111222 Email: -------@-----.com
Date: 00.00.0000
Patient’s name: Mr X
Age & Sex: 30 year, Male
Weight: 52 kg
Address: .................................
RX
Inj Ethyl Alcohol (10%) IV 50 mL in 5% dextrose slowly in one h
Followed by 7.5 mL/hr till serum methanol falls below 20 mg/dL
Inj Fomepizole 0.75 mL IV followed by 0.5 mL 12 hrly till serum methanol falls below
20 mg/dL
Inj Sodium Bicarbonate (10%) 100 mL IV 2 hrly till pH rises above 7.2
XY Jain
Stamp
t.me/Dr_Mouayyad_AlbtousH
78 Manual of Prescription Writing
CASE SCENARIO 69
A 28-year-old woman presented to the neurology clinic with a 3-month history of left-
sided pulsatile headaches occurring twice a month, each episode lasting 2–3 days. The
patient told that her headaches were usually preceded by lightheadedness and
unformed flashes of the light. Her headaches were always unilateral and were
associated with nausea, vomiting, and photophobia. She also reported sensitivity to
light and noise during these episodes. General physical and neurologic examinations
were normal. Write a prescription for this patient incorporating all the elements.
Dr XY Jain
MBBS, MD (General Medicine)
Registration no. MPMC 223344
25, Park Street, Gwalior MP
Tel. no. 111222 Email: -------@-----.com
Date: 00.00.0000
Patient’s name: Ms X
Age & Sex: 28 year, Female
Address: .................................
Diagnosis: Migraine
RX
Tab Paracetamol 500 mg orally 6 hrly for 3 days
Tab Naratriptan 2.5 mg orally at the onset, may be repeated once after 4 hr then
2.5 mg orally once daily for 3 days
Review after 3 days
XY Jain
Stamp
t.me/Dr_Mouayyad_AlbtousH
Case Scenarios and Prescription Writing 79
CASE SCENARIO 70
A 55-year-old male patient came to medical OPD with complaints of headache,
dizziness and fatigue off and on for one month. His headache was not relieved by
taking paracetamol. On examination, his BP was 150/90 mm Hg when measured
consequently for 3 days. His random blood sugar was 130 mg/dL. Blood urea was
20 mg/dL. Write a prescription for this patient incorporating all the elements.
Dr XY Jain
MBBS, MD (General Medicine)
Registration no. MPMC 223344
25, Park Street, Gwalior MP
Tel. no. 111222 Email: -------@-----.com
Date: 00.00.0000
Patient’s name: Mr X
Age & Sex: 55 year, Male
Address: .................................
RX
Tab Chlorthalidone 12.5 mg orally once daily for 7 days
Salt restriction
Regular brisk walking for at least 30 min per day
Review after 7 days
XY Jain
Stamp
Alternative Agents
Tab Lisinopril 5 mg orally once daily for 7 days OR
Tab Telmisartan 40 mg orally once daily for 7 days OR
Tab Metoprolol 50 mg orally once daily for 7 days
t.me/Dr_Mouayyad_AlbtousH
80 Manual of Prescription Writing
CASE SCENARIO 71
A 45-year-old female patient was diagnosed with hypertension 5 year back. She was
maintained on Tab Amlodipine 10 mg once daily. She came to her physician with
complaints of swelling over legs and fatigue for 20 days. Her BP was 160/90 mm Hg.
ECG was normal. Blood urea and serum creatinine were also normal. Write a
prescription for this patient incorporating all the elements.
Dr XY Jain
MBBS, MD (General Medicine)
Registration no. MPMC 223344
25, Park Street, Gwalior MP
Tel. no. 111222 Email: -------@-----.com
Date: 00.00.0000
Patient’s name: Ms X
Age & Sex: 45 year, Female
Address: .................................
RX
Tab Telmisartan 40 mg orally once daily for 7 days
Tab Metoprolol 50 mg orally once daily for 7 days
Salt restricted diet
Regular brisk walking for at least 30 min per day
Review after 7 days
XY Jain
Stamp
Alternative Agents
Tab Lisinopril 10 mg orally once daily. Tab Chlorthalidone 12.5 mg orally once daily
for 7 days
t.me/Dr_Mouayyad_AlbtousH
Case Scenarios and Prescription Writing 81
CASE SCENARIO 72
A 42-year-old woman consults her family doctor as she has planned a trip by ship. As
she has developed motion sickness on previous trips, she requests her doctor to give
her a transdermal patch that is effective for this problem about which she has heard
from a friend. Write a prescription for this patient incorporating all the elements.
Dr XY Jain
MBBS, MD (General Medicine)
Registration no. MPMC 223344
25, Park Street, Gwalior MP
Tel. no. 111222 Email: -------@-----.com
Date: 00.00.0000
Patient’s name: Ms X
Age & Sex: 42 year, Female
Address: .................................
RX
Hyoscine Hydobromide skin patch 1 mg transdermal patch to be applied locally
behind the ear 4 hr before start of journey
Repeat the same after 72 hr if required
XY Jain
Stamp
Alternative Agents
Tab Hyoscine Hydobromide 300 mcg orally once 30 min before start of journey. Repeat
after 2 hr if required OR
Inj Hyoscine 200 mcg IM if vomiting started
t.me/Dr_Mouayyad_AlbtousH
82 Manual of Prescription Writing
CASE SCENARIO 73
A 55-year-old male presented to skin outpatient department with multiple red raised
asymptomatic lesions over the body for two months. Clinical examination revealed
multiple asymptomatic erythematous plaques and well-circumscribed nodules of size
1.5–2 cm, over the forehead, nose, cheeks and both upper limbs. Bilateral ulnar nerves
were thickened, and nontender. The skin lesions showed loss of hot, cold and touch
sensations. Lepromin skin test was negative. Biopsy from the lesion showed the
presence of diffuse infiltrate of macrophages and foamy cells, with few lymphocytes
and plasma cells suggestive of tuberculoid leprosy. Slit skin smear showed acid-fast
bacilli (M. leprae). Write a prescription for this patient incorporating all the elements.
Dr XY Jain
MBBS, MD (General Medicine)
Registration no. MPMC 223344
25, Park Street, Gwalior MP
Tel. no. 111222 Email: -------@-----.com
Date: 00.00.0000
Patient’s name: Mr X
Age & Sex: 55 year, Male
Address: .................................
RX
Tab Rifampin 600 mg orally once a month on empty stomach
Tab Clofazimine 50 mg orally once daily for 1 month and 300 mg orally once a
month
Tab Dapsone 100 mg orally once daily for 1 month
To come for regular follow up every month
The colour of body secretions turns red with intake of Rifampicin
To report immediately in case of skin reaction
Refills: 11
XY Jain
Stamp
t.me/Dr_Mouayyad_AlbtousH
Case Scenarios and Prescription Writing 83
CASE SCENARIO 74
A 65-year-old woman presented to her physician with complaints of bilateral drooping
eyelids and double vision for the past two weeks. The woman also complained of
slurring of speech on continued talking, difficulty in swallowing and intermittent
weakness in her arms. Her symptoms were worse in the evenings and improved with
rest and sleep. She had notable ptosis of both eyelids. Electromyographic testing
revealed a decrementing muscle response to repetitive 2- or 3-Hz stimulation of motor
nerves. An IV dose of 2 mg of edrophonium improved her symptoms. Write a
prescription for this patient incorporating all the elements.
Dr XY Jain
MBBS, MD (General Medicine)
Registration no. MPMC 223344
25, Park Street, Gwalior MP
Tel. no. 111222 Email: -------@-----.com
Date: 00.00.0000
Patient’s name: Ms X
Age & Sex: 65 year, Female
Address: .................................
RX
Tab Neostigmine 15 mg orally 8 hrly for 15 days
Tab Prednisolone 10 mg orally once a day for 15 days
Review after 15 days
XY Jain
Stamp
t.me/Dr_Mouayyad_AlbtousH
84 Manual of Prescription Writing
CASE SCENARIO 75
A 35-year-old male from an urban slum area develops subcutaneous nodules across
his head and arms, after which he begins to experience frequent headaches with
occasional nausea and vomiting. He had two sudden attacks of seizures 3 days apart.
His seizures were generalized and tonic clonic, lasted for about 3 minutes each time
and associated with tongue bite and micturition. He has no history of trauma. On
questioning, he told that sometimes he used to take pork meat. On examination, the
patient was alert and conscious. His vital signs were normal. Cardiovascular,
respiratory and gastrointestinal systems were within normal limit. Investigations
revealed normal haematological parameters. A computed tomography (CT) scan
revealed one viable cystic lesion and one calcified lesion in the right frontal lobe of the
brain. Blood ELISA test was positive for IgG antibodies for cysticercosis. Write a
prescription for this patient incorporating all the elements.
Dr XY Jain
MBBS, MD (General Medicine)
Registration no. MPMC 223344
25, Park Street, Gwalior MP
Tel. no. 111222 Email: -------@-----.com
Date: 00.00.0000
Patient’s name: Mr X
Age & Sex: 35 year, Male
Address: .................................
Diagnosis: Neurocysticercosis
RX
Tab Albendazole 400 mg orally 12 hrly for 14 days
Tab Sodium Valproate 750 mg orally once daily for 14 days
Tab Prednisolone 40 mg orally once daily for 10 days then
20 mg orally once daily for 10 days then
10 mg orally once daily for 10 days
Start Prednisolone two days before Albendazole
Review after 14 days
XY Jain
Stamp
t.me/Dr_Mouayyad_AlbtousH
Case Scenarios and Prescription Writing 85
CASE SCENARIO 76
A 56-year-old male presented to respiratory OPD with complaints of cough with
sputum, pleuritic chest pain, and generalized weakness, loss of weight and loss of
appetite for 2 months. On examination, blood pressure was 124/82 mm Hg, respiratory
rate was 28 per min, temperature was 101ºF. Respiratory findings revealed bilateral
normal vesicular breath sounds with bilateral crepitations scattered throughout both
the lung fields. X-ray chest showed bilateral pleural effusion with underlying
consolidation. Sputum and bronchoalveolar lavage (BAL) sample were positive for
weak, acid-fast, filamentous bacilli suggestive of Nocardia. Write a prescription for
this patient incorporating all the elements.
Dr XY Jain
MBBS, MD (General Medicine)
Registration no. MPMC 223344
25, Park Street, Gwalior MP
Tel. no. 111222 Email: -------@-----.com
Date: 00.00.0000
Patient’s name: Mr X
Age & Sex: 56 year, Male
Address: .................................
Diagnosis: Nocardiosis
RX
Tab Trimethoprim/Sulfamethoxazole 160/800 mg orally 12 hrly for 1 month
Tab Paracetamol 500 mg 6–8 hrly as required for fever
Review after 1 month
Refills: 5
XY Jain
Stamp
Alternative Agents
Inj Imipenem 500 mg IV 6 hrly till required then switch to
Tab Trimethoprim/Sulfamethoxazole 160/800 mg orally 12 hrly for 6 months OR
Inj Amikacin 250 mg IV 12 hrly till required then switch to
Tab Trimethoprim/Sulfamethoxazole 160/800 mg orally 12 hrly for 6 months OR
Tab Minocycline 100 mg orally 12 hrly for 6 months
t.me/Dr_Mouayyad_AlbtousH
86 Manual of Prescription Writing
CASE SCENARIO 77
A 35-year-old married female was referred to a clinical psychologist by her family
physician. She reported about her compulsively repeating some acts in her daily life
as an obsession. She supposed that her act of repeating some actions was a result of
some specific persistent thoughts. She was obsessed with dirt and cleanliness. She
used to wash her hands many times a day as indicated by her bleeding knuckles. She
became so obsessed with cleanliness that she wanted to wash anything she touched.
Her physical examination and blood tests were normal. Write a prescription for this
patient incorporating all the elements.
Dr XY Jain
MBBS, MD (General Medicine)
Registration no. MPMC 223344
25, Park Street, Gwalior MP
Tel. no. 111222 Email: -------@-----.com
Date: 00.00.0000
Patient’s name: Ms X
Age & Sex: 35 year, Female
Address: .................................
RX
Tab Fluoxetine 10 mg orally once daily for 15 days
Review after 15 days
XY Jain
Stamp
Alternative Agents
Tab Clomipramine 10 mg orally 12 hrly for 15 days
t.me/Dr_Mouayyad_AlbtousH
Case Scenarios and Prescription Writing 87
CASE SCENARIO 78
A 55-year-old man consults a dermatologist for a persistent problem with his toenails
for four months. All of his nails on both feet have brownish-black discoloration. On
examination, the nails appear lustreless, brittle, hard, hypertrophic and the substance
of the nail is friable. The patient has tried some topical ointments without any success.
Write a prescription for this patient incorporating all the elements.
Dr XY Jain
MBBS, MD (General Medicine)
Registration no. MPMC 223344
25, Park Street, Gwalior MP
Tel. no. 111222 Email: -------@-----.com
Date: 00.00.0000
Patient’s name: Mr X
Age & Sex: 55 year, Male
Address: .................................
Diagnosis: Onychomycosis
RX
Tab Terbinafine 250 mg orally once daily for 1 month
Review after 1 month
Refills: 2
XY Jain
Stamp
Alternative Agents
Tab Fluconazole 150 mg orally once weekly for 6 months OR
Tab Itraconazole 200 mg orally once daily for 3 months
t.me/Dr_Mouayyad_AlbtousH
88 Manual of Prescription Writing
CASE SCENARIO 79
A 60-year-old man is found to have increased intraocular pressure in the right eye on
eye examination. The visual acuity is normal in both eyes. Eye examination under
mydriasis reveals increased cup-to-disc ratio in the right eye with no evidence of optic
nerve damage. Mild loss of peripheral vision is found on perimetry test. Write a
prescription for this patient incorporating all the elements.
Dr XY Jain
MBBS, MD (General Medicine)
Registration no. MPMC 223344
25, Park Street, Gwalior MP
Tel. no. 111222 Email: -------@-----.com
Date: 00.00.0000
Patient’s name: Mr X
Age & Sex: 60 year, Male
Address: .................................
RX
Betaxolol eye drops 0.5% one drop apply locally 12 hrly for 7 days
XY Jain
Stamp
Alternative Agents
Timolol eye drops 0.5% one drop apply locally 12 hrly for 7 days OR
Latanoprost eye drops 0.005% one drop apply locally once daily at night for 7 days
OR
Bimatoprost eye drops 0.3% one drop apply locally once daily at night for 7 days OR
Travoprost eye drops 0.004% one drop apply locally once daily at night for 7 days
t.me/Dr_Mouayyad_AlbtousH
Case Scenarios and Prescription Writing 89
CASE SCENARIO 80
A 25-year-old man is brought to the emergency department. He is deeply unconscious,
cyanosed and the pupils are markedly constricted. On examination, his heart rate is
62 beats per minute and respiratory rate is 7 per minute and shallow. On examination,
his pupils are pinpoint and not reactive to light. There are multiple intravenous track
marks on his arms bilaterally. On asking relatives informed that patient was dug addict
but could not name the drug. The emergency physician diagnoses it as a case of a drug
overdose. Patient was admitted in medicine ward. Write a prescription for this patient
incorporating all the elements.
Dr XY Jain
MBBS, MD (General Medicine)
Registration no. MPMC 223344
25, Park Street, Gwalior MP
Tel. no. 111222 Email: -------@-----.com
Date: 00.00.0000
Patient’s name: Mr X
Age & Sex: 25 year, Male
Address: .................................
RX
Intermittent positive pressure respiration to establish airway
Gastric lavage by pot permanganate 1:5000 solution. Give 150 mL every 30 sec
and aspirate till the colour of pot permanganate in aspirate remains pink
Dextrose 5% IV 5 mL/min
Inj Naloxone 1 mg IV stat and repeated every 5 min as needed to awaken the
patient and maintain airway protective reflexes to a total dose of 10 mg
XY Jain
Stamp
t.me/Dr_Mouayyad_AlbtousH
90 Manual of Prescription Writing
CASE SCENARIO 81
A 32-year-old male patient reported to the outpatient dental clinic with complaints of
pain and burning sensation in the mouth for 15 days which were sudden in onset,
moderate in intensity, constant in nature and aggravated while having hot and spicy
food. There was no medical and drug history revealed. On intraoral examination, the
lesion appeared creamy-white in colour without elevation seen on the hard and soft
palate, measured approximately 5 × 3 cm in size. On palpation, the lesion was non-
tender and scrapable. Brush biopsy of the lesion revealed intraepithelial pseudomycelia
of Candida albicans. Patient was not immunocompromised. Write a prescription for
this patient incorporating all the elements.
Dr XY Jain
MBBS, MD (General Medicine)
Registration no. MPMC 223344
25, Park Street, Gwalior MP
Tel. no. 111222 Email: -------@-----.com
Date: 00.00.0000
Patient’s name: Mr X
Age & Sex: 32 year, Male
Address: .................................
RX
Tab Fluconazole 100 mg orally once daily for 7 days
Clotrimazole Paint 1% apply locally with a cotton bud 8 hrly for 7 days
XY Jain
Stamp
Alternative Regimen
Tab Ketoconazole 200 mg orally once daily for 7 days
Nystatin mouth rinses 4–6 mL (400,000 to 600,000 Units) held in the mouth before
swallowing 8 hrly for 7 days
t.me/Dr_Mouayyad_AlbtousH
Case Scenarios and Prescription Writing 91
CASE SCENARIO 82
A 35-year-old male presented to the emergency department with a history of ingestion
of unknown quantity of a household insecticide which was followed by multiple
episodes of vomiting and breathing difficulty. There was no history of seizures. On
examination, he was conscious and restless with both pupils pinpoint. His blood
pressure 96/62 mm Hg, pulse 58/minute, respiratory rate 30/minute with excessive
oral secretions. Deep tendon reflexes were sluggish. Patient was admitted in medicine
ward. Write treatment for this patient.
Dr XY Jain
MBBS, MD (General Medicine)
Registration no. MPMC 223344
25, Park Street, Gwalior MP
Tel. no. 111222 Email: -------@-----.com
Date: 00.00.0000
Patient’s name: Mr X
Age & Sex: 35 year, Male
Address: .................................
RX
Inj Atropine 2 mg IV stat and then repeat every 10 minutes until dryness of mouth
occurs max dose 200 mg
Inj Pralidoxime 1 g in 100 mL normal saline IV in 1 hr
Repeat after 1 hr in the same dose, if required, up to max 12 g in 24 hr
Admit patient in aerated room. Check vitals at regular interval
XY Jain
Stamp
t.me/Dr_Mouayyad_AlbtousH
92 Manual of Prescription Writing
CASE SCENARIO 83
A 60-year-old female, weighing 65 kg, presents with pain in the right knee which
started 6 months back. The pain was insidious in onset. She states that the pain is
aggravated by walking and climbing stairs and is relieved by rest. The pain is associated
with a limited range of movements in the right knee. She tells that similar complaint
developed in the left knee one month back but it was mild. There is no history of
trauma. X-ray of the right knee reveals joint space narrowing, subchondral sclerosis
and osteophytes. Write a prescription for this patient incorporating all the elements.
Dr XY Jain
MBBS, MD (General Medicine)
Registration no. MPMC 223344
25, Park Street, Gwalior MP
Tel. no. 111222 Email: -------@-----.com
Date: 00.00.0000
Patient’s name: Ms X
Age & Sex: 60 year, Female
Address: .................................
Diagnosis: Osteoarthritis
RX
Tab Paracetamol 500 mg orally 6 hrly for 7 days
Review after 7 days
Avoid squatting posture
XY Jain
Stamp
t.me/Dr_Mouayyad_AlbtousH
Case Scenarios and Prescription Writing 93
CASE SCENARIO 84
A 68-year-old female attends orthopaedic OPD with complaint of diffuse bone pain
for the last 2 months. She states any weight-bearing activity causes her severe
discomfort. She has a history of hypertension for which she is taking
hydrochlorothiazide 25 mg orally daily for the last two year. Her bone mineral density
T-score is –2.6 at the hip and –2.0 at the spine. Write a prescription for this patient
incorporating all the elements.
Dr XY Jain
MBBS, MD (General Medicine)
Registration no. MPMC 223344
25, Park Street, Gwalior MP
Tel. no. 111222 Email: -------@-----.com
Date: 00.00.0000
Patient’s name: Ms X
Age & Sex: 68 year, Female
Address: .................................
Diagnosis: Osteoporosis
RX
Tab Alendronate 5 mg orally once daily for one month
Take half an hr before first food or drink with full glass of water and remain in
sitting posture at least for 30 min to avoid oesophagitis
Treatment will be continued based on response
XY Jain
Stamp
Alternative Agents
Tab Alendronate 70 mg orally once weekly for a month OR
Tab Ibandronate 150 mg orally once a month OR
Tab Risedronate 35 mg orally weekly for a month OR
Tab Risedronate 75 mg for two consecutive days orally once a month OR
Inj. Zolendronate 4 mg IV slow infusion in 5 min once a year OR
t.me/Dr_Mouayyad_AlbtousH
94 Manual of Prescription Writing
CASE SCENARIO 85
A 62-year-old man noticed progressively worsening tremors in his hands as well as
rigidity involving the right side and bradykinesia for the past 6 months. The tremors
were noticed at rest and disappeared with activity. He walked slowly and also
developed slowness in his daily activities. His gait is characterized by small shuffling
steps and a loss of the normal automatic arm swing. A right-sided resting tremor of
the pill-rolling type was observed, which decreased during finger-to-nose coordination
testing. A cogwheel rigidity was elicited by passively bending both arms. Write a
prescription for this patient incorporating all the elements.
Dr XY Jain
MBBS, MD (General Medicine)
Registration no. MPMC 223344
25, Park Street, Gwalior MP
Tel. no. 111222 Email: -------@-----.com
Date: 00.00.0000
Patient’s name: Mr X
Age & Sex: 62 year, Male
Address: .................................
Diagnosis: Parkinsonism
RX
Tab Levodopa/Carbidopa (250/25 mg) orally 8 hrly for 15 days
Review after 15 days
XY Jain
Stamp
Alternative Agents
Tab Pramipexole 0.25–1 mg orally 8 hrly OR
Tab Ropinirole 6–24 mg orally once daily OR
Tab Selegiline 5 mg orally 12 hrly OR
Tab Entacapone 200 mg orally 8 hrly with each Levodopa/Carbidopa dose OR
Tab Tolcapone 100 mg orally 8 hrly with each Levodopa/Carbidopa dose OR
Tab Trihexyphenidyl 2 mg orally 8 hrly
t.me/Dr_Mouayyad_AlbtousH
Case Scenarios and Prescription Writing 95
CASE SCENARIO 86
A 30-year-old man reported a large single, hypopigmented, well-defined anaesthetic
lesion on his right thigh which had been present for six months. His vitals were normal.
On local examination there was a well-defined, hypopigmented, macule of 5 cm with
anhidrosis and a raised granular margin associated with decreased sensation of touch,
temperature and pain. There was no other nerve involvement. Lepromin skin test was
positive. Biopsy from the lesion showed the presence of numerous epithelioid
granulomas in the superficial and deep dermis. Slit skin smear showed acid-fast bacilli
(M. leprae).Write a prescription for this patient incorporating all the elements.
Dr XY Jain
MBBS, MD (General Medicine)
Registration no. MPMC 223344
25, Park Street, Gwalior MP
Tel. no. 111222 Email: -------@-----.com
Date: 00.00.0000
Patient’s name: Mr X
Age & Sex: 30 year, Male
Address: .................................
RX
Tab Rifampicin 600 mg orally once a month
Tab Dapsone 100 mg orally once daily for one month
To report immediately in case of skin reaction
The color of body secretions will turn red with intake of Rifampicin
Regular follow up every month
Refills: 5
XY Jain
Stamp
t.me/Dr_Mouayyad_AlbtousH
96 Manual of Prescription Writing
CASE SCENARIO 87
A mother of 10-year-old child came to the physician with complaints of presence of
lice and itching over scalp of her child who is otherwise asymptomatic. Write a
prescription for this patient incorporating all the elements.
Dr XY Jain
MBBS, MD (General Medicine)
Registration no. MPMC 223344
25, Park Street, Gwalior MP
Tel. no. 111222 Email: -------@-----.com
Date: 00.00.0000
Patient’s name: Ms X
Age & Sex: 10 year, Male
Address: .................................
Diagnosis: Pediculosis
RX
Permethrin 1% lotion apply on scalp, allow to remain for 10 minutes then wash
with water
Repeat the same after 7–10 days if lice are detected again.
XY Jain
Stamp
t.me/Dr_Mouayyad_AlbtousH
Case Scenarios and Prescription Writing 97
CASE SCENARIO 88
A 74-year-old woman presented to her family physician complaining of epigastric
pain for 15 days. The pain occurred daily, was well localized, dull, non-radiating and
increased at night and between meals. The patient reported relief of pain with antacids.
She had been suffering from osteoarthritis for 3 months and had been taking ibuprofen
two to three times daily to relieve her pain. On examination, her vitals were normal.
Abdominal examination showed normal bowel sounds and mild epigastric tenderness.
Upper endoscopy revealed a 4 cm clean-based gastric ulcer, and a biopsy of the ulcer
site identified Helicobacter pylori. Write a prescription for this patient incorporating all
the elements.
Dr XY Jain
MBBS, MD (General Medicine)
Registration no. MPMC 223344
25, Park Street, Gwalior MP
Tel. no. 111222 Email: -------@-----.com
Date: 00.00.0000
Patient’s name: Ms X
Age & Sex: 74 year, Female
Address: .................................
XY Jain
Stamp
t.me/Dr_Mouayyad_AlbtousH
98 Manual of Prescription Writing
CASE SCENARIO 89
A 35-year-old male presents with swollen gums, bleeding from gums, pain on eating
something hard, difficulty in chewing food, pus discharge from gum on pressing and
loosening of teeth. He also complains of bad breath (halitosis) and sensitivity to hot
and cold intakes. There is slowly progressive destruction of periodontium, loss of
periodontal attachment with presence of periodontal pocket. Write a prescription for
this patient incorporating all the elements.
Dr XY Jain
MBBS, MD (General Medicine)
Registration no. MPMC 223344
25, Park Street, Gwalior MP
Tel. no. 111222 Email: -------@-----.com
Date: 00.00.0000
Patient’s name: Mr X
Age & Sex: 35 year, Male
Address: .................................
Diagnosis: Periodontitis
RX
Cap Amoxicillin 250 mg orally 8 hrly for 7 days
Tab Metronidazole 400 mg orally 8 hrly for 7 days
Tab Ibuprofen 400 mg orally 8 hrly for 5 days
Chlorhexidine mouthwash 0.2% 12 hrly
XY Jain
Stamp
t.me/Dr_Mouayyad_AlbtousH
Case Scenarios and Prescription Writing 99
CASE SCENARIO 90
A 46-year-old woman presents with a dry hacking, prolonged, paroxysmal cough for
the last 2 weeks. She states that the coughing spells are difficult to stop, and her sleep
is often interrupted by coughing. She also reported occasional wheezing and chest
tightness. During physical examination, the patient has several episodes of numerous
rapid coughs, each followed by an inspiratory effort that includes a high-pitched
whoop. On physical examination, her temperature 99.4°F, heart rate 94 beats per minute,
blood pressure 140/90 mm Hg and oxygen saturation 98%. On lung auscultation, chest
sounds clear. Culture of nasopharyngeal swab showed growth of Bordetella pertussis.
Write a prescription for this patient incorporating all the elements.
Dr XY Jain
MBBS, MD (General Medicine)
Registration no. MPMC 223344
25, Park Street, Gwalior MP
Tel. no. 111222 Email: -------@-----.com
Date: 00.00.0000
Patient’s name: Ms X
Age & Sex: 46 year, Female
Address: .................................
Diagnosis: Pertussis
RX
Tab Azithromycin 500 mg orally on day 1. Followed by:
Tab Azithromycin 250 mg orally once daily for 4 days
XY Jain
Stamp
Alternative Agents
Tab Erythromycin Estolate 500 mg orally 8 hrly for 14 days OR
Tab Clarithromycin 500 mg orally 12 hrly for 14 days OR
Tab Trimethoprim/Sulfamethoxazole 160/800 mg orally 12 hrly for 14 days
t.me/Dr_Mouayyad_AlbtousH
100 Manual of Prescription Writing
CASE SCENARIO 91
A 34-year-old female presented with complaints of sudden onset and worsening
symptoms of fever, chills, fatigue, nausea, vomiting, myalgia, productive cough and
dyspnoea. The symptoms began the day before her hospital visit and severely worsened
overnight. On physical examination, her temperature was 102.5°F and respiratory rate
was 24/minute. Chest X-rays showed bilateral infiltrates and consolidation consistent
with pneumonia. Sputum culture showed the presence of Yersinia pestis. Write a
prescription for this patient incorporating all the elements.
Dr XY Jain
MBBS, MD (General Medicine)
Registration no. MPMC 223344
25, Park Street, Gwalior MP
Tel. no. 111222 Email: -------@-----.com
Date: 00.00.0000
Patient’s name: Ms X
Age & Sex: 34 year, Female
Address: .................................
Diagnosis: Plague
RX
Inj Streptomycin 1 g IM 12 hrly for 10 days
Tab Paracetamol 500 mg orally 6–8 hrly as required for fever
XY Jain
Stamp
Alternative Agents
Cap Doxycycline 100 mg orally 12 hrly for 10 days OR
Cap Tetracycline 500 mg orally 6 hrly for 10 days OR
Tab Ciprofloxacin 1 g orally 12 hrly for 10 days OR
Tab Levofloxacin 500 mg orally once daily for 10 days
t.me/Dr_Mouayyad_AlbtousH
Case Scenarios and Prescription Writing 101
CASE SCENARIO 92
A 60-year-old male presents to his family physician with complaints of fever, chills,
malaise, dyspnea and productive cough for 5 days. The sputum is mucoid, purulent
and blood-tinged. He also complains of pain in his right chest that is more intense
during inspiration. On examination, his blood pressure 150/84 mm Hg, heart rate
114/minute, respiratory rate 24/minute and temperature 102.4°F. Auscultation of chest
reveals bilateral bronchial breath sounds, rhonchi and inspiratory crackles. Gram stain
of sputum specimen shows gram-positive diplococci. Sputum culture reveals growth
of Streptococcus pneumoniae. Write a prescription for this patient incorporating all the
elements.
Dr XY Jain
MBBS, MD (General Medicine)
Registration no. MPMC 223344
25, Park Street, Gwalior MP
Tel. no. 111222 Email: -------@-----.com
Date: 00.00.0000
Patient’s name: Mr X
Age & Sex: 60 year, Male
Address: .................................
RX
Cap Amoxicillin 500 mg orally 8 hrly for 7 days
Tab Bromhexine 8 mg orally 8 hrly
Tab Paracetamol 500 mg orally 6–8 hrly as required for fever
XY Jain
Stamp
t.me/Dr_Mouayyad_AlbtousH
102 Manual of Prescription Writing
CASE SCENARIO 93
A 45-year-old man presented to the clinic with complaints of fever, progressive
dyspnoea, weight loss and dry cough for the last 2 months. He had been treated with
broad-spectrum antibiotics for a presumed diagnosis of pneumonia without any relief
and most recently had finished a course of amoxicillin for 7 days. He had noticed a
significant decrease in appetite in the immediate past and has lost approximately
8–10 kg of weight. On examination, temperature was 102.5°F, respiratory rate was
38/min, pulse rate 110/min and blood pressure 128/84 mm Hg. Chest examination
revealed wheezes, rhonchi and crepitations on both sides. Other systems were normal.
Pneumocystis jiroveci was found in the sputum but he proved to be HIV negative. Write
a prescription for this patient incorporating all the elements.
Dr XY Jain
MBBS, MD (General Medicine)
Registration no. MPMC 223344
25, Park Street, Gwalior MP
Tel. no. 111222 Email: -------@-----.com
Date: 00.00.0000
Patient’s name: Mr X
Age & Sex: 45 year, Male
Address: .................................
RX
Tab Trimethoprim/Sulfamethoxazole 320/1600 mg orally 8 hrly for 21 days
Tab Prednisolone 40 mg orally 12 hrly for 5 days
40 mg orally once daily for 5 days
20 mg orally once daily for 11days
XY Jain
Stamp
Alternative Agents
Tab Clindamycin 600 mg orally 8 hrly for 21 days +
Tab Primaquine 30 mg orally once daily for 21 days OR
Inj Pentamidine 200 mg IV once daily for 21 days
t.me/Dr_Mouayyad_AlbtousH
Case Scenarios and Prescription Writing 103
CASE SCENARIO 94
A 29-year-old primigravida delivered healthy male baby and was given Inj Ergometrine
0.3 mg IM at the delivery of anterior shoulder. After 1 hr of delivery when patient was
shifted in the ward she complained of excessive bleeding per vagina. Her blood pressure
was 100/60 mm Hg. Pulse rate was 112/min. Write a prescription for this patient
incorporating all the elements.
Dr XY Jain
MBBS, MD (General Medicine)
Registration no. MPMC 223344
25, Park Street, Gwalior MP
Tel. no. 111222 Email: -------@-----.com
Date: 00.00.0000
Patient’s name: Ms X
Age & Sex: 29 year, Female
Address: .................................
RX
Inj Oxytocin 10 IU in 500 mL glucose at the rate of 10 mL/min till required
Inj Carboprost 250 mcg IM 2 hrly till required max 3 mg in 24 hr
XY Jain
Stamp
Alternative Agents
Inj Dinoprostone 0.25 mg IM 2 hrly till required
t.me/Dr_Mouayyad_AlbtousH
104 Manual of Prescription Writing
CASE SCENARIO 95
A 32-year-old female presented to her family physician with a 5-day history of greenish,
foul-smelling watery diarrhea, fever and abdominal pain. The diarrhea occurred
3 days after starting clindamycin therapy for a recent tooth extraction due to a dental
abscess. Physical examination of the patient revealed heart rate 110/min, and blood
pressure 130/86 mm Hg. Laboratory investigations showed leukocytosis. Stool
specimen was positive for C. difficile toxin. Write a prescription for this patient
incorporating all the elements.
Dr XY Jain
MBBS, MD (General Medicine)
Registration no. MPMC 223344
25, Park Street, Gwalior MP
Tel. no. 111222 Email: -------@-----.com
Date: 00.00.0000
Patient’s name: Ms X
Age & Sex: 32 year, Female
Address: .................................
RX
Tab Metronidazole 500 mg orally 8 hrly for 10 days
XY Jain
Stamp
Alternative Agents
Cap Vancomycin 125 mg orally 6 hrly for 10 days OR
Tab Fidaxomicin 200 mg orally 12 hrly for 10 days
t.me/Dr_Mouayyad_AlbtousH
Case Scenarios and Prescription Writing 105
CASE SCENARIO 96
A 26-year-old male, weighing 55 kg, presented with one month history of cough with
expectoration, fever, night sweats, chills, loss of appetite, retrosternal chest pain,
dyspnea on exertion and weight loss. On physical examination, the patient appeared
chronically ill and malnourished. On chest examination, a few post-tussive rales were
noted at right apex posteriorly. X-ray chest revealed a patchy area of consolidation
with a well-defined cavitary lesion in the right upper lobe. Sputum smear was positive
for acid-fast bacilli. Write a prescription for this patient incorporating all the elements.
Dr XY Jain
MBBS, MD (General Medicine)
Registration no. MPMC 223344
25, Park Street, Gwalior MP
Tel. no. 111222 Email: -------@-----.com
Date: 00.00.0000
Patient’s name: Mr X
Age & Sex: 26 year, Male
Weight: 55 kg
Address: .................................
RX
Tab Isoniazid 300 mg orally once daily for 1 month
Cap Rifampicin 450 mg orally once daily on empty stomach for 1 month
Tab Pyrazinamide 750 mg orally 12 hrly daily for 1 month
Tab Ethambutol 800 mg orally once daily for 1 month
Tab Pyridoxine 10 mg orally once daily for 1 month
Follow up after one month
Ophthalmic examination before starting Ethambutol therapy
Rifampicin makes colour of urine red
Refills: 1
XY Jain
Stamp
t.me/Dr_Mouayyad_AlbtousH
106 Manual of Prescription Writing
CASE SCENARIO 97
A 60-year-old woman presents to an orthopedician with complaints of generalized
muscle and joint pain for the past 1 year. She also has morning stiffness of joints that
lasts more than 1 h every morning. She states that her joints of both hands are swollen
and painful and that she has been taking ibuprofen and paracetamol to control her
pain but in spite of that she has to restrict her physical activities due to pain. Her vital
signs are normal. Physical examination reveals bilateral symmetrical swelling,
tenderness, and warmth of the metacarpophalangeal and proximal interphalangeal
joints of both hands and the metatarsophalangeal joints of the feet. Blood haematology
shows raised erythrocyte sedimentation rate and rheumatoid factor. X-ray of hands
and feet show narrowing of joint spaces and marginal erosion of the articular surfaces.
Write a prescription for this patient incorporating all the elements.
Dr XY Jain
MBBS, MD (General Medicine)
Registration no. MPMC 223344
25, Park Street, Gwalior MP
Tel. no. 111222 Email: -------@-----.com
Date: 00.00.0000
Patient’s name: Ms X
Age & Sex: 60 year, Female
Address: .................................
RX
Tab Ibuprofen 400 mg orally 12 hrly for 7 days
Tab Prednisolone 7.5 mg orally once daily for 7 days
Tab Methotrexate 10 mg orally once weekly for 7 days
Tab Folic Acid 1 mg orally once daily for 7 days except on the day of
Methotrexate.
Review after 7 days for dose escalation
XY Jain
Stamp
t.me/Dr_Mouayyad_AlbtousH
Case Scenarios and Prescription Writing 107
CASE SCENARIO 98
A 36-year-old man presented to the dermatology clinic with rashes of one-month
duration that were extremely itchy, particularly at night. Physical examination revealed
numerous small erythematous vesicles, pustules and wavy thread-like grey burrows
measuring 1 to 10 mm in diameter distributed primarily around the wrists, ankles,
umbilicus and interdigital spaces of the hands and feet. Dermoscopic examination of
the burrows revealed distinctive brown-coloured triangular structures that were
removed with a fine scalpel. Microscopic examination of the tissue revealed moving
mites, eggs, and faecal pellets. Write a prescription for this patient incorporating all
the elements.
Dr XY Jain
MBBS, MD (General Medicine)
Registration no. MPMC 223344
25, Park Street, Gwalior MP
Tel. no. 111222 Email: -------@-----.com
Date: 00.00.0000
Patient’s name: Mr X
Age & Sex: 36 year, Male
Address: .................................
Diagnosis: Scabies
RX
Tab Ivermectin 12 mg orally as a single dose to be repeated after 2 weeks.
Permethrin 5% cream apply once all over the body from the neck down up to
toes after bathing then wash off after 8–12 hr
Repeat after 1 week
Apply to all members of the family
Disinfect the clothes and bed sheets by boiling or ironing
XY Jain
Stamp
t.me/Dr_Mouayyad_AlbtousH
108 Manual of Prescription Writing
CASE SCENARIO 99
A 28-year-old male was referred by his family physician to psychiatric unit of a tertiary
care hospital, with complaints of hearing voices for the past 6 months, restlessness,
irritability and loss of appetite. The patient experienced auditory hallucinations. He
complained of hearing voices of people talking about him and engaged in third person
conversations involving both a man and a woman. The patient stated that he felt unsafe
and thought that people wanted to harm him. On examination, he looks tired, depressed
and avoided eye contact. Physical examination showed a thin, dishevelled-looking
man with very poor hygiene, who appeared much older than his stated age, with
normal vitals. Write a prescription for this patient incorporating all the elements.
Dr XY Jain
MBBS, MD (General Medicine)
Registration no. MPMC 223344
25, Park Street, Gwalior MP
Tel. no. 111222 Email: -------@-----.com
Date: 00.00.0000
Patient’s name: Mr X
Age & Sex: 28 year, Male
Address: .................................
Diagnosis: Schizophrenia
RX
Tab Olanzapine 20 mg orally once daily for 15 days
Review after 15 days
XY Jain
Stamp
Alternative Agents
Tab Aripriprazole 10 mg orally once daily for 15 days OR
Tab Risperidone 2 mg orally once daily for 15 days OR
Tab Quetiapine 50 mg orally 12 hrly for 15 days OR
Tab Haloperidol 5 mg orally 12 hrly for 15 days
t.me/Dr_Mouayyad_AlbtousH
Case Scenarios and Prescription Writing 109
Dr XY Jain
MBBS, MD (General Medicine)
Registration no. MPMC 223344
25, Park Street, Gwalior MP
Tel. no. 111222 Email: -------@-----.com
Date: 00.00.0000
Patient’s name: Mr X
Age & Sex: 54 year, Male
Address: .................................
Diagnosis: Snakebite
RX
Anti-snake venom (ASV) 10 vials (100 mL) IV start with 10–15 drops/min for 15 min
and watch for reaction. Continue ASV drip if no signs of reaction and to be
completed in 1 hr followed by second dose after 1 hr if there is no improvement
Inj Atropine 0.6 mg IV stat and then every 30 min max 5 dose
Inj Neostigmine Methyl Sulphate 2 mg IV every 30 min max 5 dose
Monitor vital signs at 5 min interval.
XY Jain
Stamp
t.me/Dr_Mouayyad_AlbtousH
110 Manual of Prescription Writing
Dr XY Jain
MBBS, MD (General Medicine)
Registration no. MPMC 223344
25, Park Street, Gwalior MP
Tel. no. 111222 Email: -------@-----.com
Date: 00.00.0000
Patient’s name: Ms X
Age & Sex: 28 year, Female
Address: .................................
RX
Inj Lorazepam 4 mg IV in 2 min repeat after 10 min if required (max 3 doses)
Review every 10 min till control of seizures
XY Jain
Stamp
Alternative Agents
Inj Diazepam 10 mg IV slowly OR
Inj Midazolam 10 mg IM repeat after 10 min
Note: If seizures uncontrolled with above treatment Inj Fosphenytoin (20 mg/kg) IV
at a rate of 50 mg/min.
If still seizures continue Inj Phenobarbital 10–20 mg/kg IV slowly (50 mg/min).
t.me/Dr_Mouayyad_AlbtousH
Case Scenarios and Prescription Writing 111
Dr XY Jain
MBBS, MD (General Medicine)
Registration no. MPMC 223344
25, Park Street, Gwalior MP
Tel. no. 111222 Email: -------@-----.com
Date: 00.00.0000
Patient’s name: Mr X
Age & Sex: 28 year, Male
Address: .................................
Diagnosis: Syphilis
RX
Inj Benzathine Penicillin G 2.4 Million Units deep IM once weekly for 3 weeks
Give first injection after sensitivity test (AST)
XY Jain
Stamp
Alternative Agents
Inj Ceftriaxone 1 g IM once daily for 10 days OR
Cap Doxycycline 100 mg orally 12 hrly for 14 days OR
Cap Tetracycline 500 mg orally 6 hrly for 14 days
t.me/Dr_Mouayyad_AlbtousH
112 Manual of Prescription Writing
Dr XY Jain
MBBS, MD (General Medicine)
Registration no. MPMC 223344
25, Park Street, Gwalior MP
Tel. no. 111222 Email: -------@-----.com
Date: 00.00.0000
Patient’s name: Mr X
Age & Sex: 28 year, Male
Address: .................................
RX
Tab Praziquantel 500 mg orally single dose
XY Jain
Stamp
Alternative Agents
Tab Niclosamide 500 mg 2 such tab with one glass of water on empty stomach then
2 tab after one h followed by
Tab Docusate 100 mg orally after 2 hr.
t.me/Dr_Mouayyad_AlbtousH
Case Scenarios and Prescription Writing 113
Dr XY Jain
MBBS, MD (General Medicine)
Registration no. MPMC 223344
25, Park Street, Gwalior MP
Tel. no. 111222 Email: -------@-----.com
Date: 00.00.0000
Patient’s name: Ms X
Age & Sex: 45 year, Female
Address: .................................
RX
Inj Propranolol 1 mg IV slowly saturated solution of pot iodide 10 drops orally
Tab Propylthiouracil 250 mg orally 6 hrly
Inj Hydrocortisone 100 mg IV 6 hrly
XY Jain
Stamp
Alternative to IV Propranolol
Tab Propranolol 40–80 mg orally 6 hrly OR
Tab Diltiazem 90 mg orally 8 hrly OR
Inj Diltiazem 10 mg/hr IV
Note: Plasmapheresis or peritoneal dialysis to remove protein bound hormone if above
measures fail to control disease.
t.me/Dr_Mouayyad_AlbtousH
114 Manual of Prescription Writing
Dr XY Jain
MBBS, MD (General Medicine)
Registration no. MPMC 223344
25, Park Street, Gwalior MP
Tel. no. 111222 Email: -------@-----.com
Date: 00.00.0000
Patient’s name: Mr X
Age & Sex: 21 year, Male
Address: .................................
RX
Tab Terbinafine 250 mg orally once daily for 4 weeks
Miconazole 2% cream apply locally 12 hrly for 2 weeks
XY Jain
Stamp
Alternative to Terbinafine
Tab Ketoconazole 200 mg orally once daily for 2–4 weeks OR
Tab Itraconazole 200 mg orally once daily for 2–4 weeks OR
Tab Fluconazole 200 mg orally once daily for 2–4 weeks
t.me/Dr_Mouayyad_AlbtousH
Case Scenarios and Prescription Writing 115
Dr XY Jain
MBBS, MD (General Medicine)
Registration no. MPMC 223344
25, Park Street, Gwalior MP
Tel. no. 111222 Email: -------@-----.com
Date: 00.00.0000
Patient’s name: Mr X
Age & Sex: 35 year, Male
Weight: 68 kg
Address: .................................
Diagnosis: Toxoplasmosis
RX
Tab Pyrimethamine 200 mg orally as first dose then 75 mg once daily for 6 weeks
Tab Sulfadiazine 1500 mg orally 6 hrly for 6 weeks
Tab Folinic Acid 10 mg orally once daily for 6 weeks
Tab Paracetamol 500 mg orally 6–8 hrly as required for fever
XY Jain
Stamp
Alternative Agents
Tab Clindamycin 600 mg orally 6 hrly for 6 weeks OR
Tab Trimethoprim/Sulfamethoxazole 160/800 mg orally 8 hrly for 6 weeks OR
Tab Azithromycin 1 g orally once daily for 6 weeks OR
Tab Clarithromycin 500 mg orally 12 hrly for 6 weeks
t.me/Dr_Mouayyad_AlbtousH
116 Manual of Prescription Writing
Dr XY Jain
MBBS, MD (General Medicine)
Registration no. MPMC 223344
25, Park Street, Gwalior MP
Tel. no. 111222 Email: -------@-----.com
Date: 00.00.0000
Patient’s name: Ms X
Age & Sex: 32 year, Female
Address: .................................
RX
Tab Metronidazole 400 mg orally 12 hrly for 7 days
XY Jain
Stamp
Alternative Agents
Tab Metronidazole 2 g orally single dose OR
Tab Tinidazole 2 g orally single dose OR
Tab Secnidazole 2 g orally single dose
Note: Because the Trichomonas vaginalis is a sexually transmitted infection treatment
of both partners simultaneously is recommended.
t.me/Dr_Mouayyad_AlbtousH
Case Scenarios and Prescription Writing 117
Dr XY Jain
MBBS, MD (General Medicine)
Registration no. MPMC 223344
25, Park Street, Gwalior MP
Tel. no. 111222 Email: -------@-----.com
Date: 00.00.0000
Patient’s name: Mr X
Age & Sex: 42 year, Male
Address: .................................
Trigeminal Neuralgia
RX
Tab Carbamazepine 200 mg orally 12 hrly for 5 days
XY Jain
Stamp
Alternative Agents
Tab Oxcarbazepine 300 mg orally 12 hrly
t.me/Dr_Mouayyad_AlbtousH
118 Manual of Prescription Writing
Dr XY Jain
MBBS, MD (General Medicine)
Registration no. MPMC 223344
25, Park Street, Gwalior MP
Tel. no. 111222 Email: -------@-----.com
Date: 00.00.0000
Patient’s name: Mr X
Age & Sex: 54 year, Male
Address: .................................
Diagnosis: Tularemia
RX
Inj Streptomycin 1 g IM 12 hrly for 10 days
XY Jain
Stamp
Alternative Agents
Cap Doxycycline 100 mg orally 12 hrly for 14 days OR
Cap Tetracycline 500 mg orally 6 hrly for 14 days
t.me/Dr_Mouayyad_AlbtousH
Case Scenarios and Prescription Writing 119
Dr XY Jain
MBBS, MD (General Medicine)
Registration no. MPMC 223344
25, Park Street, Gwalior MP
Tel. no. 111222 Email: -------@-----.com
Date: 00.00.0000
Patient’s name: Mr X
Age & Sex: 32 year, Male
Address: .................................
RX
Inj Ceftriaxone 2 g IV once daily for 14 days
Tab Paracetamol 500 mg 6–8 hrly as required for fever
Drink only boiled water
Avoid raw fruits and vegetables
XY Jain
Stamp
Alternative Agents
Tab Azithromycin 500 mg orally 12 hrly for 7 days OR
Tab Trimethoprim/Sulfamethoxazole 160/800 mg orally 12 hrly for 14 days OR
Tab Ciprofloxacin 750 mg orally 12 hrly for 14 days OR
Tab Levofloxacin 500 mg orally once daily for 14 days OR
Cap Amoxicillin 1 g orally 8 hrly for 14 days OR
Tab Cefixime 400 mg orally 12 hrly for 14 days
t.me/Dr_Mouayyad_AlbtousH
120 Manual of Prescription Writing
Dr XY Jain
MBBS, MD (General Medicine)
Registration no. MPMC 223344
25, Park Street, Gwalior MP
Tel. no. 111222 Email: -------@-----.com
Date: 00.00.0000
Patient’s name: Ms X
Age & Sex: 35 year, Female
Address: .................................
RX
Tab Nitrofurantoin 100 mg orally 12 hrly for 7 days
Tab Paracetamol 500 mg orally 6–8 hrly as required for fever
XY Jain
Stamp
Alternative Agents
Tab Trimethoprim/Sulfamethoxazole (160/800 mg) orally 12 hrly for 7 days OR
Tab Ciprofloxacin 500 mg orally 12 hrly for 3 days OR
Tab Cefuroxime 250 mg orally twice daily for 5 days
t.me/Dr_Mouayyad_AlbtousH
Case Scenarios and Prescription Writing 121
Dr XY Jain
MBBS, MD (General Medicine)
Registration no. MPMC 223344
25, Park Street, Gwalior MP
Tel. no. 111222 Email: -------@-----.com
Date: 00.00.0000
Patient’s name: Ms X
Age & Sex: 65 year, Female
Address: .................................
RX
Tab Tolterodine 1 mg orally 12 hrly for 15 days
Do daily pelvic floor exercises. Avoid lifting weight
Review after 15 days
XY Jain
Stamp
Alternative Agents
Tab Oxybutynin 2.5 mg orally 12 hrly for 15 days OR
Tab Darifenacin 7.5 mg orally once daily for 15 days OR
Tab Solifenacin 5 mg orally once daily for 15 days OR
Tab Mirabegron 25 mg orally once daily for 15 days
t.me/Dr_Mouayyad_AlbtousH
122 Manual of Prescription Writing
Dr XY Jain
MBBS, MD (General Medicine)
Registration no. MPMC 223344
25, Park Street, Gwalior MP
Tel. no. 111222 Email: -------@-----.com
Date: 00.00.0000
Patient’s name: Ms X
Age & Sex: 30 year, Female
Address: .................................
RX
Tab Bethanechol 25 mg orally 8 hrly on empty stomach until required.
XY Jain
Stamp
t.me/Dr_Mouayyad_AlbtousH
Case Scenarios and Prescription Writing 123
Dr XY Jain
MBBS, MD (General Medicine)
Registration no. MPMC 223344
25, Park Street, Gwalior MP
Tel. no. 111222 Email: -------@-----.com
Date: 00.00.0000
Patient’s name: Ms X
Age & Sex: 27 year, Female
Address: .................................
RX
Inj Oxytocin 5 International Units in 500 mL glucose at the rate of 2–4 Milli
International Units/min
Rate of infusion can be increased up to 20 Milli International Units /min with close
monitoring of fetal heart and uterine contractions.
Not more than 5 International Units are to be administered in 24 hr
XY Jain
Stamp
Alternative Agents
Tab Dinoprostone 10 mg per vaginal for 12 hr and then remove
Direction: To remain in recumbent position during and up to 2 hr of insertion.
t.me/Dr_Mouayyad_AlbtousH
124 Manual of Prescription Writing
Dr XY Jain
MBBS, MD (General Medicine)
Registration no. MPMC 223344
25, Park Street, Gwalior MP
Tel. no. 111222 Email: -------@-----.com
Date: 00.00.0000
Patient’s name: Ms X
Age & Sex: 30 year, Female
Address: .................................
RX
Tab Fluconazole 150 mg orally single dose after food
Clotrimazole vaginal pessary 100 mg apply locally for 6 nights
XY Jain
Stamp
Alternative to Fluconazole
Tab Itraconazole 200 mg orally once daily for 3 days
Alternative to Clotrimazole
Clotrimazole vaginal pessary 200 mg apply locally for 3 nights OR
Clotrimazole vaginal pessary 500 mg apply locally for one night OR
Nystatin vaginal cream 100,000 Units apply locally for 14 nights or twice a day for
one week.
t.me/Dr_Mouayyad_AlbtousH
Communication with Patients on Prescriptions 125
Communication with
Patients on Prescriptions
Need of Communication
Communication is the transfer of appropriate information from the prescriber to the
patient either verbally or/and in writing. Prescription, though written carefully and
correctly may be ineffective unless communicated properly to the patient and the
pharmacists. Registered medical practitioners should have proper communication skill
for successful prescribing and health care of society. Communication skill was not
paid much attention in the old curriculum. It is an art that requires training so it has
now been included in the revised curriculum. Training on communication at an early
stage of education period to undergraduate medical students is essential to ensure
1. Positive impact on patient–doctor relationship: If the doctor gives attention to the patient’s
concern and addresses his expectation about his illness, a strong relationship between
patient and doctor is built.
2. Patient’s satisfaction: If the doctor listens patiently, gives sufficient time and then
explains to him with empathy it results in better patient satisfaction.
3. Compliance of the treatment: When quality information is given to the patient then
the patient will have a greater understanding of his problem and drugs given to
him for treatment. This contributes to better adherence to treatment strategy. Elderly
patients suffering from comorbidities and on multitherapy are at a higher risk of
noncompliance.
4. Prevention of drug prescription errors: Positive interaction with patients encourages
them to follow medical recommendations correctly, thereby preventing prescription
errors.
5. Confidence of the prescriber: Improvement in patient-centered communication helps
even better confidence in the doctor. It allows the physician to provide competent
and economically prudent care within the accepted medical practice and to make
treatment decisions which are in the best interest of the patient.
6. Success of treatment: Effective doctor–patient communication has shown a positive
influence on health outcome.
Key points for effective communication skill: The amount of information given to patients
will vary according to the nature of their condition. The important information provided
to patients in the local language includes:
1. The name of the medicines and likely benefits from them.
2. The objective of the treatment.
125
t.me/Dr_Mouayyad_AlbtousH
126 Manual of Prescription Writing
3. Frequency and complexity of drug regimen and the total number of drugs,
instructions for how to use inhalers or transdermal patches.
4. Advice to judge the benefit of the drug.
5. Approximate time expected to get improvement.
6. Duration of therapy and the grounds on which the decision is made to stop or
change the treatment. Premature discontinuation of drugs may cause relapse
especially in the case of treatment of infectious diseases.
7. Inform the most suitable timings to take drugs for better efficacy. For example: PPI
should be taken on empty stomach and NSAIDs should be taken after meals.
8. Ask the patient about the previous allergy to any drugs.
9. Explain what adverse drug reactions (ADR) are possible with prescribed drugs.
Orient them about likely risks, including serious and common side effects. It is
likely that patient will not adhere to the drugs which cause confusion or altered
mental status.
10. Tell them to report immediately, if ADR occurs.
11. Warn them to avoid taking any other drug which may interact with the present
prescription, e.g. avoid alcohol when metronidazole is prescribed. Explain to them
the reason to avoid.
12. Always share the next date of follow up consultation for the review of the treatment.
13. Instruct clearly how to taper drugs before stopping them especially when steroids
are prescribed.
14. Ask them not to stop the drug abruptly when symptoms are relieved, assuming
that there is no need to continue the drug, e.g. beta blockers in hypertension, sudden
withdrawal may result in exacerbation of hypertension.
15. Instruct the patient about which food substances are to be avoided with drugs, e.g.
tetracyclines should not be taken with milk.
16. Convey clearly when not to worry if some changes are noticed under medication,
like urine colour may become dark in case of rifampicin, stool colour may be black
in case of iron.
17. Warn them not to take the drugs written on earlier prescription repeatedly without
consulting the physician. It can be dangerous in the case of habit forming drugs
like codeine in cough mixtures.
18. Do not change the dose strength of the drugs without consulting treating physician
even when there is an improvement, e.g. in case of treatment of infection.
19. To take precautions, e.g. do not drive when taking first generation antihistamines.
20. Inform about the laboratory tests needed, e.g. prothrombin time with oral
anticoagulants.
21. Avoid self-medication.
22. Communicate pregnant patients about the possible teratogenic effects of drugs to
the fetus. Tell them to avoid such drugs in the first trimester of pregnancy, unless
the benefit to the mother outweighs the risk to the fetus. Also mention that growth
of the fetus or functional development may also be affected if drugs are used in the
second and third trimester.
23. Involve the patients in decisions about their care and respect their autonomy.
Discussing therapeutic options will encourage the patient to choose a suitable
alternative and this will prevent them to abscond from getting treated.
t.me/Dr_Mouayyad_AlbtousH
Communication with Patients on Prescriptions 127
t.me/Dr_Mouayyad_AlbtousH
128 Manual of Prescription Writing
t.me/Dr_Mouayyad_AlbtousH
Common Prescription Errors 129
Common
Prescription Errors
129
t.me/Dr_Mouayyad_AlbtousH
130 Manual of Prescription Writing
Sample 1
Date: 05.01.2021
Patient’s name: Mr V
Age & Sex: 30 years, Male
Weight: 55 kg
Address: 24, Modern Colony, Gwalior
Diagnosis: Plague
RX
Inj Streptomycin 1 g IM 12 hourly for 10 days
Tab Paracetamol 500 mg orally 6–8 hrly as required for fever
Refills: Nil XY Jain
Stamp
Sample 2
Dr XY Jain
MBBS, MD (General Medicine)
Registration no. MPMC 223344
25, Park Street, Gwalior MP
Tel. no. 111222 Email: -------@-----.com
Date: 01.01.2021
Patient’s name: Mr B
Age & Sex: Male
Address: 24, Modern Colony, Gwalior
t.me/Dr_Mouayyad_AlbtousH
Common Prescription Errors 131
Sample 3
Dr XY Jain
MBBS, MD (General Medicine)
Registration no. MPMC 223344
25, Park Street, Gwalior MP
Tel. no. 111222 Email: -------@-----.com
Date: 06.01.2021
Patient’s name: Mr X
Age & Sex: 30 years, Male
Weight: 55 kg
Address: 24, Modern Colony, Gwalior
Sample 4
Dr XY Jain
MBBS, MD (General Medicine)
Registration no. MPMC 223344
25, Park Street, Gwalior MP
Tel. no. 111222 Email: -------@-----.com
Date: 07.01.2021
Patient’s name: Mr B
Age & Sex: 50 years, Male
Weight: 55 kg
Address: 24, Modern Colony, Gwalior
RX
Tab. Metronidazole 400 mg orally 8 hrly with meals
Refills: None
XY Jain
Stamp
t.me/Dr_Mouayyad_AlbtousH
132 Manual of Prescription Writing
Sample 5
Dr XY Jain
MBBS, MD (General Medicine)
Registration no. MPMC 223344
25, Park Street, Gwalior MP
Tel. no. 111222 Email: -------@-----.com
Date: 08.01.2021
Patient’s name: Mr V
Age & Sex: 50 years, Male
Weight: 55 kg
Address: 24, Modern Colony, Gwalior
Sample 6 Dr XY Jain
MBBS, MD (General Medicine)
Registration no. MPMC 223344
25, Park Street, Gwalior MP
Tel. no. 111222 Email: -------@-----.com
Date: 05.01.2021
Patient’s name: Mr Y
Age & Sex: 50 years, Male
Weight: 55 kg
Address: 24, Modern Colony, Gwalior
Diagnosis: Pediculosis
RX
Permethrin 1% lotion apply on dry hair
Allow to remain for 10 min then wash with water
Repeat after 7 days if lies detected
t.me/Dr_Mouayyad_AlbtousH
Common Prescription Errors 133
Sample 7 Dr XY Jain
MBBS, MD (General Medicine)
Registration no. MPMC 223344
25, Park Street, Gwalior MP
Tel. no. 111222 Email: -------@-----.com
Date: 08.01.2021
Patient’s name: Mr B
Age & Sex: 50 years, Male
Weight: 55 kg
Address: 24, Modern Colony, Gwalior
Sample 8 Dr XY Jain
MBBS, MD (General Medicine)
Registration no. MPMC 223344
25, Park Street, Gwalior MP
Tel. no. 111222 Email: -------@-----.com
Date: 10.02.2021
Patient’s name: Mr X
Age & Sex: 50 years, Male
Weight: 55 kg
Address: 24, Modern Colony, Gwalior
t.me/Dr_Mouayyad_AlbtousH
134 Manual of Prescription Writing
Sample 9
Dr XY Jain
MBBS, MD (General Medicine)
Registration no. MPMC 223344
25, Park Street, Gwalior MP
Tel. no. 111222 Email: -------@-----.com
Date: 09.01.2021
Patient’s name: Mr A
Age & Sex: 50 years, Male
Weight: 55 kg
Address: 24, Modern Colony, Gwalior
Sample 10 Dr XY Jain
MBBS, MD (General Medicine)
Registration no. MPMC 223344
25, Park Street, Gwalior MP
Tel. no. 111222 Email: -------@-----.com
Date: 10.01.2021
Patient’s name: Mr B
Age & Sex: 50 years, Male
Weight: 55 kg
Address: 24, Modern Colony, Gwalior
t.me/Dr_Mouayyad_AlbtousH
Common Prescription Errors 135
Sample 11
Dr XY Jain
MBBS, MD (General Medicine)
Registration no. MPMC 223344
25, Park Street, Gwalior MP
Tel. no. 111222 Email: -------@-----.com
Date: 02.02.2021
Patient’s name: Mr X
Age & Sex: 50 years, Male
Weight: 55 kg
Address: 24, Modern Colony, Gwalior
Diagnosis: Toxoplasmosis
RX
XY Jain
Stamp
Sample 12 Dr XY Jain
MBBS, MD (General Medicine)
Registration no. MPMC 223344
25, Park Street, Gwalior MP
Tel. no. 111222 Email: -------@-----.com
Date: 05.01.2021
Patient’s name: Mr X
Age & Sex: 50 years, Male
Weight: 55 kg
Address: 24, Modern Colony, Gwalior
t.me/Dr_Mouayyad_AlbtousH
136 Manual of Prescription Writing
Sample 13 Dr XY Jain
MBBS, MD (General Medicine)
Registration no. MPMC 223344
25, Park Street, Gwalior MP
Tel. no. 111222 Email: -------@-----.com
Date: 07.01.2021
Patient’s name:
Age & Sex: 53 years, Male
Weight: 56 kg
Address: 24, Modern Colony, Gwalior
Diagnosis: Gonorrhoea
RX
Inj Ceftriaxone 250 mg IM single dose
Tab Azithromycin 1 g single dose
Tab Paracetamol 500 mg 6–8 hrly as required for fever
XY Jain
Stamp
Sample 14 Dr XY Jain
MBBS, MD (General Medicine)
Registration no. MPMC 223344
25, Park Street, Gwalior MP
Tel. no. 111222 Email: -------@-----.com
Date: 02.01.2021
Patient’s name: Mr A
Age & Sex: 30 years, Male
Weight: 59 kg
Address: 24, Modern Colony, Gwalior
t.me/Dr_Mouayyad_AlbtousH
Common Prescription Errors 137
Sample 15
Dr XY Jain
MBBS, MD (General Medicine)
Registration no. MPMC 223344
25, Park Street, Gwalior MP
Tel. no. 111222 Email: -------@-----.com
Date: 06.01.2021
Patient’s name: Ms V
Age & Sex: 28 years, Female
Weight: 58 kg
Address: 24, Modern Colony, Gwalior
XY Jain
Stamp
Sample 16 Dr XY Jain
MBBS, MD (General Medicine)
Registration no. MPMC 223344
25, Park Street, Gwalior MP
Tel. no. 111222 Email: -------@-----.com
Date:
Patient’s name: Mr A
Age & Sex: 28 years, Male
Address: 24, Modern Colony, Gwalior
XY Jain
Stamp
t.me/Dr_Mouayyad_AlbtousH
138 Manual of Prescription Writing
Sample 1
Error report: Doctor’s particulars like Doctor’s name, professional degrees, and
registration number, address and telephone number are missing. A doctor without a
registration number is not authorized to write a prescription. This is a legal document.
Without doctors details pharmacy personnel are in a dilemma whether the prescription
is written by a registered practitioner or not. A pharmacist is not authorized to dispense
drugs in such prescriptions and if he does so, he may be penalized. The patient cannot
communicate with the doctor in case of any emergency.
Correction tips: Write doctor’s particulars like doctor’s name, professional degrees,
address, telephone number and email address at the top of the prescription.
Sample 2
Error report: Patient’s particulars like age and weight are missing. The age and weight
of the patient are required to confirm appropriate doses of the drugs in the prescription.
Patient’s weight on the prescription can be useful in avoiding dosing errors, particularly
when drugs are administered to paediatric patients. This also helps the pharmacy
personnel to assess the prescription and be confident of dispensing the right drug and
dosage form like syrup and not tablets for infants.
Correction tips: Write the patient’s age and weight after the doctor’s particulars.
Sample 3
Error report: Superscription symbol “RX” is missing. This is a sign written in the name
of God, which is universally accepted. It represents the specific Latin verb “recipe”
which means “take thou”. This symbol separates the superscription from inscription.
Correction tips: Write superscription symbol RX just before inscription, on the left
side.
Sample 4
Error report: Duration of treatment is missing. The patient is not advised for how long
he needs to take the drugs for the cure of amoebiasis. If the patient leaves the treatment
in between chances of recurrence will be there or the patient may take the drugs for
longer than the required duration, result in ADR or increased cost to the patient.
Correction tips: In amoebiasis Tab Metronidazole is given in a dose of 400 mg orally
8 hrly for 10 days.
Sample 5
Error report: The dosage form of medicine is missing and the generic name of the drug
is not used. What should be the route of administration is also not mentioned. The
pharmacist may find it difficult to dispense the correct drug.
Correction tips: Right inscription for this patient of pseudomembranous colitis is: Tab
Metronidazole 500 mg orally 8 hrly for 10 days.
t.me/Dr_Mouayyad_AlbtousH
Common Prescription Errors 139
Sample 6
Error report: The doctor’s signature is missing. Without the signature of the prescribing
physician, the prescription is not legal. The pharmacist cannot dispense drugs. Even if
it is a computer typed prescription, it is considered illegal without the signature of
prescribing doctor and drug addicts can misuse it. A prescription with a signature
also rules out the possibility of photocopied or invalid prescription.
Correction tips: Add signature of the doctor below the inscription.
Sample 7
Error report: Instructions to the patient as to when to take the drug is missing. The
patient should be instructed to take the drug after meals and not to worry if the colour
of the stool is black. The strength of the drug is also not mentioned.
Correction tips: Write instructions and directions at the end of the prescription like
take the drug after meals and colour of the stools may turn black because of iron. Also
mention the dose of the drug—Tab Ferrous Sulfate 325 mg orally once daily after
meals for one month.
Sample 8
Error report: How to taper steroids is not mentioned. Steroids should be tapered
gradually and then stopped. If not withdrawn gradually it can result in exacerbation
of the disease or acute adrenal insufficiency.
Correction tips: Correct way to prescribe steroid is:
Tab Prednisolone 40 mg 12 hourly for 5 days
40 mg once daily for 5 days
20 mg once daily for 11 days
Sample 9
Error report: Preferred dosage form is a capsule, not tablet because tetracycline has
irritant property can cause epigastric pain, nausea and diarrhoea. Tetracycline is not
to be given with milk because of its chelating property. It forms an insoluble and un-
absorbable complex with calcium. If given with milk may lead to treatment failure on
account of lesser drug absorption from the gut.
Correction tips: Ideally the tetracycline capsule should be taken two hours before or
two hours after food. It should not be given with milk and dairy products.
Add supplemental instructions: When prescribing tetracycline, the patient should be
warned to avoid sun exposure to prevent photoallergy.
Sample 10
Error report: The inscription part is not as per guidelines. Polypharmacy is there.
Polypharmacy is defined as the use of five or more medications daily by an individual.
Correction tips: Only Cap Amoxycillin 500 mg orally 8 hrly and Tab Paracetamol
500 mg as required for fever are sufficient.
t.me/Dr_Mouayyad_AlbtousH
140 Manual of Prescription Writing
Sample 11
Error report: Prescription is not legible as it is written in bad handwriting. It may
create confusion and increase the chances of errors. There are chances of wrong drugs
being dispensed as the pharmacist may use guesswork.
Correction tips: The inscription part of this prescription should be written in clear
handwriting or it should be typed.
Tab Pyrimethamine 200 mg orally as the first dose then 75 mg once daily for 6 weeks
Tab Sulfadiazine 1500 mg 6 hourly for 6 weeks
Tab Folinic Acid 10 mg orally once daily for 6 weeks
Tab Paracetamol 500 mg 8 hrly as required for fever
Sample 12
Error report: One very common cause of prescription errors is the use of abbreviations
for the frequency of administration as OD, BD, TDS or SOS. These abbreviations can
be misinterpreted by the patients not knowing Latin. It is recommended that
abbreviations are not to be used at all when writing medication orders.
Correction tips: Frequency of administration is to be written as:
Tab Ciprofloxacin 750 mg 12 hourly for 7 days
Tab Paracetamol 500 mg orally 6 hrly as required for fever
Tab Metoclopramide 10 mg 8 hrly before meals as required for vomiting
Sample 13
Error report: Name of the patient is missing. If the name of the patient is not mentioned,
the prescription cannot be linked to a particular patient with certainty. Complete name
of the patient must be written. A prescription is meant for an individual and if the half
name is written there may be two clients at the pharmacy with the same first/last
names whose prescriptions may get interchanged therefore along with the complete
name, patient’s age and address are important for proper identification of a patient.
Correction tips: Write the complete name of the patient.
Sample 14
Error report: The spelling of the drug’s name is wrong. There is a chance for error by
pharmacist and wrong drug may be dispensed or pharmacist may refuse to dispense
the drug.
Correction tips: The spelling of the drug name should be correct.
Tab Albendazole 400 mg orally single dose.
Sample 15
Error report: Tetracyclines are contraindicated in pregnancy as they have an affinity
with calcium. They are taken up by the growing teeth and bone of the fetus. They
should not be prescribed for pregnant patients because of their potential for permanent
discoloration of teeth in the fetus (yellow or brown in appearance), as well as
impairment of fetal long bone growth.
t.me/Dr_Mouayyad_AlbtousH
Common Prescription Errors 141
Sample 16
Error report: Date of prescription is missing. In this prescription Codeine syrup is
advised for cough which is a drug of dependence. The patient may misuse this
prescription to purchase Codeine syrup several times in absence of mention of date on
the prescription. The prescriber has also not given required important directions to
the dispenser about refills. Writing of date is very important for medicolegal cases.
Correction tips: To write the date of prescription and direction to the dispenser.
“Not to refill until instructed”.
t.me/Dr_Mouayyad_AlbtousH
142 Manual of Prescription Writing
Prescribing for
Special Groups
Dose adjustments are required when there is a change in physiology and/or associated
pathology in an individual on the account of changes in pharmacokinetics and
pharmacodynamics of drugs.
Prescription writing must be appropriate and meticulous as per individual. To write
correct doses in these different conditions are an important part of prescribing skill.
PRESCRIBING IN PAEDIATRICS
Infants and children are not small adults. Just by a proportionate reduction in adult
dose, the optimum paediatric dose cannot be obtained. Paediatric prescribing is much
more complicated than adult prescribing for a number of reasons:
• Pharmacokinetics and pharmacodynamics are different in children and they also
vary between different age groups.
• Clinical trials are rarely conducted in children; so many drugs are used off label.
• Special methods of drug administration are required for infants and young children,
e.g. solid dosage forms like tablets, capsules or metered dose inhalers are difficult
to administer in infants.
• Dose calculations are required and any error in calculation can be more serious
than in adults.
142
t.me/Dr_Mouayyad_AlbtousH
Prescribing for Special Groups 143
Drug Absorption
Gut transit time is slow and gastric acid level is low in children as compared to adults.
The absorption of ampicillin is more efficient in neonates than in adults because of the
higher gastric pH.
Systemic absorption across the skin surface is prompt because the skin is thin and
permeable to drugs.
Milk in the stomach can influence the oral absorption of drugs in neonates and infants.
Rectal absorption is very efficient in neonates, e.g. diazepam solution is given per
rectally to control febrile seizures in children less than 5 years of age.
IM route should be avoided because of less muscle mass, less blood supply and
more pain.
Drug Distribution
Body composition of water and fat content is different in neonates. A neonate has
more total body water than fat that decreases with age. The volume of distribution is
therefore increased for water-soluble drugs so higher doses of water-soluble drugs
are required than adults.
Blood–brain barrier is more permeable to drugs allowing higher concentration in
CNS.
Plasma protein concentrations are lower in neonates so plasma protein binding of
drugs is low, this results in a higher unbound fraction of the drug and increased chances
of adverse effects, e.g. warfarin.
Drug Biotransformation
Different enzyme systems in the liver mature at different rates, overall metabolism is
lower in neonates. Gray baby syndrome with chloramphenicol is seen as the enzyme
glucuronyl transferase for phase II reactions is not adequately formed. Drugs like
diazepam, nalidixic acid, lignocaine and succinylcholine act longer because they are
poorly metabolised. Enzymes maturation increases to adult levels in the first few
months and then exceeds adult rates for the majority of childhood.
Drug Excretion
Kidney function matures by the age of 6–8 months. At birth, a newborn has 33% of the
GFR and renal tubular excretion capacities of an adult so doses of drugs dependent on
renal excretion such as aminoglycosides and penicillins must be adjusted in a neonate.
Plasma half-lives of such drugs are increased by 3 to 5 times.
t.me/Dr_Mouayyad_AlbtousH
144 Manual of Prescription Writing
Age in months
Dose in child = × Adult dose
150
Dilling’s Formula
Age in years
Dose in child = × Adult dose
20
t.me/Dr_Mouayyad_AlbtousH
Prescribing for Special Groups 145
PRESCRIBING IN ELDERLY
The elderly means persons more than 65 years of age. Currently, they represent about
10% of the Indian population. Prescribing in the elderly is challenging because
• The elderly persons experience a greater incidence of diseases and physical
impairments than younger adults, e.g. impaired memory, impaired eyesight,
immobility, lethargy and tremors in hand.
• The elderly suffers more from adverse drug effects and drug-drug interactions than
younger adults because of impairment of homeostatic mechanisms and the effects
of coexisting diseases.
• Compliance is poor in the elderly may be due to impaired hearing, several drugs,
complex drug regimen, etc.
• Polypharmacy is a common problem amongst elderly people and the degree of
polypharmacy increases with the advancing age.
• Vitamins, minerals and nutrients deficiencies are more common in elderly.
• Coexisting diseases like hypertension and diabetes mellitus lead to renal compromise.
Drug Absorption
The absorption rate of drugs is reduced in the elderly due to reduced gut motility and
blood flow to the intestine, with a longer time needed to reach peak concentrations.
t.me/Dr_Mouayyad_AlbtousH
146 Manual of Prescription Writing
Prodrugs which require metabolism to change from an inactive to active form will
have lower bioavailability.
Drug Distribution
Drug distribution can be affected by the changes in body composition, i.e.
proportionally less muscle mass and total body water and more body fat in the elderly.
This can affect the volume of distribution, which may result in either increased plasma
concentrations or half-life of some drugs causing toxicity, e.g. digoxin and lithium.
Protein binding may also change causing an increase in the free fraction of highly
bound acidic drugs and a possible decrease in the free fraction of basic drugs, e.g.
Plasma protein binding of warfarin may decrease.
Drug Excretion
As the age advances there is a reduction in GFR by 6–10% every 10 years after the age
of 40 years. At least a 50% reduction in GFR is seen by the age of 70 years. Creatinine
clearance is markedly reduced in the elderly, so drugs doses have to be reduced, e.g.
Streptomycin 0.75 g after 50 years of age and 0.5 g after 70 years of age is prescribed as
compared to young adults requiring 1 g. Renal plasma flow and active secretion of
drugs may decrease so excretion of renally cleared drugs may decrease, e.g. digoxin,
lithium, penicillin, vancomycin.
t.me/Dr_Mouayyad_AlbtousH
Prescribing for Special Groups 147
• Maintenance doses of renally excreted drugs, e.g. digoxin should be lower than
usual.
• Whenever an elderly patient develops a new or unexplained medical problem after
drug intake, consider adverse drug reaction as a cause, e.g. delirium, hypotension,
arrhythmias, renal failure, electrolyte disorders and constipation.
• Be clear about the diagnosis. Take care not to prescribe a drug to manage an adverse
effect from another drug unless this is unavoidable.
• Think about whether the drug you are considering causes a particular problem in
the elderly patients or has a narrow safety margin, e.g. warfarin. Can it be safely
used in this patient?
• While treating elderly patients it is always needed to check that the drugs prescribed
are appropriate to achieve the aim and if not, stop them. Properly communicate
with the patient and family after reviewing the medications and provide written
directions.
• Considering old age problems like forgetfulness and difficulty in opening the
packaging, keep the drug regimens as simple as possible.
One should not deny prescribing beneficial medicines to elderly patients just because
they are old. Since clinical trials are not performed in elderly patients so there is a
chance that the use of drugs may not provide the same benefit to the old age group as
in the younger population.
Hypnotics
Benzodiazepines should not be routinely prescribed for insomnia. Increased sensitivity
to benzodiazepines is well recognized with increasing age and also there is a possible
increase in half-life, both of which contribute to ‘hangover’ effects seen the next day.
Elderly patients are at increased risk of falls and cognitive impairment. If a
benzodiazepine must be used, choose one with a short half-life such as lorazepam and
use it for the shortest possible time.
Neuroleptic Drugs
Neuroleptic drugs, often prescribed for agitation in the elderly, can cause delirium,
extrapyramidal symptoms and postural hypotension in the short-term and possibly
stroke in the long-term. Avoid these drugs if possible and consider other non-
pharmacological measures.
Analgesics
Opioid sensitivity is increased in the elderly as is a reduction in clearance. Initial doses
of 25 to 50% of normal adult doses are recommended in the elderly. Tramadol works
on serotonin and adrenergic receptors as well as opioid ones, giving it a different side-
effect profile. This includes a reduced seizure threshold and delirium, which may make
it less useful in the elderly. Pethidine should also be avoided.
t.me/Dr_Mouayyad_AlbtousH
148 Manual of Prescription Writing
Digoxin
Digoxin has a narrow therapeutic index and its pharmacokinetics are altered in the
elderly (decreased volume of distribution, reduced clearance), making toxicity a
potential problem. Both loading and maintenance doses should be reduced, with a
maximum maintenance dose of 125 micrograms (half of the normal maximum dose).
Anticoagulants
Elderly patients are more sensitive to the effects of warfarin, and at greater risk of
adverse effects. However, it is also often omitted unnecessarily in elderly patients
with atrial fibrillation who may benefit from it. Particular risks may be: Possible
interactions with antiplatelet drugs, gastrointestinal bleeding and tendency to fall of
patients. If warfarin is used, lower loading and maintenance doses are recommended.
Antihypertensive Drugs
Elderly patients have a lower resting heart rate and decreased tachycardic response to
postural changes. They are therefore more susceptible to the bradycardiac and
hypotensive effects of many drugs. Angiotensin-converting enzyme inhibitors are less
useful in the elderly and may cause increased renal impairment. Calcium channel
blockers may contribute to constipation. Both verapamil and diltiazem have increased
antihypertensive effects in the elderly.
Anticholinergic Drugs
Many drug classes have anticholinergic effects (e.g. antihistamines, antidepressants
and neuroleptics) that can be more pronounced in the elderly. Urinary retention,
constipation and postural hypotension can be more problematic in the elderly. Try to
avoid multiple anticholinergics in combination.
Antibiotics
Antibiotic associated diarrhoea and Clostridium difficile infection are more common in
the elderly. Some other adverse effects of antibiotics like delirium, seizures,
haematological abnormalities and renal damage are also more common in the elderly
than in younger patients.
PRESCRIBING IN PREGNANCY
To balance the risks and benefits of drug therapy during pregnancy is a challenge for
every clinician because pharmacokinetic and pharmacodynamic processes are altered
by pregnancy and any drug administered to a pregnant woman may harm the
developing fetus or influence her pregnancy adversely.
However, pharmacological treatment is unavoidable in pregnancy with some
medical conditions, e.g. asthma, epilepsy and hypertension to prevent complications
in the mother.
t.me/Dr_Mouayyad_AlbtousH
Prescribing for Special Groups 149
t.me/Dr_Mouayyad_AlbtousH
150 Manual of Prescription Writing
• Consider the risk of relapse against potential adversity to fetus and mother.
• Use those drugs that are effective at minimal doses and avoid drugs which have
demonstrated evidence of harm to the fetus during pregnancy, e.g. sodium valproate.
• Keep in mind the changes in pharmacokinetics with the progress of the pregnancy
while prescribing drugs. Dosing intervals may need to be shortened in pregnancy.
For example, once a day agent to control blood pressure may not be enough and
dosing frequency may have to be increased for adequate control of blood pressure.
• Periodic monitoring of drug levels should be done before adjusting doses. Instead
of total serum levels, free drug levels are better guides.
Effects of drugs on pregnancy: In general congenital anomalies occur in 2–4% of all
live births.
t.me/Dr_Mouayyad_AlbtousH
Prescribing for Special Groups 151
t.me/Dr_Mouayyad_AlbtousH
152 Manual of Prescription Writing
t.me/Dr_Mouayyad_AlbtousH
Prescribing for Special Groups 153
PRESCRIBING IN LACTATION
In mothers who are breastfeeding, consideration of whether the child will be exposed
to the drug, and what consequences this may have, is important. Unfortunately, data
on these aspects may not be available because of the lack of clinical trial in breastfeeding
patients. Most data comes from observational studies, and so there may be more
information about older drugs.
Many substances ingested by lactating women are present in breast milk, although
the levels tend to be less than in utero. It is important to know both the likely
concentration and the potential effects in order to estimate risk. Toxicity to infant can
occur if the drug enters the milk in significant quantities. The concentration of some
drugs in milk may exceed that in the maternal plasma, e.g. iodides so that the
therapeutic dose in the mother can cause toxicity to the infant.
Some drugs may inhibit the suckling reflex in infants, e.g. phenobarbitone.
t.me/Dr_Mouayyad_AlbtousH
154 Manual of Prescription Writing
Impaired Absorption
Bioavailability of lipid soluble drugs may be reduced in cholestasis.
Impaired Metabolism
Liver disease may reduce the capacity to metabolise certain drugs leading to toxicity.
This is caused by both the reduction in the metabolic capacity of the liver and the
changes in liver blood flow. The dose of drugs that have a high hepatic extraction ratio
(high first pass metabolism) needs to be reduced. On the other hand, prodrugs, e.g.
captopril might become ineffective.
Impaired Excretion
In hepatobiliary disease, the biliary system is obstructed affecting excretion of some
medicines and may lead to accumulation of drugs and toxicity. Renal impairment can
also occur as a consequence of liver disease, e.g. in cirrhosis there is a reduction in
glomerular filtration rate so dose adjustment is important for renally cleared drugs.
Altered Pharmacodynamics
Complications of liver disease such as hepatic encephalopathy, ascites and deficiencies
in clotting factors can increase sensitivity to particular drug effects. Drugs that cause
sedation, constipation or hypokalaemia, if given in severe liver disease can cause
encephalopathy. Drugs that cause fluid retention, e.g. nonsteroidal anti-inflammatory
t.me/Dr_Mouayyad_AlbtousH
Prescribing for Special Groups 155
drugs and prednisolone if given in ascitic patients may worsen it. Anticoagulants will
require careful monitoring and adjustment in severe liver disease as clotting factors
are usually depleted.
t.me/Dr_Mouayyad_AlbtousH
156 Manual of Prescription Writing
• Aspirin and NSAIDs: Because synthesis of clotting factors from liver is decreased,
there is increased risk of bleeding with aspirin, whereas fluid and water retention
with NSAIDs can increase ascites.
• Anticoagulants: Synthesis of clotting factors is reduced so the effect of warfarin may
increase. Thrombin inhibitors may increase bleeding risk in liver disease for similar
reasons.
• Antidiabetic drugs: There is an increased risk of lactic acidosis with metformin therapy.
An increased risk of hypoglycemia is seen with sulphonylureas. Glitazones should
be avoided because of the potential for hepatotoxicity.
• Opiates and benzodiazepines: May precipitate encephalopathy.
Impaired Distribution
Protein binding of drugs may be altered because of hypoalbuminaemia (caused by
proteinuria) or uraemia. Highly protein bound drugs with narrow therapeutic index
are likely to cause toxicity due to reductions in protein binding.
Impaired Metabolism
Certain drugs are metabolized in the kidney and may be affected by various types of
renal diseases. Drug metabolism in the liver tends to be slower in chronic renal disease.
Many active metabolites of drugs depend on the kidney for their removal. They increase
in renal impairment thereby increasing the incidence of ADR.
Impaired Excretion
The kidney excretes drugs through the processes of glomerular filtration, active tubular
secretion and passive tubular reabsorption. All these functions are reduced in renal
disease. Drugs that are renally cleared may require dose adjustment or avoidance.
Pharmacodynamic Changes
Sensitivity to certain drugs is altered in renal diseases leading to various adverse drug
effects.
t.me/Dr_Mouayyad_AlbtousH
Prescribing for Special Groups 157
Nephrotoxicity by Drugs
Many commonly used drugs are nephrotoxic at intrinsic or post-renal levels. Many
drugs cause intrinsic renal damage by acute tubular necrosis or acute interstitial
nephritis. Post-renal obstruction can be occasionally drug induced. Nephrotoxic drugs
should be avoided if possible in patients with renal disease as they may cause a further
reduction in renal function. Examples of nephrotoxic drugs include: Aminoglycosides,
amphotericin B, NSAIDs, sulphonamides and cyclosporine.
t.me/Dr_Mouayyad_AlbtousH
158 Manual of Prescription Writing
t.me/Dr_Mouayyad_AlbtousH
Critical Appraisal of Prescription and Prescription Audit 159
6
Critical Appraisal
of Prescription and
Prescription Audit
Prescription Audit
It is a systematic review for quality improvement process with predefined criteria and
proper use of available resources to deliver the right medicine to the right person in a
health care system. Prescription auditing is also an educational activity that should be
done regularly.
Objectives of Audit
1. To evaluate the effectiveness of a therapy
2. To prevent problems related to medication
159
t.me/Dr_Mouayyad_AlbtousH
160 Manual of Prescription Writing
t.me/Dr_Mouayyad_AlbtousH
Critical Appraisal of Prescription and Prescription Audit 161
Note: The WHO/ INRUD have provided a list of medicines which should usually be categorized into
the group of antibiotics and have advised that where researchers deviate markedly from this
categorization, this should be mentioned in the study’s methodology. The WHO classification of
antibiotics has been outlined in Table 6.1. The determination should be made whether dermatologic
creams and eye care products should be regarded as antibiotics.
B. Patient-care Indicators
• Average consultation time
• Average dispensing time
• Percentage of drugs actually dispensed
• Percentage of drugs adequately labelled
• Patients’ knowledge of correct dosage.
t.me/Dr_Mouayyad_AlbtousH
162 Manual of Prescription Writing
t.me/Dr_Mouayyad_AlbtousH
Legal and Ethical Aspects of Prescribing Drugs 163
t.me/Dr_Mouayyad_AlbtousH
164 Manual of Prescription Writing
Doctor’s Name:
Qualification (e.g. MBBS,MD)
Reg. no.:....................................... (ALLOPATHY)
Full Address, Contacts: (Telephone No. E-mail etc.)
Date:
Name of Patient …………………………………………………………………………........
Address*……………………………………………………………………………………………
Age and Sex………………………………………………Weight**……………...................
RX
1. Name of Medicine***
Strength, dosage instruction, duration, and total quantity***
2. -do-
3. -do-
Doctor’s Signature
Stamp
DISPENSED
Date …………………Pharmacist………………….
City …………………………………………………
Minimum size of the Prescription blank should be (a) 14 × 21 cm (A5 size) and
(b) XI × XI cm
t.me/Dr_Mouayyad_AlbtousH
Legal and Ethical Aspects of Prescribing Drugs 165
t.me/Dr_Mouayyad_AlbtousH
166 Manual of Prescription Writing
E-Prescribing
Benefits of E-prescribing
1. It is potentially convenient and advantageous in eliminating subjective errors due
to illegible handwriting and incorrect medicine dispensing.
2. Electronic prescribing potentially ameliorates the adverse drug events and drug-
drug interactions in the ambulatory care settings due to prescription check software
which provides automated alerts and reminders at the time of prescribing.
3. E-prescribing systems automatically highlight patient allergies making prescribers
aware of them.
4. Each prescription can be linked with a high-quality database to check that the drug
prescribed is appropriate for a particular age, gender, comorbidity and laboratory
investigation reports so that patient safety is ensured by reducing fatal mistakes.
5. E-prescribing systems prompt the prescriber to choose the most appropriate drug
available and quickly add common instructions, leading to faster prescribing and
dispensing correct doses.
6. Prescribers are enabled to receive a cost-effective recommendation of drugs and
patients get financial benefits.
7. E-prescribing eliminates the risk of tampering or altering prescriptions.
8. Chances of incorrect interpretation of electronic orders by a pharmacist are very
less.
9. Increased access to patient medication records, and improved pharmacy workflow.
10. Easy refilling of medicine from pharmacy by getting clinician's permission
electronically particularly in cases of chronic diseases reduces the waiting time of
the patients.
11. Mobile devices and wireless networks enable prescribers to modify or edit
prescriptions from the places wherever they are.
166
t.me/Dr_Mouayyad_AlbtousH
E-Prescribing 167
Challenges of E-prescribing
1. The high initial cost to establish and implement an e-prescribing system.
2. Additional efforts are required to train all concerned members to adopt effective
integration of this technology with the office management, scheduling system, billing
systems, and electronic medical records for successful e-prescribing.
3. Protection of the security and confidentiality of medical records and personal
identifiable information (PII) of the patient is of extreme importance.
4. Poor designing of e-prescribing software can cause unintended harm to the patients.
Though electronic prescribing is safe and efficient, it should not be considered as a substitute
for personal attention to the individual patient by the health professional.
t.me/Dr_Mouayyad_AlbtousH
168 Manual of Prescription Writing
Background
Prescription of a licensed medicine for a use that is not approved by a regulatory
agency and not included in the drug label or in the product information is off label
prescribing. The practice is common and legal in many countries. The regulatory bodies
do not have the legal authority to regulate the practice of the medicine, and the physician
may prescribe a drug off label for standard care to the patients.
Off label prescribing includes the use of an approved drug for the following
unlabeled conditions:
• Indication, e.g. anticancer drug mitomycin is indicated for the treatment of gastric
and pancreatic carcinoma also used for the treatment of lung, bladder, breast,
cervical, and other carcinomas not approved by the US FDA.
• Age group: Lack of paediatric indications on drug labels often lead to off label
prescribing, hence many drugs indicated for adults are also prescribed off label in
the paediatric population.
• Dose strength: When a drug is approved at a dose of 500 mg but prescriber advises
other than approved dose.
• Route of administration, e.g. drugs approved by IV route and given by intrathecal
route e.g. pentazocine, gentamicin.
• Dosage form: When a medication is approved as a tablet or capsule and is used
topically or when tablets are given as suspension to children.
168
t.me/Dr_Mouayyad_AlbtousH
Off Label Prescribing 169
indications. Many new therapies and evidence have emerged out of off label
prescribing, however, for pharmaceutical companies to get approval for new uses of
old drugs by clinical testing is time-consuming and costly. Some such off label uses
are scientifically valid giving treatment benefits to patients. Paclitaxel initially approved
for treating ovarian cancer, was found to be effective for treating breast cancer when
used off label. After the publication of reports showing its effectiveness in breast cancer,
it was later approved by regulatory bodies. Gabapentin an antiepileptic drug is used
for bipolar disorder, fibromyalgia, headache, hot flashes, and restless leg syndrome as
off label.
On the other hand, inappropriate use of off label drugs is a cause of concern when
it is used without risk-benefit analysis. The safety with off label use has attracted much
attention when caused serious adverse effects. For example “Fen-phen” an unapproved
combination of fenfluramine and phentermine used for weight loss resulted in heart
valve disease.
Though off label drug use is very common throughout the world, most often this is
done without adequate scientific data. To date, there is no uniform regulatory
framework available to help clinicians to assess the appropriateness of off label
prescribing.
t.me/Dr_Mouayyad_AlbtousH
170 Manual of Prescription Writing
4. The patient asks for a formulation of a drug that is not specified in an applicable
license.
5. Due to a temporary shortage in supply of a suitable licensed medicine, other suitable
approved drug which is not indicated for this condition is used to treat the patient.
6. The prescribing forms part of a properly approved research project.
7. In case of emergency when there is an urgent need for a new drug or indication,
and there is a report of nonefficacy of labeled drug in certain demographics.
8. In cases when label drug is costly for the patient and physician prescribes another
drug which is cost effective though not having a label for that particular use. For
example, bevacizumab a low-priced drug approved for the treatment of metastatic
colorectal cancer is used off label for age-related macular degeneration in place of
approved drug ranibizumab which is very costly.
t.me/Dr_Mouayyad_AlbtousH
Bibliography 171
Bibliography
1. Ackermann E, Litt J and Morgan M. Prescribing Drugs of Dependence in General Practice, Part
C1: Opioids (2017).
2. Aronson JK, Henderson G, Webb DJ and Rawlins MD. A Prescription for Better Prescribing
(2006).
3. Bennet PN, Brown MJ, Sharma P. Clinical Pharmacology, 11th edn, 2012, Charchill Livingstone,
Edinburg.
4. Buxton ILO. Principles of prescription order writing and patient compliance. Goodman and
Gilman’s The Pharmacological Basis of Therapeutics, 12th edn, 2011, McGraw-Hill, New York.
5. Cash JC and Glass CA (Eds.). Family Practice Guidelines, 2017, Springer Publishing Company.
6. De Vries TPG, Henning RH, Hogerzeil HV, Fresle DA, Policy M, and World Health Organization.
(1994). Guide to Good Prescribing: a practical manual (No. WHO/DAP/94.11).
7. Guidelines of Programmatic Management of Drug Resistant Tuberculosis In India 2019 by
Revised National Tuberculosis Control Programme, Central TB Division, Directorate General
of Health Services, Ministry of Health and Family Welfare, Nirman Bhawan, New Delhi.
8. Hitchings A, Lonsdale D, Burrage D and Baker E. The top 100 drugs: Clinical Pharmacology
and Practical Prescribing (2018). Elsevier Health Sciences.
9. Hogerzeil HV (editor), et al. Teacher’s Guide To Good Prescribing (No. WHO/EDM/PAR/
2001.2).
10. International Ethical Guidelines For Health Related Research Involving Humans by CIOMS in
collaboration with WHO 2016.
11. Jameson LJ, Fauci A, Kasper D, Hauser S, Longo D & Loscalzo J. Harrison's Principles of Internal
Medicine, 20 th edn, 2018 (Vol.1 & Vol.2), McGraw-Hill Education.
12. Katzung BG, Masteres SB, Trevor AJ (Eds): Basic and Clinical Pharmacology, 14th edn, 2018,
New Delhi, McGraw-Hill, Education (India).
13. Kennan K, Mathiharan K (editors). A Textbook of Medical Jurisprudence and Toxicology, 25th
edn, 2016 Lexis Nexis, Reed Elsevier, Gurgaon, Haryana, India.
14. National Formulary of India (NFI 2016). Indian Pharmacopoeia Commission, Ministry of Health
and Family Welfare, Directorate General of Health Services, New Delhi, 2015.
15. National List of Essential Medicines (NLEM 2015) Govt of India, Ministry of Health and Family
Welfare, Directorate General of Health Services, New Delhi, 2015.
16. Papadakis MA, McPhee SJ and Rabow MW. Current Medical Diagnosis and Treatment, 2020,
Lange McGraw-Hill, New York.
17. Proposed guidelines for prescription writing and handling for drugs having misuse or abuse
potential, proposed by FDA, Goa at the stakeholders meeting held on 23rd September, 2009.
18. Rang HP, Ritter JM, Flower RJ, Handerson G. Rang and Dale's Pharmacology, 8th edn, 2015,
London: Elsevier Health Sciences.
171
t.me/Dr_Mouayyad_AlbtousH
172 Manual of Prescription Writing
19. Teichman PG and Caffee AE. Prescription writing to maximize patient safety. Family Practice
Management, vol 9 no. (7), 2002.
20. VenkateshS, Chauhan LS, Gadpayle AK, Jain TS, Ghafur A and Wattal C. National treatment
guidelines for antimicrobial use in infectious diseases. India: National Centre for Disease Control,
MOHFW, Government of India, (2011). 1–64.
21. WHO. How to Investigate Drug Use in Health Facilities: Selected Drug Use Indicators: EDM
Research Series No. 007. 1993. [Last accessed on 2015 May 05]. Available from: http://
www.apps.who.int/medicinedocs/en/d/Js2289e/
22. Woodfield G, Phillips BL, Taylor V, Hawkins A and Stanton A. Essential Practical Prescribing.
(2016). John Wiley & Sons.
t.me/Dr_Mouayyad_AlbtousH
Index 173
Index
t.me/Dr_Mouayyad_AlbtousH
174 Manual of Prescription Writing
Clarithromycin 29, 97, 99, 101, 115 Erythromycin 10, 33, 44, 47, 51, 67, 68, 99, 137,
Clavulanic acid 36, 130 152t, 156
Clindamycin 47, 69, 70, 102, 104, 115, 156 Escitalopram 41, 170t
Clobazam 49, 127t Eszopiclone 62
Clopidogrel 17 Ethambutol 72, 73, 105, 152t, 158
Clotrimazole 90, 114, 124, 152t Ethionamide 72, 82, 156
Cloxacillin 18 Ethosuximide 9
Colchicine 16, 37, 156 Ethyl alcohol 77
Contraception 38, 45
Famciclovir 53
Cryptococcal meningitis 39 Febuxostat 37
Cycloserine 73 Ferrous fumarate 63
Cyclosporine 157 Ferrous gluconate 63
Dabigatran 40 Ferrous sulphate 63, 133
Dalteparin 40 Fexofenadine 22
Dapsone 82, 95 Fidaxomicin 104
Darifenacin 121 Filariasis 48
Deep vein thrombosis 40 Fluconazole 39, 87, 90, 114, 124
Depression 31, 41, 152t, 170t Flucytosine 39
Desloratadine 22 Fluoxetine 41, 86
Dexamethasone 84, 134 Fluticasone propionate 22
Dextrose 39, 59, 77, 89 Fluvoxamine 86
Folic acid 74, 106, 150t, 152t
Diabetes mellitus type II 42
Folinic acid 115, 140
Diabetic ketoacidosis 43
Fomepizole 77
Diazepam 17, 110, 127t, 143, 145, 146, 152t
Fosphenytoin 110
Diclofenac 92
Frovatriptan 78
Diethylcarbamazine 48
Furosemide 64
Diiodohydroxyquin 12
Diloxanide furoate 12, 152t Galantamine 23
Diltiazem 113, 148 Gamma benzene hexachloride (BHC) 107
Dimenhydrinate 75, 152t Generalized tonic clonic seizures 4, 49, 110
Dinoprostone 103, 123 Gentamicin 13, 20, 32, 61, 66, 100, 118, 152t, 168
Diphtheria 44 Glucagon 59
Disulfiram 21 Glucose 43, 103, 123
Divalproex sodium 9, 31, 170t Glyceryl trinitrate (GTN) 57
Dolutegravir 54 Gonorrhoea 50, 136
Donepezil 23 Granuloma inguinale 51
Dopamine 64
Doxycycline 10, 11, 26, 29, 32, 34, 50, 51, 67, 69, H. influenzae pneumonia 52
70, 98, 101, 111, 118, 137 Haloperidol 108, 153
Dutasteride 30 Herpes simplex 53
HIV 54, 102, 152t
Edoxaban 40 Hydrochlorothiazide 93
Efavirenz 54 Hydrocortisone 24, 113
Emergency contraception 45 Hydroxocobalamin 74
Enoxaparin 17, 40 Hyoscine 81
Entacapone 94 Hypercholesterolemia 55
Enterobius vermicularis (pinworm) 46 Hypertension in pregnancy 56
Erysipelas 47 Hypertensive emergency 57
t.me/Dr_Mouayyad_AlbtousH
Index 175
t.me/Dr_Mouayyad_AlbtousH
176 Manual of Prescription Writing
t.me/Dr_Mouayyad_AlbtousH
Index 177
Trimethoprim 10, 19, 35, 51, 52, 66, 85, 99, 102, Vaginal candidiasis 124
115, 119, 120, 133 Valacyclovir 53
Tularemia 118 Vancomycin 61, 104, 146, 158
Typhoid fever 119, 135 Venlafaxine 41, 170t
Zidovudine 152t
Uncomplicated UTI 120 Zolendronate 93
Urge incontinence (overactive bladder) 121 Zolmitriptan 78
Urinary retention (postoperative) 122 Zolpidem 62
Uterine inertia 123 Zopiclone 62
t.me/Dr_Mouayyad_AlbtousH
Reader’s Notes
t.me/Dr_Mouayyad_AlbtousH
Reader’s Notes
t.me/Dr_Mouayyad_AlbtousH
Reader’s Notes
t.me/Dr_Mouayyad_AlbtousH
t.me/Dr_Mouayyad_AlbtousH